Nothing Special   »   [go: up one dir, main page]

Oceanic Question Bank Year 4

Download as pdf or txt
Download as pdf or txt
You are on page 1of 358

1 Oceanic Series End of Posting Compilation

Oceanic series
st
QUESTION BANK 1 Edition
YEAR 4: SPECIALIZATION
7/9/2021 – Cik Syurga, Derp, YoRHa, Aaron P, Agent K, N

1
2 Oceanic Series End of Posting Compilation

From the desk of the Chief Editor


When you are reading this, I am sure you are nearly towards the end of this journey. Keep
holding on because the light at the end of the tunnel is there. Do not be afraid, your futures
are yours to hold, and nobody else’s. Do your best and pray the best.
This project initially began as an initiative to ease our revisions by simplifying various
questions throughout years of compilations. When the pandemic hits during this project’s
nascent state, we felt the link between us, and our juniors are lacking. Therefore, we hope
this project will be one of the many bridges between us and carried further as a continuous
effort.

We hope our project will be continued by our juniors as a legacy for our time here. May our
efforts carry you towards excellence in the upcoming professional exam.
All rights are not reserved.

Collaborators, 2021 1st edition


Anaesthesiology - YoRHa
Dermatology - Derp
Primary Care – Cik Syurga
Ear Nose & Throat - Aaron P
Psychiatry – Agent K
Ophthalmology - N
And those who memorized throughout the years.

2
3 Oceanic Series End of Posting Compilation

Oceanic Question Bank 1st Edition


PRIMARY CARE

3
4 Oceanic Series End of Posting Compilation

PRIMARY CARE PRINCIPLE


1. Mr Wong, a 67-year-old man was treated by his family Continuity of care
doctor for the past 20 years. Recently, he was referred to
a cardiologist for heart attack, but he is still in contact with
his family doctor, and he trusted his family doctor.
2. Dr Lee is a primary care doctor and usually catered to Primary prevention
diabetic and hypertensive patients. To prevent coronary
artery disease from developing in his patients, he
constantly optimized the medication for diabetes and
hypertension
3. A 5-year-old boy had uncontrolled bronchial asthma and Family-oriented
was given nebulizer several times. Despite the treatment care
given, he still had recurrent asthmatic attack. He was then
referred to a specialist. The specialist then went for a home
visit and saw that the parents are shouting and quarrelling
with each other in front of the child.
4. The primary care doctor makes an early diagnosis taking Holistic care
account physical, psychological,
social and spiritual factor.
5. Alma Ata Conference: Essential care based on practical, Primary Health
scientifically sound & socially acceptable methods & Care
technology, made universally accessible to individuals &
families in community
6. Came to GP with chief complain of fever, but then he Gatekeeper health
worried he has a heart disease. So, he asked the GP care
doctor to write a referral letter to hospital for admission. But
then the Dr said no because his condition does not need
any admission.
7. A GP, who is a solo practitioner, wants to review his Medical audit
management plan on his patients. He goes through his
diabetes patients to check if he has given the best
treatment. This way is known

4
5 Oceanic Series End of Posting Compilation

INVESTIGATION/ PROCEDURES
8. A 57-year-old woman with history of left mastectomy 2 Mammography
year ago. She currently complained of pain at the right
breast. There was no swelling, no redness, and no fever
9. Madam Mariam, a 60-year-old woman was presented with Thyroid function
palpitation. She also recently lost weight about 2 kg test
despite her increased appetite. On examination, it was
found that she had atrial fibrillation.
10. A 24-year-old woman presented with vaginal bleeding. Cervical smear
She was on oral contraceptive pill for several years. On
examination, she had ectropion with no contact bleeding.
11. Woman worried of ovarian ca. Doctor advised not to do Ca-125
this screening test
12. A 35-year-old lady with history of multiple sexual partners Colposcopy
presented with intermittent vaginal bleeding. Her Pap
smear showed high squamous intraepithelial lesion.
13. A 50-year-old with irritable bowel disease wanted to screen Colonoscopy
for colon cancer. What test do the doctor suggest?
14. Asymptomatic 50-year-old woman, with average risk Faecal occult blood
requested for colorectal CA screening. Doctor
recommends grade A test from the US Preventive Task
Force (USPTF).
15. This tumour marker increases in colorectal cancer. Carcinoembryonic
However, it also increases the tumour of other organs antigen (CEA)
16. Tumour marker elevated 80% in epithelial ovarian cancer CA 125
17. 22 y/o female, having palpitations at night for 2 months. Holter ECG
Blood test and ECG was normal. what investigation would
you like to order for her?
18. During a visit by KK’s school health team, a standard 1 Tumbling E chart
student was found unable to read the writings of the
teachers due to blurred vision.
19. The boy with the same scenario above is found to be big Anthropometric
in size for his age. measurement

5
6 Oceanic Series End of Posting Compilation

20. The school health nurse was confirmed that Nazri was Referral
small. This would be the next step taken by the nurse
PSYCHOLOGICAL ISSUE
21. A 55-year-old lady was involved in a car accident 3 months Posttraumatic
ago. In that accident, she sustained head injury and her disorder
cousin died. Since then, she had been having recurrent
flashbacks and dreams about that accident. She had been
avoiding the street were the accident occurred. She had
also been feeling guilty and loss of interest. She had
become more irritable and was unable to sleep.
22. Mrs Y had recently loss her husband 4 months ago due to Abnormal grief
sudden death. Since then, she cried every day. She felt it reaction
was impossible to manage things without her husband.
23. Madam Maria presented with difficulty of using her right Multi-infarct
and stiffness of her left leg. She also complained of she is dementia
being forgetful, and her memory worsen over the past 2
months. She is currently on diabetes and hypertension
treatment for the past 30 years.
24. Mr Daud recently forgot his 70th birthday. He also Depression
complaints of listlessness most of the time and had
recently lose some weight. He is physically active but used
to smoke in his younger days. He stopped smoking when
his younger brother who had hypertension asked him to.
25. 35 y/o lady, fearful after her friends died of sudden heart Panic disorder
attack. She developed sudden shortness of breath,
palpitation, sweating and feels like collapsing.
26. 78 y/o man had difficulty takes care of himself. He was still Problem with daily
not clean after bathed himself. functioning.
27. Mrs A, 70 years old lady presence with progression Alzheimer
memory loss. She has difficulty finding her own bedroom
and forgets the names of her own children. No significant

6
7 Oceanic Series End of Posting Compilation

past medical and surgical history and is not on medication.


She was able to control her bladder and bowel.
28. A 9-year-old boy presenting with chronic cough that started Psychogenic
as soon as new school term started. The cough did not
improve despite various medications given by doctors. It
was worst in the morning and improved as the day
progressed to evening. The cough did not disturb his sleep
in the night. On physical examination, there was no
remarkable abnormalities, but bruises were found on his
leg which he claimed that he sustained them while playing
games at school.
HEAD INJURY
29. Mr X is an alcoholic. He kept mixing up day and month Subdural
since a month ago. He became slower and often unsure haematoma
where to go. It was noticed that he often had difficulty in
talking. He has hypertension and diabetes mellitus but
currently under control.
DEFINITION
30. The amount of previous screening which is diagnosed and Yield
brought to treatment as screening program shown
asymptomatic.
31. Probability of person have disease when the screening test Positive predictive
is positive. value
PEDIATRICS
32. An 8-month-old girl presented with cough and fever for 4 Measles
days and was irritable. Her temperature was 40.5oC. On
further examination, rhinorrhoea and conjunctivitis were
present with generalized erythematous blanching
maculopapular rash on face and torso on the first day of
fever.
33. A 3-year-old boy presented with fever, sore throat and rash Scarlet fever
on the second day of fever. On examination, he had

7
8 Oceanic Series End of Posting Compilation

pharyngitis, conjunctivitis, erythematous maculopapular


blanching rash and circumoral pallor.
34. A child develop fever when came back from his hometown. Typhoid fever
Had fever for 9 days, maculo erythematous rash over the
truck. Spleen can be felt.
35. 6mths-2 years old age, sudden high fever (3-4 days), Roseola infantum
febrile convulsion, red rash appears after child recovered
from high fever usually begins on trunk, spreading to legs
and neck, rashes not itchy and may last 1 to 2 days.
36. A 2-year-old child presented with fever and bilateral Kawasaki disease
conjunctival redness. On physical examination, there were
red lips, mouth & tongue and desquamation of palm and
sole.
37. 7 yrs. old boy come with abdominal pain and difficulty of Henoch schonlein
walking because of painful ankle. On examination, the purpura
child is afebrile and have symmetrical non blanching
palpable lesion on buttock and extensor surface of limb.
The ankle is warm and tender.
MAN’S STD
38. A 24-year-old man presented with painless papules on the Treponema
shaft of his penis. The papules developed into superficial pallidum
ulcers after 3 weeks
39. Presented with umbilicated papules Molluscum
contagiosum
VACCINE HANDLING
40. The pharmacist ensures the vaccines are labelled with Inventory
name and expiry date. information

41. The medical students are asked to see the vaccine Vaccine handling &
monograph. use

OBSTETRICS AND GYNAECOLOGY

8
9 Oceanic Series End of Posting Compilation

42. Presented with PV bleed. On examination, presence of Cervical polyps


pedunculated mass.
43. A 23-year-old woman who had multiple sexual partners Herpes simplex
has malaise, dysuria, and multiple erythematous ulcers at virus
labia.
44. 20 y/o lady presented with dysuria and purulent urethral N. gonorrhoea
discharge; microbial investigation shows gram negative
cocci.
45. Pumi, 8 weeks amenorrhea. lower abdominal pain + Inevitable
vaginal bleeding. positive UPT 2 weeks ago. abdominal miscarriage
examination normal. VE - active bleed, OS open. u/s:
collapsed gestational sac
46. 28-year-old primigravida (8 weeks). Pain left lower region, Ectopic pregnancy
no bleeding. Ultrasound uterus empty. Examination
tenderness at left lower region.
47. A lady with a history of breast cancer, had severe left-sided Metastasis of
throbbing headache. On fundoscopy, there was cancer
papilledema
DRUGS & ITS REACTIONS
48. Cardio selective beta-adrenergic blocker. Decrease Atenolol
rennin. Given once daily.
49. Elderly with isolated hypertension and gouty arthritis. Amlodipine
50. An elderly man with BPH needs drug to treat his mild HTN Alpha blocker
51. This drug is effective to treat elderly with hypertension but Hydrochlorothiazide
can cause hyperuricemia and hypokalaemia.
52. This patient was on drug A for his hypertension. He Telmisartan
developed persistent cough since started on drug A. The
doctor changed drug A to drug B since it has the most
similar mechanism as A with less side effects. What is drug
B?
53. This patient was given antihypertensive drug X. Later he Moduretic
develops hypokalaemia. The doctor then gives him a
potassium-sparing agent, drug Y. What is drug Y?

9
10 Oceanic Series End of Posting Compilation

54. 26 years old lady presented with itchiness. On Drug induced


examination, there was urticaria of various sizes over limb pruritus
and body. She was prescribed with sulphamethaxazole
and trimethroprim for UTI for 4 days.

55. 52-year-old type 2 DM already maximum dose of Sitagliptin (DPP4


metformin. Diabetes is not controlled but he is compliance. Inhibitors)
BMI is 29. GP decided to add on second medication which
will not cause weight gain.
56. 45 year old, a mechanic, type 2 DM, on metformin 1 g TDS, Metformin SR
uncontrolled DM. he admits he forget to take afternoon
dose. He is overweight, the rest of examination is okay.
The dr decided to switch to this medicine.
57. 70 years old hypertension, on perindopril 8mg 0D, CCB-
hydrochlorothiazide 50mg OD, came for a visit, BP 120/90 dihydropyridine
same with last visit. Pulse 78. JVP not raised, chest clear,
normal heart sound.
58. Chronic hypertension already on perindopril, presenting Loop diuretic
with sob on exertion, orthopnoea, peripheral edema, chest
crackles bibasal has pitting oedema. What medication are
you going to add?
59. 25 y/o women comes with the c/c of runny nose for 3days. Polypharmacy
Medication prescribed. Syrup diphenhydramine 10 ml
TDS, Tab Chlorphenamine 4mg TDS.
ETTICS
60. A woman had depression after divorce, went to a doctor Non-sexual
for treatment. She got treatment for one year and stop. boundary crossing.
After 2 years, she met the doctor at the beach during
holiday, then they start dating.
61. 26-year-old drug addict with HIV just engaged with a lady Confidentiality
from Sarawak. He does not want his fiancée and the family
know about his condition since they are planning to get
married at the end of the year. The doctor felt helpless.

10
11 Oceanic Series End of Posting Compilation

62. Doctors cannot have a sexual or improper emotional Sexual boundary


relationship with their patient, or someone related to them. crossing
This is an example of
63. A 40-year-old lady with many underlying diseases came to Nonsexual
your clinic and always bring you and your stuffs food. You boundary crossing
prioritize her for appointment as she is your favourite
patient.
64. Doctor refused to treat homeless patient Justice
RESPIRATORY & CARDIOVASCULAR SYSTEM
65. A 19-year-old male had cough and mild fever. On physical Community
examination, there was occasional Ronchi on auscultation. acquired
Chest X-ray shows perihilar patchy opacities pneumonia
66. 23 years old female working in a factory handling with Occupation induced
solvents. She developed cough and difficulty in breathing asthma
few days after that. On examination, there was presence
of bilateral expiratory Ronchi. She was well before.
67. For the lung function test to be reliable, the manoeuvre that Deep breath in
needs to be performed before the patient breathes out.
68. Immigrant, chronic cough, haemoptysis, fever chills and TB
rigor with night sweat – tuberculosis
69. 8 y/o chronic cough, not relieved by syrup and antibiotic, Asthma
eczema on extremities
70. 50 y/o woman chest tightness at night for 5 mins. No family Prinzmetal angina
history of CVD, smoking 20 cigarettes/ day.
71. Woman with history of using OCP, presenting with sudden Pulmonary
onset chest pain which aggravated by breathing. embolism
72. A 30-year-old gentleman complained of gradual onset Pericarditis
retrosternal chest pain associated with breathlessness for
2 days. No history of trauma. The pain in stabbing in
nature, radiated to neck and left shoulder, aggravated by
lying flat and relieved by sitting and leaning forward
FEVER & HEADACHE

11
12 Oceanic Series End of Posting Compilation

73. 20 years old male just came back from jungle trekking. He Eschar Typhus
has fever and headache. Examination revealed fever
splenomegaly.
74. 24 years old woman had throbbing headache. It was Migraine
associated with nausea, vomiting, and light flaring right
visual field on both eyes. Other examinations are normal.
75. A 60-year-old woman presented with bitemporal headache . Giant cell arteritis
and blurring of vision. She also had pain when combing
her hair. Investigation revealed raised erythrocyte
sedimentation rate
76. A 30-year-old obese woman presented with headache and Benign intracranial
diplopia. On eye examination, there is papilledema. hypertension
Physical examination revealed that she was alert with no
focal neurological symptoms and signs
77. She has 4 days of fever, hepatomegaly, came back from Dengue fever
PLKN
GASTROINTESTINAL SYSTEM
78. Black colour stool, board like rigidity, on medication for perforated peptic
knee pain ulcer
79. 10 times diarrhoea, left iliac fossa pain ulcerative colitis
80. A 50-year-old obese businessman presented with cough Gastroesophageal
for past 3 months. He was given antihistamine, reflux disorder
bronchodilator, and corticosteroid but still the cough did not
improve. The cough was worst at early morning.
81. A 28-year-old man with history of diarrhoea alternating with Irritable bowel
constipation for four months. Abdominal examination syndrome
revealed no bleeding and no abnormalities.
82. A 60-year-old man presented with history of alternating Colon carcinoma
diarrhoea and constipation. He had loss of weight and loss
of appetite for the past months. He passed out stool with
mucus. He had history of diarrhoea when taking dairy
products in the past. On examination, he looked pale. His
abdominal examination and per rectal examination were

12
13 Oceanic Series End of Posting Compilation

normal. Sigmoidoscopy was done and revealed no


abnormalities.
JOINT PAIN
83. 58 y/o with knee pain. Worst when going downstairs than Osteoarthritis
climbing up
84. 60 y/o painful swollen left knee. Overweight, knee is warm Gouty arthritis
85. 39 years old woman having severe left knee pain. BMI is Septic arthritis
31. Temperature 38.5. knee swollen, tender, warm

URINARY SYSTEM
86. Afebrile, pain on micturition Renal calculi
87. Abdominal pain, no DM, no fever, ballotable kidney. Polycystic kidney
Positive proteinuria & blood in urine, subside one week disease
later.
88. A woman has got flu and fever. She did a urine Glomerulonephritis
examination because she used to have dark coloured
urine after a case of sore throat when she was a child.
Urinalysis revealed proteinuria, but it subsides a week
later.
89. Fever, rigor, loin pain Pyelonephritis
90. Right side abdominal pain, go to groin. Severe Ureteric colic
91. A 56-year-old man complained of hesitancy and difficulty Urethral stricture
in maintaining smooth flow over the past three months. In
the past, he has been treated for gonorrhoea and
underwent endoscopy for painful haematuria. Per rectal
examination was normal
HAEMOTOLOGY
92. A 16-year-old boy had mild upper respiratory tract infection Glucose-6-
and was given cephalosporin. 3 days later, he came back phosphate
to the doctor and presented with dark coloured urine. On dehydrogenase
physical examination, he looked yellowish and had deficiency anaemia
splenomegaly.

13
14 Oceanic Series End of Posting Compilation

93. A 60-year-old man was a chronic alcoholic. Has fissuring Megaloblastic


of corner of lips and sore of tongue. anaemia secondary
to vitamin B12
deficiency
THYROID
94. 40 years old lady with episodes of irritability for 1 week. Hyperthyroid
Complaint of poor sleep and easily anxious without
apparent reason. Has difficulty in comb hair.
PROSTATE
95. 60 years old have enlarged prostate newly diagnosed Give alpha blocker
hypertension
96. 67 years old noticed worsening of urinary symptoms: Prostate ca
difficulty in starting urination and maintaining smooth flow
and frequency. He experienced reddish urine several
times and back pain for past one month.

NERVE SYSTEM
97. An alcoholic man fell asleep on couch with arm hanging Radial nerve
the arm rest. He cannot dorsiflex and loss of sensation at
the lateral 3 digits and dorsal part of hand

98. Loss of sensation at lateral aspect of thigh. No muscle Lateral cutaneous


weakness. Pelvic compression positive. femoral nerve

99. A 72-year-old presented with chronic back pain associated Spinal


with numbness at thighs and legs. She feels pain more decompression with
when climbing downhill than climbing uphill. Her laminectomy
movement is very restricted. On MRI, there were
osteoarthritic changes and trefoil shaped lumbar spinal
canal.
RECOMMENDATIONS

14
15 Oceanic Series End of Posting Compilation

100. 70-year-old man wanted to do PSA test after his best friend Grade D.
died of prostate Ca. He consulted you if he should do the Recommended
PSA test. According to the UPSTF, what should you against
recommend him?
101. 50 years old men, asymptomatic, came for colorectal Grade A
cancer screening because friend had colon cancer. He
reluctant to get scope into his gut. Dr suggest doing annual
faecal occult blood test. What is the grade of
recommendation by UPSTF?

15
16 Oceanic Series End of Posting Compilation

MEQ

16
17 Oceanic Series End of Posting Compilation

MEQ 1
Edward, a 38-year-old salesman presented to his GP with discomfort passing urine for the
past few days, since returning from a conference in Vietnam. He had been married with his
wife for 5 years and was blessed with a son. (As discussed, time Role Play session)

1. List and explain how you want to manage this patient according to the steps of
management.
1) Do investigation: urinalysis, urine culture and sensitivity
2) Prescribe analgesics, antibiotics
3) Ask to bring her spouse for screening as well.

2. You suggested Edward to bring his wife so that she could be treated, otherwise she might
suffer from long term harm. Edward insisted he did not want to tell his wife about the extra
marital sex he had in Vietnam. List two medical ethics you need to apply in this situation.
1) Confidentiality
2) Patient autonomy

17
18 Oceanic Series End of Posting Compilation

MEQ 2

Mr Xavier aged 65 years was seen by one of the part time general practitioners (GPs) in the
Klinik ABC, Dr Zachary. He complained of a two-week history of productive cough with yellow
sputum. When he consults a GP, he was only given (could not remember which medication).
Mr Xavier was quite unsatisfied and meet another GP. At that time, he was given, another
medication (could not remember the name). After two to three days, Mr Xavier’s condition
becomes worse. He then went back to Klinik ABC and blaming the GP who treated him at the
first place.

1. List the steps of Significant Event Analysis based on this situation.


1) Two-week history of productive cough with yellow sputum.
2) Incomplete treatment

2. What do you think the factors that lead to this situation?


1) Patient low awareness on the importance having a continuation in treatment
2) Doctor lack of capability to educate the patient regarding the flows of
treatment

18
19 Oceanic Series End of Posting Compilation

MEQ 3

Madam Julia had successfully delivered her baby in Hospital XYZ. Upon examination, both of
her and her child was in good condition. The doctor then discharged her. After one week,
Madam Julia come back to meet the doctor with chief complaint of fever (39C) and lower
abdominal pain.

1. List 4 dangerous differential diagnosis


1) Puerperal sepsis – endometritis / Abscess in uterus
2) Urinary tract infection
3) Wound infection
4) Pelvic infection

2. What is your investigation to rule out your differential diagnosis?

1) FBC
2) Ultrasound of transvaginal (detailed smaller part) / transabdominal (wider view)
3) High vaginal swab for culture

19
20 Oceanic Series End of Posting Compilation

MEQ 4

Male patient aged 20-year-old came to the clinic with sore throat, dry cough, fever. Runny
nose whitish colour. Upon examination, pharynx a bit injected. Dr diagnosed him as having
URTI

Dr prescribed him with the drugs. Find 6 errors in the prescription


1) No date
2) Short form (URTI) needs to write in full term + viral pharyngitis
3) PCM should write in full word which is paracetamol
4) Benadryl should write as diphenhydramine
5) Antibiotic no need to give since it’s viral infection
6) Chlorpheniramine no need to give as diphenhydramine already given (both
are antihistamine)
He was transferred to a new workplace. Have persistent cough and SOB. He complained that
his current work area is exposed to fumes. Before this okay.

So, what is his diagnosis? Occupational asthma

What 2 drugs should give him now?

1) MDI salbutamol
2) MDI corticosteroid (budesonide, beclomethasone)

One advice should give to him: wear face mask while working (avoid the triggering factor)

20
21 Oceanic Series End of Posting Compilation

MEQ 5

56 y/o female, lawyer came for diabetic follow up, obese. BP 140/90 mmhg. FBS 4.5 mmol/L.
Medication she is currently on metformin tablet 500mg TDS. She had xanthelasma under her
both lower eyelids.

NORMAL:

• RBS: <11 mmol/l


• FBS: < 7 mmol/l
List all four aims of management of continuing of care of diabetic patient

1) To optimise blood sugar control – compliancy to medication


2) To prevent complication of the diabetes such as diabetic retinopathy
3) To find out if there is any cardiovascular risk factor
4) To treat acute complaint
5) To educate patient to having a healthier lifestyle

She complained of shaky hand, what is her condition? Hypoglycaemia

She is a busy person. Had to go meeting here & there. Eat a little during lunch. Sometimes
skip lunch. Give advice to her regarding:

1) Diet – don’t skip lunch.


2) Medication- doctor will try to reduce the dose of drug. Currently taking
metformin TDS (three times daily), need to reduce it to BD (twice daily).
List three investigation that you need to do on her now based on finding of her physical
examination her and state the reason

Investigation Reason
Hba1c To find out if her blood sugar level is well controlled
Liver profile Since she has xanthelasma, so we need to find out if she has
hyperlipidaemia
Renal profile test To check the kidney function

21
22 Oceanic Series End of Posting Compilation

MEQ 6

Female patient, 28 y/o, primigravida, came for her first booking. LMP 27th July 2018

a) Calculate her EDD


4th May 2019/ 3rd May 2019 (LMP+ 9m 7d)

b) Name of the method to calculate the EDD and 2 criteria to use it.
Method- Naegele’s rule
Criteria:
• Must confirm her LMP
• Regular cycle of 28 days

c) Other than full blood count, name 2 other investigation done during first booking and for
what disease
Investigation Disease
VDRL Syphilis
Hep B Hep B

Rhesus factor, ABO blood grouping, urine dipstick (check glucose and protein) also
can.

d) From the ix, her Hb is 10.1 mmol/. Low so Dr prescribe her drug. Name of the drug
Tablet ferrous fumarate 300mg

e) When is her next appointment? 24th weeks (for OGTT)

22
23 Oceanic Series End of Posting Compilation

MEQ 7

Mrs X, a 40-year-old woman just gave birth to her first child 6 weeks ago. The antenatal history
and delivery were uneventful. She had no past medical and surgical history. Her father had
hypertension. She is not smoking and non-alcoholic. She is currently breastfeeding the baby.

1. State 4 options of contraceptives that you can recommend to her.


1) Progesterone only pills
2) Intrauterine contraceptive device
3) Continue breastfeeding
4) Implanon

2. State 3 suitable histories that you want to elicit from this patient.
1) Past medical history (hypertension, venous thromboembolism, breast disease)
2) Allergy history (progesterone, latex, copper)
3) Menstrual history – any unexplained vaginal bleeding
4) Drug history (antibiotics –rifampicin; epilepsy drugs-carbamazepine, phenytoin,
phenobarbital; antiretrovirals)

3. State 3 other physical examinations that you would like to do and give a reason for each.

a) General examination To check signs of anaemia and jaundice


b) Vital signs To check blood pressure for hypertension
To look for any murmur due to valvular heart
c) Cardiovascular
disease which is prone to infection
examination
(contraindication in IUCD)
d) Abdominal examination To look for hepatomegaly, ascites, uterine mass

4. Mrs X would like an easy, free, painless, with no side effects yet effective method of
contraception. Give one such method and advise her on how to accomplish it.
Breastfeeding method (lactational amenorrhoea). Ask her to breastfeed the child
exclusively for 6 months.

23
24 Oceanic Series End of Posting Compilation

MEQ 8

Mdm Sundari, a 28-year-old housewife, presented to your health clinic with complaint of
contraction since yesterday. She was currently 32 weeks into her first pregnancy. She was
sure of her dates. She had no significant past medical, surgical, and family history. She is a
non-alcoholic and non-smoker.

1. State 2 possible diagnosis from her presenting complaint.


1) Braxton hick contraction
2) Pre-term labour
3) Placenta abruptio
4) Chorioamnionitis
5) UTI
6) DKA

2. State 3 significant symptoms that you would like to ask her to help you to arrive to a
provisional diagnosis.
1) Is it intermittent regular contraction which increase in frequency and duration?
2) Presence of show (mucus + blood)
3) Presence of leaking liquid
4) Any fever?

3. No significant history was revealed. You then moved to physical examination. State 4
physical examinations that you would like to do and give a reason for each.

To palpate fundal height/detect tenderness/


a) Abdominal examination
calculate foetal heart rate using pinard stethoscope
To check the OS opening/ liquor presence/ cervical
b) Vaginal examination
changes
To look for signs of anaemia, fever (temperature),
c) General examination
renal tenderness (renal punch)

4. Her physical examination was unremarkable. There is no indication to refer her to hospital.
State the most likely diagnosis.
Braxton Hicks contractions

24
25 Oceanic Series End of Posting Compilation

MEQ 9

Mrs Zara brought her 6-month-old child to health clinic for follow up and vaccination. She
missed her child last scheduled check-up but completed others prior to that.

1. State the age of her child when she missed the scheduled check-up. 5 months old

2. List the infections that can be prevented with the vaccine which her child missed.
1) DTAP
a. Diphtheria
b. Tetanus
c. Acellular Pertussis
2) Polio (inactivated polio vaccine)
3) Haemophilus influenza type B
4) Hepatitis B

3. The child was found to be healthy with no contraindications for vaccination. So, the doctor
decided to give the child the missed vaccination in this session.

a) The preparation of the vaccine to be given:


DTaP-IPV-Hib- Hep B vaccine (combination 6-in-1)
b) Route and site of administration: Intramuscular at anterolateral side of mid-thigh

4. The child was also scheduled for another vaccination at her current age (6-month-old).

a) Name the vaccine that a 6-month-old should receive: Pneumococcal vaccine strain
10 and measles if in Sabah
b) State which dosage of the vaccine should be received. 2nd dosage
c) State the amount of dosage of the vaccine should be received by that child throughout
his life according to the schedule. 3 dosages (4th month, 6 months, 15 months)

25
26 Oceanic Series End of Posting Compilation

5. The doctor wanted to give the vaccine above to the child.

a) Describe how the doctor can give the vaccine above to the child with the missed
vaccination.
Give both at the same time at different locations or give it a month later to avoid
any reactions.

b) Route and site of administration. Intramuscular at anterolateral side of mid-thigh

26
27 Oceanic Series End of Posting Compilation

MEQ 10

Mrs Christy was a 60-year-old retired teacher. She was referred to your Diabetic Clinic after
found to have diabetes mellitus in her previous check-up.

1. List 4 aspects in history that you would like to ask to look for any predisposing factors.
1) Lifestyle (sedentary: diet, exercise)
2) Drug history (hyperglycaemic drugs: steroid, hydrochlorothiazide)
3) Family history of diabetes mellitus
4) Weight history (obesity, overweight)
5) Obstetric history (gestational diabetes mellitus, macrosomia)
6) History of impaired fasting glucose/impaired glucose tolerance

2. Describe 3 systems that should you assess in physical examination for this patient.
a) General To calculate body mass index, waist circumference, and
examination check feet for non-healing wounds.
b) Eye examination
To look for diabetic retinopathy and cataract
(fundoscopy)
To check blood pressure for hypertension, look for signs
c) Cardiovascular
of peripheral vascular diseases (diminished peripheral
examination
pulses) and cardiomegaly
d) Nervous system
To assess peripheral neuropathy (loss of sensations)
examination

3. She had a weight of 70 kg with height of 160 cm. Her blood investigations were done. Her
HbA1c level is 8% and her fasting blood sugar is 8 mmol/L. List 3 other investigations that you
would like to do for this patient with reason.

a) Renal function test Renal impairment


b) Fasting lipid profile Hyperlipidaemia (CVS risk factor)
Hypo albuminuria, urinary tract infections and
c) Urinalysis
diabetic nephropathy
Coronary artery diseases, ischemic changes and
d) Electrocardiogram
cardiomegaly

27
28 Oceanic Series End of Posting Compilation

4. State the drug you would give to this patient to control her diabetes mellitus and the target
level for her to achieve in fasting blood sugar.
Metformin. The target level is 4.4 to 6.1 mmol/L.

28
29 Oceanic Series End of Posting Compilation

MEQ 11

For 6 months, 76-year-old male, having slow movement and walking in smaller steps. He is
on hydrochlorothiazide for mild hypertension, simvastatin for hypercholesterolemia and
sedative to help for sleep. His BP is 150/70 mmHg, has slow tone speech but clear but noted
tremors during resting. He has rigidity on his lower limbs and slow upon standing.

(a) Based on history and physical examination, what other chronic medical condition he
suffered from?
Parkinson’s disease / multi-infarct dementia / multi-infarct syndrome

(b) List 3 risk factors and preventive measurement.

Possible side-effects of Ensure minimum effective dose


hydrochlorothiazide Change hypertensives drugs
Ensure adequate taking of potassium (supplement/take
vegetables high in K+
Monitor serum electrolytes regularly
Check for postural hypotension during examination
Possible side effects of Counsel for other way for sleep problems
sedatives Change to short-acting sedatives
Instability and mobility Physiotherapy and physical exercise-improve muscular
problems strength, balance, and flexibility /
Assess functional status including risk for fall and counsel
accordingly
Take more care during walks
Consider starting on anti-Parkinson drugs

29
30 Oceanic Series End of Posting Compilation

MEQ 12

30 y/o teacher just delivered her baby 8 months ago. She wants contraception. She has no
medical illness and not smoking. Preferred contraception with no medication and not a device.

1) State 2 contraception and disadvantages.


1. COCP – increased risk of cardiovascular disease and worsen migraine
2. Copper IUD – dysmenorrhoea and menorrhagia

2) Comment on effectiveness of both contraception method


Copper IUD and COCP up to 99% effective.
After discussing with her husband, she decided on IUCD.

3) 2 contraindications of IUCD
1. Can cause allergic reaction for those who allergic to copper
2. Cannot be used for those having current STD/PID

4) Most common material used for IUCD in health care & time effective.
1. Mirena – 5 years
2. Copper – 5 and 10 years

5) Precaution manoeuvre after insertion of IUCD


Warn woman regarding signs of infection and menstrual cramps, heavy menstruation
for few months. If a woman is high risk, give prophylactic antibiotics

30
31 Oceanic Series End of Posting Compilation

MEQ 13

55 y/o gentleman, 1st time coming to the clinic for hypertension follow up. No history of
dyslipidaemia and diabetes mellitus.

1) 4 details history.
1. Drug history
2. Family history – any DM/HTN/CVD
3. Social history – occupation?
4. Lifestyle history – diet, exercising

2) Comment on blood pressure 150/92, waist circumference 95cm, and BMI 30.
BP, BMI high

3) 3 management measures
1. Increase dose of hypertensive drug
2. Lifestyle modifications
3. Refer dietician

31
32 Oceanic Series End of Posting Compilation

MEQ 14

Mr Ahmad, 76 years old, brought by his daughter to clinic. He complained of dizziness for a
while on getting up from bed. Take Hydrochlorothiazide, simvastatin, aspirin and sleeping pills
for insomnia. Have stroke 3 months ago

i. List 3 potential causes of fall and prevention measures

Associated with postural Be careful and take more care from getting up of the
hypotension bed/chair
Hypokalaemia, Ensure adequate taking of potassium
hyponatremia (supplement/take vegetables high in K+)/ change to
(hydrochlorothiazide) another anti-hypertensive drug
Drowsiness- side effect Counsel for other way to sleep
from sleeping tablets
Side effect of aspirin- Change to another anti-platelet drugs
anemia (GI bleeding)
Stroke cause imbalance Physiotherapy-improve muscular strength /Use aid
(weak limb) when standing or walking/ slower pace at home

ii. Briefly describe 2 measures for his daughter for her own self-care (to cope with stress)

1) Learnt about her father illness – so that she will be more confident to do
things
2) Maintain her psychological wellbeing by talk to counsellor
3) Have regular break for herself
4) Get support from friends, relatives, and any NGO
5) Looking for herself well physically: exercise regularly and take nutrition
meals
6) Maintain hobby for relaxation

32
33 Oceanic Series End of Posting Compilation

MEQ 15

Mrs Ain had her 1st baby (6weeks) uneventful pregnancy, not breastfeeding and wanted
contraception. She is non-smoker

• 4 suitable methods of contraception

1) COCP
2) Barrier
3) IUCD
4) Implanon

• Keen to take oral contraceptives, last period 1 week ago. How to administer?

1) Everyday pills (Micogynon ED) 21 pills + 7 pills free/placebo


2) Take about the same time everyday

Miss OCP for 3days but doesn't have SI for 1 week. Preventive measures she should take

1) If in week three, finish active pills and start a new pack.


2) Throw the inactive pills
3) Use barrier method/ avoid SI

33
34 Oceanic Series End of Posting Compilation

MEQ 16

60 years old pensioner newly diagnosed as diabetes mellitus in public health clinic

i. 4 areas doctor should assess

1) Specific symptoms of diabetic – weight loss, polyuria, polydipsia.


2) Pre-disposition to diabetic – overweight, physical activity, diet
3) Risk factor of cardiovascular disease – smoking, HTN, hyperlipidemia
4) General symptoms looking for complications – sexual dysfunction,
cataract, numbness

ii. 4 systems doctors assess

1) General examinations – BMI, BP


2) CVS
3) Eye – blurry of vision, cataract
4) Neurovascular system (Feet): sensation, circulation

iii. 4 investigations should order

1) Fasting lipid profile


2) Fasting blood sugar
3) HBA1C
4) Renal profile

BP: 130/80mmHg, Height: 145cm, Weight: 75kg, BMI: 31.5

iv. Non-pharmacological advice to him

1) Diet – low salt, high fibre


2) Exercise – 20-30 minutes for 3-4 days in a week
3) Reduce weight

v. Diabetic drug
Tablet Metformin 500 mg BD

34
35 Oceanic Series End of Posting Compilation

MEQ 17

Immunization

i. Vaccine at 12 months old baby?


Measles Mumps Rubella (MMR) 2nd dose

ii. Details history from mother before giving immunization and why?
• Previous allergy reaction to any vaccine to avoid anaphylaxis
• Completed previous immunization- ensure no missed vaccine
• Any current illness-high fever so that postpone vaccination
• Adverse effect after any vaccine- example febrile fit
• Immunocompromised state (undergone any chemotherapy, radiotherapy,
steroids, active hiv) because it is a contraindication for live vaccine
• Drug hx – on steroid treatment / immunoglobulin
• Take any live vaccine pass 1 month? If yes do not give another live vaccine

iii. Preferred site of immunization


Anterior lateral aspect of mid thigh

iv. Response to mother about autism and vaccine

1) Provide reliable sources for her to refer.


2) Discuss the benefits of immunization and disadvantage not taking
immunization
3) If still concern, then respect her opinion
4) Suggest joining group support for autism’s parents.
5) Follow up to discuss later.

35
36 Oceanic Series End of Posting Compilation

MEQ 18

Puan Zara, 6 months old child to KK, bring according to schedule.

1. At first month of life, what condition of infection should be vaccinated.


1) BCG
2) Hepatitis B

2. Doctor decided to give vaccines: Pneumococcal vaccine 2nd dose and Measles 1st
dose if in Sabah

3. Route of administration: IM at anterolateral side of mid-thigh.

36
37 Oceanic Series End of Posting Compilation

MEQ 19
3 years old, 4 days fever, 37.4-degree Celsius, no rash.

• Other history & reason?


1) History of cough? (Rule out URTI)
2) History of ear discharge? (Rule out any ear infections such as Otitis Media)
3) History of crying during urination? (Rule out UTI)
4) History of epistaxis or gum bleeding (warning signs for dengue fever)

• Hess test positive. What is your diagnosis?


Dengue fever

• Other blood investigations


1) Full blood count (↓ platelet, ↑ haematocrit, ↓ lymphocyte count)
2) ↑ ESR
3) Coagulation profile (↓Prothrombin Time, ↓activated partial thromboplastin
time)

• Home care measure

1) Remove mosquito habitat (clear up old tyres, clean clogged drains, put
larvicides in unused water)
2) Fogging
3) Use mosquito netting
4) Wear protective clothing (long sleeve shirt & long trousers)
• How to take care of him?

1. Bed rest
2. Give Paracetamol (analgesic & antipyretic)
3. Give adequate water & fluid
4. If patient develop gum bleeding and/or epistaxis, come back to see
doctor

37
38 Oceanic Series End of Posting Compilation

MEQ 20

Mr Ahmad complaint of productive cough with white copious sputum, work in factory

1. Other history & reason.


1. Does he work in dusty environment?
2. What type of work he deals with? (Rule out occupational cause)
3. Does he have recent contact with TB worker?
4. How long does he have been coughing? (To determine severity)

2. Management
1. Ask him to change department that does not involve him with dusty
environment in the factory
2. Tell him that this is due to occupational causes (Pneumoconiosis).
3. Tell him not to worry as it can be cured if he avoids the trigger factors

3. Mr Syamsul call regarding Mr Ahmad. How do you handle Mr Syamsul?


Tell Mr Syamsul that patient’s health information is considered confidential. Mr Ahmad
needs to be told first to obtain the permission

38
39 Oceanic Series End of Posting Compilation

MEQ 21

Hypertension first appointment in your clinic. BP 150/100mmHg, height 170cm, weight 90kg,
smokes 5 sticks per day.

1. Aspect of history
a. Ask for other underlying comorbidities (Diabetes Mellitus, Hyperlipidaemia)
b. Family history of comorbidities
c. Assess target organ damage symptoms (Brain, Eye, Heart, Renal)
d. Assess prevention control (lifestyle, diet, BP machine at home)

2. Other investigations & reason


a. Fasting blood glucose for DM
b. Fasting lipid profile for hyperlipidaemia
c. Renal profile to assess renal damage
d. ECG to rule out heart failure as a complication

3. How to manage this patient


a. Tell him he is on hypertension stage I
b. Tell him not to worry but exercise caution and prevention to further aggravate
the condition
c. Lose some weight to achieve ideal BMI count and waist circumference
d. Exercise minimum 150mins per week (20-30 minutes for 3-4 times per week)
e. Diet (practice good food portion, low fat, oil, sugar and salt intake
f. Stop smoking
g. Prescription = Tablet Perindopril 4mg OD 3/12 (Any ACEI would do), tell
about dry cough as a side effect of drug

39
40 Oceanic Series End of Posting Compilation

MEQ 22 ASTHMA

a. What is the reading to be expected in this patient?


i. Highest PEFR reading

b. Calculate the percentage

i. Highest PEFR reading x 100 = ___%


Expected PEFR reading

ii. How to differentiate between the types of asthma?

Asthma types Controlled Partially controlled

Daily Symptoms <2 / week ≥2 / week


MDI usage <2 / week ≥2 / week
Nocturnal None Yes
Symptoms
Limited activities None Yes
Lung function ≥80 % <80 %
(PEFR)

*GINA guidelines
** Uncontrolled asthma = ≥3 of partially controlled
c. What are the causes (triggering factors) for her asthma?
i. Pollution (Smoke, dust)
ii. Pollen
iii. Pets
iv. Cold weather

d. How to monitor? PEFR using peak flow meter

40
41 Oceanic Series End of Posting Compilation

e. What is the spirometry curve expected in this patient?

MEQ 23
Madam Fauziah presented with Diabetes Mellitus and need to continue medications.

1. Please list 10 errors in prescription given


1) Write illness in short form; ‘DM’ instead of Diabetes Mellitus
2) IM injection Chlorpheniramine, should give tablets
3) T. Glibenclamide 2.5mg BD
4) T. Mefenamic acid 2/52 (overdose), instead of 1/52
5) T. PCM QID 2/52 (overdose), instead of 1/52
6) T. Bacampicillin is not necessary in viral fever
7) MMC registry
8) Qualification
9) Rubber stamp

2. Mr Wong came from the Great Eastern Insurance called to inquire about Madam
Fauziah’s illness. What are the 2-medico legal or ethical issues involved?

1) Confidentiality (Never give information via phone)


2) Informed consent (get written consent form)

3. How would you like to approach this?


Take consent from Madam Fauziah first. Tell her that the information to be given
is confidential

41
42 Oceanic Series End of Posting Compilation

MEQ 24

Patient came with the complaint of throbbing headache, progressively worsening, exacerbated by
movements, has not enough sleep recently and feeling comfortable at dark room.

1) Provisional diagnosis
Migraine

2) Differential diagnosis
1) Cluster headache
2) Tension headache

3) Red Flags not to be missed

1) Space occupying lesion


2) Subdural haematoma
3) Subarachnoid haemorrhage

4) Medications to relieve headache

1) T. Sumatriptan 100mg
2) T. Aspirin x1/52

5) How would you advise the patient?

1) Identify & avoid triggering factors


2) Practice relaxation technique
3) Stress management
4) Practice healthy lifestyle

42
43 Oceanic Series End of Posting Compilation

MEQ 25 HYPERLIPIDAEMIA CASE

1) List 2 other relevant history to assess the cardiovascular risk

1) Current history of chest pain


2) History of Diabetes Mellitus and/or stroke
3) Family history of ischaemic heart disease and/or hyperlipidaemia
4) Diet history, smoking, exercise

2) List 2 physical findings to elicit

1) Presence of xanthelasma (indicating familial hyperlipidaemia)


2) High blood pressure reading
3) High reading of waist circumference
4) Presence of acanthosis nigricans (indicates Diabetes Mellitus / metabolic syndrome)

3) List 6 points in management of this patient

1) Reassurance – Assure the patient whether the cholesterol reading is high or normal
2) Advice – Advice on low saturated fat and cholesterol diet, practice healthy lifestyle
like exercise for 150 mins/week
3) Prescription – T. Lovastatin 5mg ON x3/12
4) Investigation – ECG
5) Observation – Follow up after 3 months to assess cardiovascular risk & to recheck
cholesterol level
6) Prevention – Pap Smear, Mammogram (if elderly woman)

43
44 Oceanic Series End of Posting Compilation

MEQ 26

52 years old businessman, previously on follow up at Johor Bharu for hypertension for 10 years, came
to your GP for hypertensive medication. He was previously on 100mg of hydrochlorothiazide and B
blocker (atenolol)

1) List 4 history that need to be assessed in this patient


1) History of hypertension
2) Medication history (how long he was on, adverse effects, compliance)
3) Signs & symptoms of end organ damage (chest pain, TIA, sudden loss of vision, urinary
problem, intermittent claudication)
4) Other comorbidities (DM, hyperlipidaemia, asthma)
5) Family history of comorbidities
6) Social history (smoking, alcohol consumption, workplace, diet history, exercise)

On examination, blood pressure was 150/100 mmHg, obese, pulse rate is low, presence of nicotine
stain on the finger. He is having muscle weakness and shortness of breath on brisk walking but no
symptoms of chest pain

2) What relevant investigations that can be done


1) Fasting lipid profile (since patient is obese)
2) Fasting blood sugar (since patient is obese)
3) ECG (to look for evidence of MI)
4) BUSE (to look for signs of electrolyte imbalance since he is taking hydrochlorothiazide
– hypokalaemia may be the reason for low pulse rate & muscle weakness) plus
hyponatremia

3) How to manage this patient


a. Reassure & explain
i. Explain his blood pressure is uncontrolled even after taking medication
ii. Explain that his low pulse rate and symptoms of muscle weakness can be
due to electrolyte imbalance from taking hydrochlorothiazide
iii. Explain that he is having shortness of breath might be due to B blocker that
he is taking as it can cause bronchoconstriction thus causing shortness of
breath
b. Advice

44
45 Oceanic Series End of Posting Compilation

i. Advice on his dietary management since he is obese


ii. Advice on smoking cessation

c. Prescribe
i. Patient is already having bradycardia and muscle weakness due to
hydrochlorothiazide and atenolol
ii. Consider other types of antihypertensive medication
iii. Can replace with ACE Inhibitors since patient’s age is less than 55 years old
d. Referral
i. Refer to dietician
ii. Refer to quit smoking clinic

e. Investigation
i. Fasting lipid profile
ii. Fasting blood sugar
iii. Renal profile
iv. ECG
v. BUSE
nb
f. Observation
i. Follow up this patient 2 or 3 months once to monitor the blood pressure and
smoking status

g. Prevention
i. CVD risk factor screening
ii. Exercise
iii. Workplace = Wear PPE

45
46 Oceanic Series End of Posting Compilation

MEQ 27

A 35-year-old woman presented with right leg swelling for the past 3 days. She had a C-section done
5 years ago for her 2nd child

1) What are the further questions that you would like to ask this patient regarding her swelling?
1) Is there any pain?
2) Onset?
3) Progression?
4) Has the swelling increased in size?
5) Any history of insect bite?
6) Any history of walking without shoes or slippers on bare soil
7) History of trauma to the leg
8) Any medical illness?

2) State 2 differential diagnosis

1) Deep vein thrombosis


2) Right leg cellulitis
3) Congestive heart failure
4) Swelling due to local trauma

3) State 2 red flags not to be missed

1) DVT
2) Cellulitis

46
47 Oceanic Series End of Posting Compilation

MEQ 28

Mr Lee 63 years old treated with hypertension for 15 years ago. He was on Hydrochlorothiazide 25
mg OD. Blood pressure is not well control. His GP added Amlodipine 5mg OD but complaining bilateral
lower limb swelling for 3 months. He just returned from Japan but cannot tolerate to cold weather.

1) List 4 aspects in history that should be asked to determine the cause of the ankle swelling
1) Acquire symptoms for any symptoms of heart failure or heart disease (SOB,
orthopnoea, PND)
2) Symptoms (stigmata) of liver disease
3) Renal problems (including past medical history of it)
4) Any alcohol consumption
5) Drugs history such as Amlodipine (for side effect) – headache, flushing, feeling tired,
swollen ankles

2) Management

R → explain BP is high, not well controlled and he’s obese. Cause of swelling is due to drugs.
Reassure the swelling will resolve after stopping the drug
A → reduce weight. Regular exercise
P → Stop amlodipine. Start on ACEI (perindopril). Continue HCTZ.
R → Quit smoking clinic
I →Fasting lipid profile, FBG, ECG, TFT
O→ Follow up to reassess the BP, the ankle swelling, to review blood ix
P → colon ca screening

• β-blockers S/E: bradycardia, lethargy, sexual dysfunction


• hydrochlorothiazide S/E: hypokalaemia, hyponatremia, ↑ serum uric acid, ↑risk for
hyperglycaemia

47
48 Oceanic Series End of Posting Compilation

MEQ 29

50 y/o men had HTN for 5 years came to KK with a chief complaint of leg swelling for 1 month. His BP
is 140/90 and PR 60 bpm. Today, his BP is 150/90 mmHg has pedal edema, BMI 30. He is on T.
amlodipine 10mg OD and T. simvastatin 20mg ON.

1. List 3 causes persistent high blood pressure


1) Resistant HTN
2) White coat HTN
3) Not compliance to medication
4) Lifestyle – high salt intake
5) Not exercise
6) Smoking
7) Renal impairment

2. State 3 investigations for his pedal edema and give reasons each
1) Renal profile TRO renal impairment
2) Urinalysis – proteinuria (Nephrotic syndrome)
3) ECG – ischemic changes (ACS)
4) LFT – hypoalbuminemia
5) CXR – cardiomegaly
6) Doppler ultrasound- to exclude DVT

3. List 3 non-pharmacological management


1) Exercise at least 30 min 5 times
2) Decrease salt intake
3) Avoid canned food
4) Decrease weight (ideal BMI)
5) Decrease alcohol intake

4. Give one antihypertensive drug for this patient and give reason
1) Diuretic – help in pedal edema (side effect of amlodipine)
2) ACEI – better in person age <55 years old

48
49 Oceanic Series End of Posting Compilation

MEQ 30

40 y/o mother delivered her third baby normally. No past medical and surgical history. She is not
breastfeeding her baby and she is a chronic smoker.

1. List 3 options contraceptive for the mother


1) POP
2) Implanon
3) IUCD
4) Tubal ligation (because has 3 children already,40y/o)

2. Give 3 areas of history you would like to ask to the mother


1) PMH – history of unexplained vaginal bleeding, current STI/PID, heart disease
2) Allergy history – allergy to copper?
3) Social history – financial status
4) Husband preference – completed family?
5) History of taking any contraception before.

3. List 3 physical finding you aspect to see in this patient and give reasons
1) BMI increase - Obese
2) Breast lump – oestrogen can increase the size of breast lump
3) Calf tenderness – DVT (hypercoagulability common after delivery)
4) Decrease air entry – chronic smoker may cause obstructive lung disease.

4. This contraception is reversible, easy, painless, and effective for a few years.
1) Implanon
2) IUCD

49
50 Oceanic Series End of Posting Compilation

MEQ 31

Mr A brought his 6-month-old child to KK to assess his development milestone. He come from a large
family. He is Chinese while his wife is Malay and maid Indonesian. They are spoken in multiple
languages at home.

1. List 4 major areas of development milestone and state DM in 1 year old child

Development milestone 1 year old child

Gross motor • Stand and walk alone


Fine motor • Neat pincer grasp
• Bangs 2 cubes
Language • Know and responds to name
• Understand simple instructions
Self-care / social behaviour & Play • Gives toys on request
• Drink from cups and dressing with
assistance

The child can say no word other than papa and mama at 18 months old. He is cooing and babbling at
appropriate age. He has no medical illness.

2. Define cooing and babbling


Cooing = vowel sound “aaaa” “ooo”
Babbling = consonant and vowel sound “dah”

3. What is the most likely reason for his lack of words at 16 month old.
Different multiple languages spoken in the house the baby need a longer time to interpret it

50
51 Oceanic Series End of Posting Compilation

MEQ 32

46-year-old woman had chest discomfort and palpitation. She is worried that she might have heart
problem. She is a full-time teacher, struggling to take care of her children as her maid is left and her
husband is going for outstation.

1. State 3 aspects in history that should be asked to assess cardiovascular risk


1) Medical illness she is having now
2) Family history of heart problem
3) Social history – smoking, diet

Her height is 150 cm and weight are 65 kg, BP 130/80 mmHg. Thyroid function test, FBS and lipid
profile was normal.

2. List 2 health issues


1) Obese/overweight
2) Stress

3. List 3 nonpharmacological management


1) Balanced diet (less fat, high fibre)
2) Regular exercise to reduce weight
3) Stress management
Her last pap smear test was 5 years ago, and the result was normal.

PAP SMEAR -> In Malaysia, women who are sexually active aged 20 to 65 years old
recommended to do pap smear. If first 2 consecutives test are negative, screening every 3
years are recommended.

4. State when she needs to do further pap smear


Immediately (because already 5 years)

5. Advise the patient about tumour marker on cancer screening


It detects cell changes and not confirm cancer, if the pap smear is positive, we will
undergo colposcopy to detect pre-cancer state.

51
52 Oceanic Series End of Posting Compilation

MEQ 33 CHEST PAIN – MI

1. 2 other life-threatening disease


1) Pulmonary embolism
2) Acute dissecting aneurysm
3) Tension pneumothorax
4) Acute coronary syndrome

2. Initial investigation – electrocardiogram


Expected finding – ST segment elevation

3. Reassure – he will be admitted, offered to cardiologist


Advice – need to be admitted to hospital right away

4. 4 immediate management
a) Morphine
b) Oxygen
c) IV Nitrite (GTN)
d) Aspirin 300 mg

52
53 Oceanic Series End of Posting Compilation

MEQ 34

Mr Nathan, 39 years old, presented to general practitioner because of his low back pain for two weeks.
He was a long-distance lorry driver. He often lifted heavy goods from the lorry. He watched television
mostly on his free time. He was a cigarette smoker. No previous illness.

1. Classification of his low back pain based on the duration


Acute low back pain

2. Based on the history, Mr Nathan’s back pain has no radiation. State the location of the low back
pain.
Lumbosacral area

3. Three risk factors for the low back pain based on the history.
1) Sedentary lifestyle
2) Cigarette smoking
3) Lack of exercise

On physical examination, he has normal BP. No jaundice, no anaemia.

4. Three other physical signs that you want to find.


1) Deformity of spine
2) Straight leg test – PID (positive pain at 30-70 degree)
3) Lasegue’s test
4) Bowstring test
5) Limping movement

5. 3 management beside reassurance and explanation.


1. Advice on having enough rest
2. Exercise regularly
3. Reduce weight
4. Smoking cessation
5. Do CT scan of spine

53
54 Oceanic Series End of Posting Compilation

MEQ 35

Mr Lim was brought to Dr Wong’s clinic by his daughter because of reduced oral intake for the past 2
weeks. Mr Lim was wheelchair bounded and had difficulty in speech for 2 years ago after he suffered
from a stroke. However, his cognitive function was normal and could communicate through writing
(Broca aphasia). On examination, his pulse was 90 bpm and blood pressure 110/160. He looked pale.
His abdomen was soft and no mass palpable. There was no melena. His HB was 6g/dl.

Dr Wong decided to admit Mr Lim for further investigation and blood transfusion. However, the
daughter refused admission as she claimed that her father disliked hospital admission.

a. State the ethical issue and the ethical principle involved in the scenario above? (2 marks)
1) Ethical issue: Mr Lim’s daughter refused for hospital admission on behalf of his father
2) Ethical principal: Patient’s autonomy

b. State 3 information that should be given to patient and his daughter to assist them in making
informed consent regarding the admission for blood transfusion. (3 marks)
1) Current condition and complication: Mr Lim has severe anaemia which can give rise to
potential complication like heart failure
2) Procedure: Explain about blood transfusion procedure
3) Benefits: compensate the blood circulation

After the counselling, Mr Lim and his daughter agreed for admission. His daughter requested for a MC
for herself as she could not return to her workplace to resume her work.

c. Would Mr Lim’s daughter eligible for a MC? State your reason. (2 marks)
No, because she is not sick.

d. List 3 essential features of a good medical record. (3 marks)


1) Focus history (positive relevant and negative relevant)
2) Focus physical examination (positive relevant and negative relevant)
3) Differential diagnoses

54
55 Oceanic Series End of Posting Compilation

MEQ 37

Pain in urination, yellow & cloudy urine. Come back from Bangkok. Early morning stiffness.

1. Relevant history?
• Sexual history – unprotected sexual history, promiscuity.
• Any medical illness?

Result: Leucocyte 3+ Nitrite 1+ Blood 3+

2. Gram negative diplococci organism?


Neisseria gonorrhoeae (gonococcal urethritis)

3. Management plan. (RAPRIOP)


a. Reassure – explain about the disease, related to sexually transmitted disease
b. Advice – safe sex
c. Prescription – ceftriaxone (3rd generation cephalosporin)
d. Referral – notify the patient to inform his partner →communicable disease
e. Investigation – HIV, Hep B, Hep C, syphilis (VDRL)
f. Observation – follow up for result. Do screening on his partner.
g. Prevention – condom. Etc

55
56 Oceanic Series End of Posting Compilation

MEQ 38: DIABETES MELLITUS

56 years old male. Working with machine. Currently on metformin 250mg TDS.
Symptomatic – polyuria. Missed tablet. A smoker. Not exercise.

BP: 130/90
BMI: 28
FBS: 8.6
HbA1c: 9.12

1. DM controlled?
• Uncontrolled DM

2. How to reassure?
• Comfort him; tell him that his DM can still be controlled with lifestyle modification
& drug compliance.

3. Advice?
• Need to be compliant to drug. Stop smoking. Exercise regularly. Reduce weight.

4. Prescription?
• Reduce frequency + increase dose.
• Tablet Metformin 500mg BD.

5. Refer?
• Stop smoking clinic.
• Dietician.

6. Investigation? (Annual basis)


1) FLP, FBS, RP, ECG, HbA1c, LFT.
2) Fundoscopy/ Fundus camera.
3) Diabetic foot examination (sensation, vibration)

56
57 Oceanic Series End of Posting Compilation

7. Prevention?
1) Faecal occult blood test (FOBT) – for colon cancer
2) CVS risk factors (weight management, smoking, sedentary lifestyle)
3) Prevention of injury (wear boots-PPE, wear ear plug)

57
58 Oceanic Series End of Posting Compilation

MEQ 39 MC & MEDICAL ASSURANCE

A) 36 years old, right ankle sprain at work (industrial accident). He didn’t go to work for the past 3
weeks. Ask for mc for 3 weeks.

1. Ask for 2nd MC to cover 3 weeks previously to claim insurance.


1) Cannot give MC. Patient need to come back previously if want MC.
2) Can give/do report, for only 3 days.
3) Cannot give extra MC
4) According to examination, it was a sprain, range of movement is normal, and he
was already given an MC. Ask him to come back for review

2. How long record should be kept in industrial accident?


Indefinite / keep it as long as you can up to 10 years

B) Chickenpox, febrile. Get 3 days MC. Want to claim insurance.

1. Still have healing lesion.


1) Chickenpox → take at least 2 weeks to heal.
2) Dr. should give follow up.

2. Essential good medical record?


1) Relevant detail of history.
2) Examination finding.
3) Differential diagnosis.
4) Treatment and investigation.
5) Follow up & management.
6) Record RAPRIOP.

58
59 Oceanic Series End of Posting Compilation

MEQ 40

Mrs Chong happily married. Complained of dysuria. No changes of urine (appearance/smell). On


examination: suprapubic tenderness. No hx of trauma. Not wearing any contraceptive device

• UTI
• Calculi
• Cystitis
• Urethritis
• PID

a. What further hx to elicit? Give reason


1) Any hx of vaginal irritation/discharge? – TRO vaginal infection
2) Any hx of fever – to differentiate upper/lower UTI
3) Sexual history – to exclude PID (multiple partner/ homosexual/ heterosexual)
4) Pain throughout the micturition or at early/mid/late? – bladder calculi at late of
micturition

b. On examination, nothing remarkable except suprapubic tenderness.

b. Most likely diagnosis?


• Acute cystitis

c. What diagnostic test in (GP setting) would you order to confirm your diagnosis
• Urine dipstick or UFEME
• Urine culture & sensitivity

d. What’s the most likely the aetiological agent?


• Escherichia coli

e. What drug to prescribe?


• Trimethoprim

59
60 Oceanic Series End of Posting Compilation

MEQ 41

42 years old man presented with history of cough and high fever for 2 days. He denies of having sore
throat. No significant history and social history other than he are a smoker, he used to smoke 20
cigarettes per day since he was 15 years old.

a) 4 details to be ask for cough and fever.


1) Cough: Is it productive, frequency, severity, characteristics, any chest pain,
colour/amount/blood in the sputum
2) Fever: Pattern, does it associate with chills and rigor

b) 2 other symptoms need to be asked if suspect lower respiratory system involvement


1) Chest pain associated with cough or deep breathing
2) Shortness of breath
3) Rapid breathing
4) Wheezing

On physical examination, his temperature is 39 degrees Celsius, pulse rate 100 per minute, blood
pressure 130/70 mmHg, respiratory rate 30 breath per minute, on chest expansion it is symmetrical
on both sides but shallow (indicate that patient is having pleuritic chest pain as deep breath can make
the chest pain worse), the trachea is at the centre.

c) 4 other signs need to be elicited on auscultation


1) Air entry equal or not on both sides of the lungs (COMPARE)
2) Breath sound, is it vesicular or bronchial?
3) Presence of rhonchi, crepitation
4) Vocal fremitus

Chest x-ray shows localized left mid zone haziness

d) What is the most likely diagnosis?

Left middle zone lobar pneumonia/ community acquired pneumonia

60
61 Oceanic Series End of Posting Compilation

e) How to manage this patient


1) Explanation: Explain finding from history, physical examination and investigation that
indicate he is having a lung infection
2) Reassurance: Inform him that this disease is a treatable condition
3) Advise: Wear face mask, avoid going to crowded area
4) Prescription: Tablet Paracetamol for fever, course of antibiotics such as Macrolide
5) Investigation: Sputum and blood for culture and sensitivity
6) Observation: Follow up after 2 days to review his condition (not follow up after 1 week
or more than that) or can admit him to hospital (as he looks quite ill)
7) Prevention: Stop smoking to reduce risk of getting lung disease and ischemic heart
disease

61
62 Oceanic Series End of Posting Compilation

MEQ 42

20 years old, male, a known case of bronchial asthma for 5 years. He was on: MDI Beclomethasone
200mg BD, MDI Salbutamol 200mg prn. For the last few days/weeks, he had used his MDI Salbutamol
for 3 times. He was also unable to join his favourite mountain biking activities. PEFR? 70%. Upon
presentation, he was well. No significant finding in physical examination.

1) State the level of control: Uncontrolled

2) 3 Possible reasons:
1) Drugs: Poor compliance, wrong inhaler technique
2) Triggers: Exposure to allergens
3) Smoking
4) Infection: Any associated symptoms such as fever and haemoptysis. To rule out TB
and Pneumonia

3) No remarkable findings in the history and physical exam. What is the most possible reason of his
condition? Ans: Drugs were not enough --> need to step up

4) Plan of management (RAPRIOP)


1) Explanation: Explain to him that his asthma is uncontrolled, which may be due to poor
compliance/drugs were not enough.
2) Advice: Continue inhalers with right technique as prescribed but maybe we need to
step-up. Avoid trigger factors. Teach him on how to recognise the worsening of his
asthma. (Asthma Action Plan- when there is acute deterioration). Encourage to have
an asthma diary.
3) Prescription: Step up MDI Beclomethasone 600mg OR maintain the steroid dose and
add long-acting B-agonist (Salmeterol/Seretide/Symbicort)
4) Investigation: Chest X-ray, Lung Function Test (Spirometry)
5) Observation: Ask patient to come back to reassess his asthma control
6) Prevention: Wear PPE when doing physical activity. Stop smoking.

62
63 Oceanic Series End of Posting Compilation

MEQ 43

A 28-year-old lady come to a Public Primary Care Clinic for pap smear. Her last pap smear done 1
year ago, and the result was negative and was asked to repeat pap smear 3 yearly. She has 1 child
and has been taking OCP since last 3 years. She was told by her friend that taking OCP can increase
risk of getting cervical cancer. There is no FMH of cervical cancer.

Question 1: She insisted to repeat the pap smear. What is your respond to the patient? Explain to the
patient.

Answer: It is true that the patient on OCP can increase the risk of getting Cervical Ca by 3x,
but the main cause of Cervical Ca is Human Papilloma Virus.

Question 2: List down the aspects of Fundamental Ethics and the details

Answer: JITA-CB
1. Justice (equity) - need to treat equally regardless of race, ethnic or religion.
2. Autonomy
3. Informed consent
4. Truth telling
5. Confidentiality
6. Beneficence - do good, do no harm

Question 3: If the doctor does the cervical smear as being requested by the patient, what are the
benefits doctor and patient get?

Answer:
Doctor - gain trust
Patient - less worry, help patient to detect cervical cancer early

Question 4: 2 potential harms of doing more frequent test than normal

Answer:
1. For doctor: inappropriate use of source
2. For patient: patient is subjected to unnecessary discomfort

63
64 Oceanic Series End of Posting Compilation

Patient mention about tumour markers (Ca 125) and request to do it, but willing to pay by herself.

Question 4: How do you response to the patient request? Name the fundamental issue.

Answer:

Truth telling - tumour marker test is not a screening or diagnostic test. It is performed to
monitor the progression of the disease. So, it is not recommended for the patient to do it.

MEQ 44

55 years old, complain of acute chest pain 40 min. Just came back from mecca 2 days ago, well, and
healthy. No comorbid, pulse normal. BP 140/95. Total cholesterol high. LDL is high also.

1. Differential diagnosis for

1) Cardiac – Acute coronary syndrome


2) Non cardiac – Pulmonary embolism (Just came back from Mecca – long sitting on plane
may cause deep vein thrombosis)

On ECG - ST elevation lead ii,iii and AVF with st depression lead 1 ,AVL ,v5 ,v6

2. Actual diagnosis: Inferior myocardial infarction

3. Immediate management:

1. Set iv drip
2. Nitrite – Sublingual/ iv GTN
3. Iv morphine
4. Oxygen
5. Fibrinolytic – Tenecteplase, streptokinase
6. Primary percutaneous intervention

64
65 Oceanic Series End of Posting Compilation

MEQ 45

Patient on diabetic follow up. Medication Tablet Metformin 1 g. FBS 4.5, BP 140/80 mmhg, height 150
cm, weight 65 kg, on examination patient have xanthelasma.

1. 5 overall aims of continuing management in diabetic patient

1) To control her glycaemic level


2) To prevent any further DM complications
3) To detect any complications as early as possible
4) To educate healthy lifestyle
5) To give patient a quality life

2. Always skip lunch because of busy schedule. She admitted to have shaking ,tremor of hand Give
reason and cause of her symptoms – hypoglycaemia due to low glucose in body

3.Advice based on diet and medication

1) Compliance to medication
2) Eat healthily – good portion and on time

4.Give 2 causes of her current BP increase

1) White coat hypertension


2) Hyperlipidaemia

5. 3 investigations to be done on her based on physical examination findings?

1) Fasting lipid profile


2) Liver function test
3) Renal function test

65
66 Oceanic Series End of Posting Compilation

MEQ 46

54 years old man come for follow up for hypertension. Continue to take antihypertensive drugs and
satisfied with the drugs.

1) Give TWO target organs affected with hypertension and ONE Symptom that you would like to elicit
for each organ.
• Kidney – dysuria
• Heart – chest pain

2) Give two investigations related to target organ.


• Renal function test
• ECG

MEQ 47: GLYCOSURIA IN PREGNANT WOMAN

Pregnant woman, at 28 weeks (this is her third pregnancy) was found to have a trace of glucose in
her urine.

1. 3 Differential diagnosis.

1) Gestational Diabetes Mellitus


2) Urinary tract infection
3) Overt DM

2.Things need to ask based on

1) History
• Any polyuria?
• Any polydipsia?
• Thirsty more than usual?
• Any fever?
• Any abdominal pain?

2) Past medical history


• Any DM?
66
67 Oceanic Series End of Posting Compilation

3) Past obstetric history


• Any GDM?
• Ever giving birth to a macrosomia baby?
• Mode of delivery for other babies?
• Any miscarriage?

3. Investigations to do to confirm most common cause.

• Urinalysis
Finding normal for the ix above.

4. Diagnosis and reason

• Gestational Diabetes Mellitus

5. Colour code

• Green

67
68 Oceanic Series End of Posting Compilation

MEQ 48

A 42-year-old diabetic man was admitted to the ED for worsening scrotal and perineal pain for over
three days. He also had nausea, loss of appetite and vomiting.

Initial findings are:


Body temp :39 degree
Dry skin and mucous membrane
Pulse rate 115bpm, respiratory rate 30 breaths per minute

Examination: Necrotizing scrotal and perineal fascia

Capillary blood glucose 24mmol/L


Abg result
Ph 6.9
Pco2 24
Po2 98
Hco3 10

Partially compensated metabolic acidosis

1. Which zone to triage the patient?

Red

2. Name the diagnosis and underlying cause.

• Strangulated inguinal hernia


• Cause: obstructed inguinal hernia

3. 1 other blood investigation to confirm the diagnosis.

Full blood count

68
69 Oceanic Series End of Posting Compilation

4. List 5 initial management for this patient.

1) Ensure airway, breathing, and circulation are secured.


2) Set up the iv drip
3) Give oxygen
4) Give iv analgesic
5) Refer to surgery team
6) Beta blocker to reduce the heart rate

5. The doctor wanted to give the vaccine above to the child.

a) Describe how the doctor can give the vaccine above to the child with the missed vaccination.
Give both at the same time at different locations or give it a month later to avoid any
reactions.

b) Route and site of administration. Intramuscular at anterolateral side of mid-thigh

69
70 Oceanic Series End of Posting Compilation

MEQ 49 GYNAECOLOGY

A 36-year-old lady presented with fever and chill during postnatal check-up. She just delivered her
baby 3 days ago. She was allowed to go home after delivery.

(a) List 4 cause of differential diagnosis for her fever.

1) Mastitis

2) Endometriosis

3) Infected episiotomy wound

4) Urinary tract infection

(b) Apart from perineal exam, what other systems to be examined and its finding.

1) Abdominal examination: Tenderness at suprapubic area / +renal punch

2) Breast examination

On examination, there was foul—smelling and purulent lochia.

(c) What are the 2 investigations to be done to confirm your diagnosis?

1) High vaginal swab


2) Culture & sensitivity
3) Transvaginal ultrasound

(d) How do you manage this patient?


Admit her and start on IV antibiotics.

70
71 Oceanic Series End of Posting Compilation

MEQ 50
36 years old, right ankle sprain at work (industrial accident). He didn’t go to work for the past 3
weeks. Asking for mc for 3 weeks.

a) Ask for 2nd MC to cover 3 weeks previously to claim insurance.


a. Cannot give MC. Patient need to come back previously if want MC.
b. Can give/do report, for only 3 days.

b) How long record should be kept in industrial accident?


a. Should not be destroyed
b. Put aside & keep it.

Chickenpox, febrile. Get 3 days MC. Want to claim insurance.

c) Still have healing lesion.


a. Chickenpox → take at least 2 weeks to heal.
b. Doctor should give follow up.

d) Essential good medical record?


c. Relevant detail of history.
d. Examination finding.
e. Differential diagnosis.
f. Treatment and investigation.
g. Follow up & management.
h. Record RAPRIOP.

71
72 Oceanic Series End of Posting Compilation

MEQ 51

Mrs. Kim 52-year-old teacher, referred to GP because she was found anemic during checkup for
blood donation. She had history palpitations and breathlessness on exertion. She thought it was due
to anxiety due to her stress at work. She also complained she had indigestion on and off and taking
antacids in nearby pharmacist.

1. Interpret the result: microcytic, hypochromic anemia

2. History to elicit

1) Fatigue?
2) Loss of consciousness?
3) Any medical illness?
4) Any family history of Thalassemia?
5) More details history of drug

3. Differential diagnosis

1) Iron deficiency anemia


2) Thalassemia

4. Relevant investigations

1) Serum ferritin
2) Hb electrophoresis

5. How would you manage this patient

1) Give tablet ferrous fumarate


2) Educate on stress management

72
73 Oceanic Series End of Posting Compilation

MEQ 52

Patient was on Amlodipine and hydrochlorothiazide. All examinations are normal. Bp 140/90 except
creatinine 108 to 140.

1. What further investigation you would do?


1) Buse urea serum electrolyte
2) Renal function test
3) Random blood glucose
4) HBA1c

2. What are possible causes of kidney deteriorating?


1) Hyponatremia
2) Hypokalemia
3) Hyperuricemia

3. How would you manage?


1) Change to another drug
2) Regular checkup of renal function

73
74 Oceanic Series End of Posting Compilation

MEQ 53

18-year-old girl complaints of episodes of recurrent headaches for past 1 year. The headache was
unilateral, temporal region and described as "hammer-banging her head". She had nausea and
photophobia. She noticed these symptoms appear when she is sleep-derived, hungry, and stressed
out. These symptoms were disabling till she cannot go to class and had to "sleep it off". She had
these symptoms most frequent nearing her SPM examination.

She has these episodes 2-3 times per month. After her SPM examination, she still had recurrent
headaches. Her mother also has headaches. She looked well and physical examinations were
unremarkable. She had frequent abdominal discomfort during childhood but resolved on its own.

She consulted you if it’s a sinister cause. She wanted to do a brain scan.

1) Most likely diagnosis and 2 reasons

1. Chronic migraine
2. Unilateral headache associated with photophobia (typical presentation migraine)
3. Relieved by rest in a dark room
4. Family history of migraine (mother)

2) List 6 indication for neuroimaging in such cases.

1. Loss of consciousness
2. Projectile vomiting
3. Personality changes
4. Neurological symptoms or deficits such as parasthesia or paralysis
5. Aggravated by movement
6. Presence of fever (meningitis)
7. Seizures
8. Post-trauma injury

74
75 Oceanic Series End of Posting Compilation

3) Give your outline management

R- Explain patient its migraine, common condition, not likely brain Ca or something serious.
no need brain scan

A- Rest in dark quiet room. Stress management. Time management. Get enough sleep. Avoid
triggering factor, such as bright light, strong odors.

P- Tablet Sumatriptan 100mg OD x1/12, Tablet metoclopramide 10mg PRN x3/7

R- no need now

I- No need now

O- Come back if symptoms persist or worsen immediately.

P- Safe sex (STD prevention), safe driving, safe drinking etc.

MEQ 54

Low back pain, after lifted a cement pack, moderate pain, sharp in nature, radiate to back of his legs,
aggravated by movements, relieved by rest

1) Probable diagnosis – prolapsed intervertebral discs

2) 2 differential diagnosis – spondylolisthesis, spondylosis

Experienced low back pain before, chronic smoker, normal BMI

3) Non pharmacology management – change the job scope

4) Pharmacology management – analgesic

5) Investigations – cerebral spine x-ray, CT scan of cerebral spine

75
76 Oceanic Series End of Posting Compilation

MEQ 55

70 years, dizziness upon getting up from bed in the morning (on/off) for the past 3 days after a bout of
diarrhea episode. She described the symptoms as near black out.

• Near black out, think of possible system involve CNS: Transient Ischemic Attack, CVS: MI,
Cardiac arrythmia

No Spinning sensation (vertigo), from position changes (ask history of vision problems etc). She has
U/L HTN and currently on Hydrochlorothiazide. BP examination today is 140/90 mmHg. Investigation
normal. She lives alone, all children are abroad.

• As she was on Thiazide, we interested on her renal profile

1. Probable dx
Orthostatic hypotension (postural hypotension) secondary to dehydration

2. 2 Differential Dx
Transient ischemic attack, Myocardial infarction, Cardiac arrythmia

On General examination, oral mucosa slightly dry, skin turgor normal.

3. What is the relevant physical examination and expected findings?


Vital sign • BP drop (drop of 20mg systolic or 10mg diastole)
• Pulse: regular rhythm (relevant negative for cardiac arrythmia)
• Inc Heart Rate at least 30beat upon standing
CVS • No murmur, no carotid bruits
CNS No pass pointing, no dysdiadochokinesia
Normal tone, reflex, coordination (relevant negative for TIA)

All investigations are normal.

76
77 Oceanic Series End of Posting Compilation

4. What are the management for this patient?

• Reassurance: not serious problem, due to diarrhea, dehydration


• Advice: increase fluid intake, dont get up too early, sleep with the head of the bead
slightly elevated, wear compression stocking, exercise leg before standing
• P: ORS
• R: not needed
• I: no need because previous were normal
• O: Follow up in 1 week to review her symptoms, blood pressure
• P: Fall prevention, Colon cancer screening

MEQ 56

1. Describe the lesion:


Vesicle papule plaque honey coloured crusted lesion, single plaque with satellites
lesion

2. Location and distribution


Single plaque with satellites lesion

3. Diagnosis
Non bullous Impetigo

4. Name 2 causes
Streptococcus Pyogenes, Staph Aureus

5. Treatment and duration


Tablet Cloxacillin in 5 to 10 days

6. Prevention of recurrence
Finish antibiotics, treat condition properly, wash hands with antibiotics, cut nails, treat
the source of infections, practice hygiene (cut nails, hand clean, wash with soap,
identify and treat source of infection, treat carrier site)

77
78 Oceanic Series End of Posting Compilation

7. Prevention of transmission
Stay away from schools 24hrs until crust is dried, cover wet towel, regular hand
washing, wash your towel regularly

MEQ 57

A 35-year-old man came to clinic complaining of Upper Abdominal Pain for two days and loss of
appetite. Last night he vomited food particles once. He could only tolerate small number of fluids. He
had several similar symptoms last 3 years since he started to work as a salesman. He also has
frequent headaches, and he smokes 10 cigarettes per day.

1. 2 Probability Dx
Acute gastritis, peptic ulcer, AGE, food poisoning

2. 2 Red flag
Acute appendicitis, acute pancreatitis, acute cholecystitis

3. 2 Often missed
Depression, anemia, Anxiety

Mild tenderness of upper abdomen

4. Probable diagnosis and 3 Risk Factor


Acute gastritis/stress induced gastritis
Risk factor: NSAIDS, stress, smoker, alcohol

5. Management

• R- explain that it is due to gastritis, this can be treated if patient compliance,


exercise, healthy lifestyle, avoid spicy sour food, reduce stress,
• P- antacids, PPI (name, dose)
• R- quit smoking
• P-educate about sex (related), time management

78
79 Oceanic Series End of Posting Compilation

MEQ 58
A 32 y/o man had travelled to Vietnam, had an affair. He had sexual intercourse with his wife. Went
to the GP and was diagnosed with gonorrhoea. Dr advice to bring his wife to test for gonorrhoea but
he refused to tell his wife about his affair.

A) Ethical principle for the patient regarding not telling the wife about the affair.
Autonomy/consent
B) Ethical principle for not telling the truth to his wife.
Confidentiality

The husband said his wife was due for her medical check-up follow-up, so he asked the doctor to
diagnose his wife as urinary tract infection but give gonorrhoea treatment.

C) Based on this situation, give 2 ethical principles and details on the GP action.
1) Truth telling, you need to tell the truth with no lies.
2) Autonomy/Consent, need to get consent to do investigation and treatment.

The patient did not come for follow up for nine months. Later, the patient came in with a depressed
face as his recent HIV test came out positive. He did not tell his wife regarding the gonorrhoea as his
wife is asymptomatic. Currently, his wife is 4 months pregnant with their first child.

D) What advice do the GP want to give to the patient?


1) Need to tell his wife regarding the results.
2) Bring his wife to the clinic to do an HIV screening test.

E) List four aspects the GP should discuss with the patient after HIV counselling.
1) Report to medical teams.
2) Advice the patient to tell his wife within 2 days.
3) Practice a protective sex like using condoms.
4) Refer him to HIV clinic

79
80 Oceanic Series End of Posting Compilation

MEQ 59

A 46-year-old male has been attending follow up for the past 18 months for elevated blood pressure.
His blood pressure fluctuates between 130/90 mmhg- 160/110mmhg. He is asymptomatic.
Recently, he was promoted to regional sales manager in a pharmaceutical shop. His wife is a
homemaker. He has two children who are college students. He smoked 20 cigarettes per day but
didn't drink alcohol.

The blood pressure for the current visit is 150/85mmhg, height: 150cm, weight:72kg.
Lipid profile finding:
• Total cholesterol: high
• HDL: normal
• LDL: high
• Triglyceride: normal

1. State 4 problem list from the above scenario:


He was reluctant to start treatment because he was asymptomatic. Explain to him that it is
important to start the treatment.

2. State 2 different target organ damage due to hypertension with manifestation of the complications.
• Heart
• Kidney

3. State two relevant investigations to check for the target organ damages.
• Renal function test
• ECG

4.State two non-pharmacological interventions relevant to the scenario. Be specific where appropriate.
• Stop smoking
• Stress management

5. State the treatments with the starting dose you want to initiate.
Tablet Perindopril 2 mg OD

80
81 Oceanic Series End of Posting Compilation

MEQ 60

A 19-year-old female came to the health clinic with fever and chills for 3 days. She also developed
rashes and backache which is not relieved by paracetamol.

1. list all the differential diagnosis


1) Probability: dengue fever
2) Serious: leptospirosis
3) Pitfall: drug-induced allergy
4) Masquerade: malaria

2. list all the history you want to obtain


1) History of illness: duration of fever
2) History of illness: severity of backache
3) Family history: any family members with the same symptoms
4) Social history: any history of travelling

Examination was done. Her temperature was 37.8. pulse 88. bp 110/80. NS-1 antigen was positive.
you also noticed there is petechiae at the cubital fossa. state your management

• R: explain to the patient that she had been infected with dengue. Reassure that it can
be treated with medication
• A: drink plenty of water and have a good rest at home
• P: tab paracetamol 1 gm qid x 5/7 (NSAIDs are contraindicated in dengue!!)
• R: no referral needed. Must notify pejabat kesihatan daerah
• I: fbc, coagulation study,
• O: daily monitoring of symptoms
• P: wear long sleeve shirt whenever going out

81
82 Oceanic Series End of Posting Compilation

MEQ 61
10-year-old boy come to the public clinic because of his diarrhea, nausea, and vomiting. He has mild
dehydration and treated as out-patient.

1. Fill in the table on the difference between mild and moderate dehydration according to given
indicators (refer Murtagh page 498)

Indicators/Dehydration status Mild Moderate


General appearance Restless Lethargic
Oral mucosa Normal Dry
Skin turgor Normal Retracts slowly
Urine output Normal Decreased

2. Name 2 responsible organisms and the typical food they are usually associated with, in causing
food poisoning (refer Murtagh 491)
a. Staphylococcus aureus – custards and cream
b. Salmonella species – chicken/meat

3. Management: RAPRIOP

82
83 Oceanic Series End of Posting Compilation

MEQ 62
An 18-year-old veterinary student presented with wheezing and cough with whitish sputum for the past
2 months. He has history of childhood eczema, and her mother has asthma. Two months ago, she
started her industrial training in a chicken farm. Dr diagnose him with bronchial asthma.

1. List 3 reasons to support the diagnosis based on the scenario


a. Childhood eczema
b. Family history of asthma
c. Triggering factor – chicken feathers

2. List 4 findings expected to see during physical examination in asthma patients


a. Skin condition – any marks of scratch due to allergic or eczema
b. Use of accessory muscle to breath, any recession?
c. Chest movement
d. Chest expansion equal or not?
e. Auscultation usually reveals diffuse high-pitched wheeze throughout inspiration
and most expiration.

3. State one diagnostic test which can be done in primary care setting and state its expected
finding
a. Test: spirometry
b. Finding: FEV1/FVC post bronchodilator >12 %

4. Management – RAPRIOP

83
84 Oceanic Series End of Posting Compilation

OSCE

84
85 Oceanic Series End of Posting Compilation

OSCE 1 HYPERLIPIDEMIA

1. Describe the distribution of the lesions in the picture shown.


Bilateral, symmetrically, medially, and at both upper and lower eyelids

2. Describe the morphology and name the type of lesion.


Pale and yellowish, well defined, nodules at upper eyelid and plaque at lower eyelid

3. State the name of the lesion.


Xanthelasma

4. Describe the pathophysiology of this lesion.


Accumulation of lipid-containing macrophage in the dermis

5. What is the clinically significant of this lesion?


50% patient that has this lesion have hyperlipidemia.

6. What biochemical test should be done?


Fasting lipid profile

7. What is the treatment and its disadvantage?


Laser treatment. The lesion can reoccur.
85
86 Oceanic Series End of Posting Compilation

OSCE 2 URINE DIPSTICK

1. Steps before press the timer.


i. Check expiry date ii. Wear gloves
iii. Take dipstick strip iv. Close the container

2. Steps to remove excess urine from the dipstick strip


i. Slide the edge at the rim of the vessel
ii. Dab behind the strip briefly with tissue

3. When should start reading the result?


More than 60 seconds, less than 120 seconds

4. Two steps before interpreting the result


i. Discard the strip into the clinical waste bin
ii. Remove the gloves and discard into the clinical waste bin

5. Interpret the result

86
87 Oceanic Series End of Posting Compilation

OSCE 3 SKIN

A child was brought in by his mother with fever for 2 days and had refuse food since yesterday
evening. He takes only sips of milk. This morning his mother notices a rash on both ofhis hands.

(a) Describe the rash.


1) Multiple lesions scattered on the palmar aspect of the hand.
2) Erythematous, fluid-filled lesions
3) Vesicles
4) Elevated lesions, <10mm

The child was not toxic, has ulcers in his mouth and both his feet.

(b) Diagnosis: Hand, foot, and mouth disease


(c) Causative agent: Coxsackie virus

(d) List 2 modes of transmission.


1) Direct contact
2) Fluid
3) Stool
4) Contaminated individuals

(e) List one complication.


1) Encephalitis
2) Meningitis
87
88 Oceanic Series End of Posting Compilation

OSCE 4 DEELOPMENTAL MILESTONES

(a) Name the maneuver (pic). Pull-to sit

(b) Expected findings in 6 week and 12-week-old baby.

6 weeks 12 weeks
During the maneuver Head lag present Slight head lag present
After maneuver Heads occasionally bobs
Holds head up momentarily in
forward
same plane as body

(c) List the 2 milestone and developmental area assessed.

• Milestones: Head
and trunk control
• Developmental area:
Gross motor

(d) The developmental area assessed for palmar grasp


Fine motor

OSCE 5 PAP SMEAR

88
89 Oceanic Series End of Posting Compilation

Follow the instruction from Doctor’s sheet for clinical skill labs

Take note:

a. Check the equipment (look, show and tell the examiner each equipment)
b. Label the slide
c. Explain the patient’s position CLEARLY and tell the patient to undress from waist
down
d. Choose the right size of speculum
e. Choose the right part of wooden spatula for the patient

OSCE 6 ACUTE VIRAL CONJUNCTIVITIS (STATIC STATION)

Mrs Linda, 56 years old come to your clinic with itchiness on her left eye, her husband had
the same symptoms1 week ago. She has normal eyesight and is not a contact lens user.

List 3 other eye symptoms

1. Eye pain
2. Eye discharge
3. Eye redness

List 2 possible differential diagnosis that you would like to refer her to the ophthalmologist

1. Dx: Acute angle closure glaucoma


Specific symptoms: Frontal headache, eye pain
2. Dx: Corneal ulcer
Specific symptoms: Photophobia

89
90 Oceanic Series End of Posting Compilation

List 4 clinical features (normal or abnormal) that you would like to include

No pus, no crust, no foreign body, no swelling of eyelid, pupil is no dilated/normal,


corneal is clear.

Acute eye condition that makes you would like to refer her to ophthalmologist based on her
history

Acute angle closure glaucoma

Final diagnosis

Acute viral conjunctivitis

OSCE 6 SUGGESTIVE OF MALIGNANYCY

OSCE 7 COMMUNICATION SKILL – ETTIC

90
91 Oceanic Series End of Posting Compilation

You recently consulted a 16-year-old girl and gave her OCP pills. She also has a boyfriend.
Based on your judgement she was matured enough to receive the pills (Gillicks competency)
. Today the mother of the girl gave you her visit and was demanding you to tell her regarding
her daughter’s OCP pills which she found with your name on it as the prescriber

How would you confront this lady?

• Do not ever reveal anything about the daughter or her personal information as well
as her medical notes. DO NOT reveal why she had was given an OCP.
• Even though the daughter was underage, you had given the OCP as you judged her
to be matured enough so you must respect and maintain confidentiality as how you
would treat an adult patient.
• Advise the mother to sit down with her family and discuss the issue with the daughter
• If that is not possible you can arrange a meeting for both to come and discuss this
matter with you.
• You can’t deny that you didn't give OCP to her daughter because of the OCP in your
clinic package and your name was there. This one was not considered break the
confidentiality.
• If you told anything other than point above, you fail = (6/20)
• Benda2 yg buat kita fail.
o Bagitau dia dtg dgn siapa.
o Past record hx UTI and AGE acute
o She understands contraindications of OCP smua ni.
• Introducing yourself and addressing patient by name
• Nonverbal & verbal communication skill
• Appropriate use of language non-medical Jargon.
• Exploring patient reason for attendance about finding the OCP, wanting to know why
the Dr prescribed OCP to her daughter
• Able to respond the patient anger.
• Show willingness to talk and listen
• Allow pt. to ventilate her anger
• Respond appropriately to pt. consent and expectation, clarify pt. consent and
expectation
• Express willingness to help but maintain ethical principle and confidentiality
• Explore family, friend, and social relationships
• Show empathy and understanding pt. feel
• Ask for further queries or need any more clarification

91
92 Oceanic Series End of Posting Compilation

• Summarized.
• Safety net.
• How to end this
• From my understanding the reason why you come today because you found the
OCP that was prescribed by me. I hear your frustration and I understand your
consent as well. But unfortunately, I must follow my ethical principle I cannot reveal
anything about the consultation I had with your daughter. I’m sorry about that. You
should try to approach your daughter again to ask her to come with you or if you
are willing, I can try to ask her if she okey to discuss this with you or she just come
in with you to discuss this together but unless I get a consent unfortunately, I were
not able to say anything else about the consultation. Do you anything else to ask?
Thank you

92
93 Oceanic Series End of Posting Compilation

OSCE 8 INSULIN PEN

1. Ask the patient’s name and introduce yourself.


2. Explain the intention of the session: “medication is not adequate because your DM is not
controlled, so you need to add insulin injection using insulin pen (Novopen). So, I would
like to demonstrate to you on how to use it”
3. We must wash hands before start
4. Remove the cap & cartridge holder. Tell the parts of the insulin pen:

5. Check name & expiry date


6. Roll the cloudy insulin – mix horizontally (10x)
7. Insert the insulin into cartridge holder & fixed in the pen
8. Swab the cartridge head
9. Remove the outer cap & set it aside, remove the small inside cap & immediately discard
10. Priming: tap to remove air bubbles (pen is in vertical position) Prime (pull & rotate) by
2 units, 2 units until insulin is inserted
11. Turn to 10 units (according to question as prescribed)
12. Explain about the injection sites:
o Upper outer arm
o Thigh
o Buttock
13. Peri umbilical region / belly button -> the best injection site for better absorption.
However, we must rotate the injection site about 2 fingers away around the belly button
to prevent scarring of the skin
14. Clean the injection site with alcohol swab, let the alcohol dry, pinch the skin and inject it
perpendicularly. Wait for 6-10 seconds for the insulin well absorbed.

93
94 Oceanic Series End of Posting Compilation

15. Recap the outer cap and removed together with the needle into the sharp bin or steel
dustbin
16. How to store the insulin pen?
Not in the too cool or dry: fridge. Ok to keep in cupboard or drawer. Regarding the insulin
which has not been used yet, you may have to store them in the fridge.
17. “Can you understand?”
18. “Can you demonstrate to me based on what you understand?”
19. “Is there anything you would like to ask”
20. Explain about hypoglycaemia
o It is a condition when the glucose level in your blood is low.
o Signs and symptoms: sweating, lethargy, intense hunger, headache, confusion.
o What to do? = bring sweets / candy in your pocket
21. Suggest for home glucose monitoring
22. Thank you

TIPS

• The statement was “I am here to teach you on how to use insulin Pen because oral
medication isn’t enough to control your DM”. No need to go through the history and
oral antidiabetic agent.
• Don’t say insulin PENSIL.
• See the name of the medication properly. Now, there was insulin Rapid, Insulin
Combo and Insulin Basal. (What we need at that time was Insulin Basal)
• If the insulin is cloudy, roll until UNIFORM, not until CLEAR.
• If roll horizontally, make sure the position is flat. If invert, invert it for 180 degrees.
• Swab @ tip of insulin cartridge, not at the outside/beyond
• Mention to do priming for 2 units.
• If you want to recap the outer cap, make sure to hold the cap at the lateral side.
• Explain to the patient to have a proper disposable puncture proof such as milo or
biscuit tin at home.
• If have time, explain about storage, hypoglycemic, blood glucose monito
• Fail if: Recap with small cap, not disposing sharp needle properly

OSCE 9 BREAKING BAD NEWS

94
95 Oceanic Series End of Posting Compilation

95
96 Oceanic Series End of Posting Compilation

96
97 Oceanic Series End of Posting Compilation

OSCE 10 OTITIS MEDIA (STATIS PICTURE)

Otoscopy findings

1. Symptoms

a. Fever

b. Otalgia

c. Otorrhea (discharge)

d. Rabbling of ear

2. Findings from the otoscopy

a. Redness of the tympanic membrane

b. Bulging of tympanic membrane

c. Acute perforation of ear drum with pus visible


in the external ear

d. Pus accumulation

3. Commonest cause

a. Viruses: Respiratory Synctitial virus, rhinovirus

b. Bacterial: Pneumococcus, H. influenza

4. Complications

a. Speech difficulty

b. Learning difficulty

c. Loss of hearing

5. Diagnosis = Acute Otitis Media

6. Management

a. Painkiller: Paracetamol

b. Regular analgesia

OSCE 11 RED EYES: ACUTE VIRAL CONJUNCTIVITIS (STATIC PICTURE)

97
98 Oceanic Series End of Posting Compilation

Mrs L, 56 years old come to your clinic with redness of her left eye, her
husband had the same symptoms 1 week ago. She has normal eyesight
and she is not a contact lens user.

1. Other symptoms

a. Eye discharges

b. Itchiness

c. Lacrimation

d. Swelling

e. Pain – sand like grittiness

2. Two possible diagnosis that must refer to ophthalmologist

a. Acute angle closure glaucoma - severe frontal headache, eye pain

b. Corneal ulcer- photophobia

3. Describe the distribution of the redness based on the picture

a. Generalized

b. More on peripheral (lateral/medial cantus)

c. Not at circumcorneal region

4. Four clinical features

a. Normal pupil/ pupil is not dilated

b. Clear cornea

c. No purulent discharge / pus

d. No foreign body seen

e. No swelling of the eyelid

5. Eye condition (red flag) / Acute eye condition that makes you would like
to refer her to ophthalmologist based on her history

Acute angle closure glaucoma

6. Most likely diagnosis/final diagnosis

98
99 Oceanic Series End of Posting Compilation

Acute viral conjunctivitis

7. Management

a. Artificial eye drops

b. Proper hygiene

99
100 Oceanic Series End of Posting Compilation

OSCE 12 ACUTE VIRAL PHARYNGITIS

1-day hx of sore throat. Well, no activity limitation.

a. What other symptoms to ask?

• Fever with chills and rigor


• Body aches
• Stuffy nose
• Cough. Productive not?

b. 2 +ve and 2 –ve findings in the picture?

∙ 2 +ve:

o Injected uvula

o Uvula oedema

∙ 2 –ve:

o No tonsillar enlargement

o No exudates from tonsils

c. Additional physical examination findings you would look for and


give reason?

• Cervical lymphadenopathy – if present, more likely


from strep infection ∙
• Temperature – higher grade: strep infection.
• Lungs – listen for rhonchi/crepitation.
• Skin rash – indicates infectious mononucleosis

d. Provisional diagnosis?

Acute viral pharyngitis. (If u left out acute, u only get half the marks)

100
101 Oceanic Series End of Posting Compilation

e. 4 management measures

▪ R: Reassure that this is of viral cause, and it is self-


limiting in 3 days ∙ A: drink lots of water, rest
▪ P: paracetamol for fever, mouth gargle
▪ O: to come back if sore throat persists for the next 4-5 days

101
102 Oceanic Series End of Posting Compilation

OSCE 13 GLUCOMETER

1. Greet the patient, introduce yourself

2. Explain about the session: “ I am going to guide you on how to use the glucometer in order
to determine the approximate concentration of glucose in the blood”

3. Equipment:

1) Glucometer
2) Test strips
3) Lancet holding device
4) Lancets
5) Alcohol swabs
6) Dry swabs
7) Yellow bin for disposal of sharps
8) Nonsterile gloves

4. Wear the non-sterile gloves

5. “Can you understand?” “Can you demonstrate to me based on what you understand?” “is
there anything you would like to ask”

6. Thank you

OSCE 14 PEFR – ASTHMA


Madam Janice, recently diagnosed with bronchial asthma for 1 month
ago, but still having SOB Teach her how to do peak flow metre and

102
103 Oceanic Series End of Posting Compilation

interpret the result.

Indication to do PEFR:
1. Uncontrolled asthma
2. Based on asthma action plan
3. Based on Asthma diary

1) Introduce, address patient by name


2) Move the marker to the bottom
3) Your finger should not block the indicator
4) Stand up straight
5) Before blowing, take a deep breath
6) Hold your breath while you put the mouthpiece in your mouth, b/w teeth. Make sure
tongue does not block it.
7) Blow as hard and fast in a single blow
8) Look at where the indicator, write down the number you get
9) After not the indicator, push back to 0 point. Repeat steps for 3times.
10) If you cough, did not do the steps right, repeat agai
11) Check the understanding

Interpretation
• Personal best is 400
• Highest PEFR = 350
• Percentage is 350/400 x 100 = 87.5 %
• Severity of asthma: mild bronchial asthma
• When can be use: every morning when you wake up
• Classification of asthma: mild, moderate, severe

103
104 Oceanic Series End of Posting Compilation

What factors to look for to classify his asthma control?


1) Daytime symptoms
2) Night-time symptoms
3) Use of reliever
4) Activity limitations
5) PEFR

What is his PEFR? Take the highest from the 3 readings given

Determine predicted value of his PEFR from chart given according to his height and age.
4 possible causes for his uncontrolled asthma?
1) Exposure to dusty environment
2) Improper inhaler technique
3) Non-compliance
4) Occupational factor
5) Wrong dosage
Management?

• R: Explain what asthma is. Reassure it can be controlled/improve


• A: Advice to avoid trigger factors. Correct inhaler technique
• P: Add MDI corticosteroids
• I: PEFR
• O: Follow up next month
• P: Quit smoking

104
105 Oceanic Series End of Posting Compilation

OSCE 15 METERED DOSE INHALER


1. Introduce, explain about the session.
2. Wash hands
3. Parts of the Inhaler:
a. Canister
b. Plastic case
c. Mouthpiece cover
d. Base
e. Mouthpiece (when take off the cover)

4. Before you use the device, always check it inside and out to make
sure that it is clean. You don’t want any dirt, dust, or fluff in the
mouthpiece
5. If this is a new device, of if you haven’t used it for a week or more,
you’ll need to test it. To do this, shake it well.
6. Hold it away from you; press the top of the canister once to release
one puff into the air. It should produce a fine mist like this. If it
doesn’t, check that there’s nothing blocking the mouthpiece. If it is still
doesn’t work, you should speak to your pharmacist.
7. This is how to use the device. It may take a few goes before you get
the hang out of it. Just relax and don’t rush yourself. At first, you might
find it useful to use the device in front of a mirror. The aim is to inhale
the fine mist into your lungs.
8. Begin by shaking the device well
9. Then, hold it upright like this with your thumb on the base below
the mouthpiece.
10. Breathe out as far as you can without it feeling uncomfortable.
11. Then, place the mouthpiece in your mouth between your teeth. Close
your lips around the mouthpiece, but don’t bite it.
12. Start to breathe in as slowly as possible through your mouth. Just as
you do this, press down on the top of the canister to release a puff
while still breathing in steadily and deeply. Then, hold your breath
13. If you see mist coming from the top of the device or… from the
sides of your mouth, you’re doing it wrong, and you should start
again.
14. Still holding your breath, take the device out of your mouth and your
finger away from the top of the canister. Continue holding your breath

105
106 Oceanic Series End of Posting Compilation

for a few seconds more, or for as long as you can without it feeling
uncomfortable.
15. If you are meant to take another puff, shake the device again, and
repeat the whole process
16. When you are finished, always replace the mouthpiece cover by
snapping it back on.
17. Rinse your mouth thoroughly with water. Spit out the water, do
not swallow.
18. If you have any problems using the device, please speak to your
pharmacist, nurse or doctor. Thank you.

Oceanic Question Bank 1st Edition


ANAESTHESIOLOGY 106
107 Oceanic Series End of Posting Compilation

Contents
EMQ ....................................................................................................................................... 108
MEQ ....................................................................................................................................... 110
MEQ 1 Intubation...................................................................................................................... 110
MEQ 2 Oxygenation ................................................................................................................. 112
MEQ 3 ........................................................................................................................................... 114

107
108 Oceanic Series End of Posting Compilation

MEQ 4 ........................................................................................................................................... 116


MEQ 5 ........................................................................................................................................... 117
MEQ 6 ........................................................................................................................................... 118
MEQ 7 Spinal anesthesia ............................................................................................................... 118
MEQ 8 ........................................................................................................................................... 120
MEQ 9 ........................................................................................................................................... 120
OSPE ..................................................................................................................................... 122
THORACOCENTESIS PROCEDURE ................................................................................................... 141
LUMBAR PUNCTURE OSCE ............................................................................................................ 141
BASIC LIFE SUPPORT ...................................................................................................................... 141
EPIDURAL NEEDLE ......................................................................................................................... 142

EMQ

1. Oxygen percentage usually given by ventilator when 100%


patient is intubated during GA.
2. A patient went to the clinic and his doctor ask him to open Mallampati Class 3
his mouth. Opening of mouth shows only uvula and soft
palate. The fauces, pillars are not seen.

108
109 Oceanic Series End of Posting Compilation

3. An athlete was touching and rubbing his ankle with both Modulation of
hands to get relief from the ankle sprain, stimulating the spinal dorsal horn
A beta fibre which are larger than A delta fibre and C fibre ganglion
4. Bier’s block was done on upper limb with a tourniquet for Venous
a short surgery administration
5. Eutectic Mixture of Local Anaesthesia (EMLA) was given Covering the skin
to a child before inserting an IV line
6. Face to face with patient. Hard palate, soft palate, uvula, Mallampati Class II
fauces can be seen. Pillars not visible.
7. Epiglottis, posterior half of voice cord are visible Cormark-Lehane
Grade II
8. Moribund patient with road traffic accident needs urgent ASA V
intubation
9. Patient with mild hypertension come for facial surgery ASA II
10. Percentage of oxygen via Mouth to mouth 16%
11. Mild hypertension, prepared for emergency surgery of ASA IIE
ruptured appendicitis
12. Brain death emergency organ donation ASA IVE
13. Oxygen percentage given during preoxygenation. 88 – 90 %
14. On auscultation, air entry into left lung noticed, but absent Left Main Bronchus
in the right lung. No gurgling sound or other sound in Intubation
epigastrium.

15. A lady planned for elective plastic surgery and had mild ASA II
DM under metformin. What is her ASA classification?
16. Dorsum of the hand is numb to give iv cannulation EMLA
17. Remove Foreign Body from eye Instillation
18. Percentage of oxygenated Hb with pvo2 75%
19. volume of exhale normal quietly 500ml
20. Total body water of a 70 kg man 42 L
21. Volume of third space 11L
22. This is the site where spinal cord ends. Between L1 and L2
23. The level of depression of nervous system is monitored in BIS monitor
general anesthesia
24. Monitor CO and peripheral resistance sphygmomanometer

109
110 Oceanic Series End of Posting Compilation

25. This line shows the highest level of lumbar vertebrae


when someone was lying down. Spinal anest can be
L3 vertebrae
injected below this.
26. oxygen concentration pre oxygenation was given before Denitrogenating
intubation. nitrogen was removed and oxygen replaced it
in the functional residual capacity of the lungs
27. a road traffic accident with moribund condition ASA 5
28. A normal healthy patient come for breast surgery ASA 1
29. Patient with ruptured abdominal aortic aneurysm, cannot ASA 5
survive with or without
30. Patient with limited functional disability, uncontrolled ASA 3
diabetes mellitus prepared for
31. EMLA was used on a child before cannulation local application
32. Ancient Chinese practice to ease labour pain. Acupuncture
33. Inhalation agent used for obstetric analgesia. Entonox
34. Mount Everest oxygen concentration. 21
35. Oxygen concentrations give to newborn. >21
36. A 24-year-old lady presented with generalised urticarial, Intubate the patient
bilateral periorbital swelling and giddiness after she was
stung by a swarm of bees. On examination, she was alert.
She was having hoarseness of voice, and tachypnoeic.
Her vital signs were BP 85/55, HR 100 & SPO2 97% on
room air. On auscultation of her chest, generalised
rhonchi were heard.
What is the best immediate action to be taken?

MEQ
MEQ 1 Intubation

1. 5 place to auscultate to confirm intubation


2. Appearance of tube outside of mouth on inspiration and expiration, and give reason
3. How to visualize the tube during intubation?
4. How does the anaesthetist know by hand?
5. Placement of tube on the X-ray
6. Appearance of the chest wall during ventilation

110
111 Oceanic Series End of Posting Compilation

111
112 Oceanic Series End of Posting Compilation

MEQ 2 Oxygenation

Patient was sent for a surgery, and a general anaesthesia was performed.
1. Pre-oxygenation for this patient?
• Percentage
100%
• Circuit
General anaesthesia circuit
• Duration
3 to 5 minutes
• Reason:
To wash out Nitrogen and for super-saturation of Oxygen in the lung, allowing
if prolong intubation needed.

2. 3 intravenous induction drugs


• IV Propofol
• IV Midazolam
• IV Rocuronium

3. A laryngoscope is used to assist the endotracheal intubation.


• The time to remove the mask.
• State where in the mouth, the tip of the blade is placed
• Manoeuvre of the blade?
• Final position of the tube?

4. State
• When is the time to inflate the cuff?

112
113 Oceanic Series End of Posting Compilation

• When to anchoring the tube around the mouth?

5. Name a drug for each group


• Muscle relaxant
• Vapour
• Analgesics

6. Name the reversal drugs for the following drug


• Midazolam antagonist
Flumazenil
• Opioid antagonist
Naloxone

113
114 Oceanic Series End of Posting Compilation

MEQ 3

A 65-year-old man need spinal anaesthesia for elective transurethral resection of prostate.
Blood pressure was 140/85mmHg, heart rate of 80 beats/min. On auscultation, there is
bilateral rhonchi heard. ECG showed mild ischemic heart disease. However, he is not keen to
do spinal anaesthesia.

a) What is the ASA classification and state your reason?

ASA 3, Patient is currently having hypertension which has escalated to Chronic heart
failure being the presence of bilateral rhonchi heard

b) State four advantages of spinal anaesthesia in this case.

• Less post operative pain


• Reduce incidence of DVT
• Less bleeding
• Preserved protective reflexes
• Reduce risk for aspiration
• Cost effective
c) Patient refuse to do spinal anaesthesia. Do you still insist on doing spinal anaesthesia?
Give your reason.

114
115 Oceanic Series End of Posting Compilation

Yes, patient has a severe systemic disease, hence preventing any exacerbation due to
GA

After getting additional information about the spinal anaesthesia, you get consent from him to
do the procedure. After 25 minutes of operation, blood pressure dropped to 80/65mmHg, and
pulse rate was 100.

d) Give the causes of hypotension in this case.

Negative influence on a relatively higher resting sympathetic tone and decreased


baroreceptor activity causes hypotension

e) State how you manage the hypotension in this case.

Preload colloid infusion, compression stocking

2. Bag Mask Ventilation Procedure

a) State 2 fingers to push down the mask.

Thumb and index finger

b) State 3 fingers to raise the jaw.

Middle, ring, and 5th finger

c) Name the technique used

d) What is the other hand are doing? Explain the procedure.

e) How many time you squeezed the bag in 5 second?

f) How many percentage of oxygen delivered through the bag valve mask?

115
116 Oceanic Series End of Posting Compilation

MEQ 4
You are the medical officer in a hospital. A man suddenly fell down 10 meters away from you
in your ward.

a) Based on National Committee of Resuscitation Training (NCORT), state first 2 steps that
you would need to do?

b) After you completed the steps above, state next 3 steps and how to do it?

c) After the steps above, state the next 2 steps?

d) Finally, before starting the compression:

i) What do you need to check?

ii) How to do it?

iii) When to start?

116
117 Oceanic Series End of Posting Compilation

MEQ 5

Mr Ahmad was to be given GA for cholecystectomy. Succinylcholine (suxamethonium) was


used during induction phase.

1. State how this drug blocked neuromuscular junction and mention its duration of action
with standard dose.

Intubation was done successfully within 3 minutes and breathing returns after 2 minutes
later.

2. Name the enzyme that metabolized suxamethonium and the reason it was possible.

Long-acting muscle relaxants were used during the operation. At the end of operation, the
anaesthetist used peripheral nerve stimulator to check residual action of relaxants.

3. List 4 stimuli used to check it.

Anaesthetists find that residual muscle relaxant action was still present. 2 drugs were used
to reverse it.

4. List the 2 drugs.

117
118 Oceanic Series End of Posting Compilation

MEQ 6

1. What is the exhibit shown? Endotracheal tube


2. Maneuver when the laryngoscope is not used? Blind intubation
3. Maneuver when patient is conscious? Awake intubation
4. Intubate from mouth? Oral intubation
5. Intubate from nose? Nasal intubation
6. When to start cricoid pressure? When patient is unconscious
7. When to stop cricoid pressure? After the tube is inserted
8. When off face mask, how many minutes do you have before patient develops hypoxia?
Up to 8 minutes

MEQ 7 Spinal anesthesia

Spinal anesthesia given to a patient. The heavy Marcaine blocked the transmission of

impulses in the mixed spinal nerves. Both the lower limbs became numb. Then, the
analgesia level rose to the umbilical level and the surgery for right inguinal hernia was
initiated.

1. Mention the 3 types of fibers in the mixed spinal nerves blocked by the marcaine and the

origin of them from the spinal cord horns. (1.5 marks)

(a) Type: Sensory

Origin: Posterior horn

(b) Type: Motor

Origin: Anterior horn

(c) Type: Sympathetic

118
119 Oceanic Series End of Posting Compilation

Origin: Lateral horn

As the time went on the level pain blockage was rising gradually. The BP was going down
slowly.

2. Mention the effect of spinal anesthetic upon the systemic peripheral resistance, why it

happened and how would you treat it. (1.5 marks)

(a) Effect: Reduction of SVR

(b) Reason: Blockage of sympathetic fibers

(c) Treatment: Give IV Ephedrine

3. Mention the effect of spinal anesthetic upon the capacity (size) of vascular compartment,

why it occurred, and how would you manage. (1.5 marks)

(a) Size: Increased, relative hypovolemia

(b) Reason: Venodilatation by sympathetic blockage

(c) Treatment: Run the NS drip down, increasing the volume

The patient was unable to raise his legs.

4. Mention reason of that inability, the effect upon the BP and the reason for that effect.

(a) Reason: Motor block

(b) BP: Hypotension

(c) Reason: Paralysis of calf muscles decreasing venous return

Finally, the level of analgesia rose to the nipple line. The BP fell more.

5. Mention the reason for the hypotension, how it occurred and how would you combat it.
(1.5

marks)

119
120 Oceanic Series End of Posting Compilation

(a) Reason: Bradycardia

(b) Cause: Cardiac sympathetic block

(c) Treatment: IV Atropine

Starling’s law is an important law for the heart.

6. Define Starling’s law. (2.5 marks)

Ability of the heart to change its force of contraction and increases in length until a

certain extent it cannot compensate anymore”

MEQ 8

1. The duration of compression (in minutes) which must be accomplished before changing

place with another person.

= 2 minutes

2. The number of ventilations in one cycle of CPR

= 2 ventilations

3. Hypotension occurs in spinal anesthesia due to the blockage of sympathetic fibers.

= B fibers

4. These nerves crosses to the opposite side of the spinal cord and passes up to the
thalamus in the spinothalamic tract to the thalamus.

= Second order neuron

MEQ 9

1. Mention the first step, to prevent your forearms from crossings each other.

= STAY AT HEAD END

2. Mention the second step regarding the facemask and why you do it.

120
121 Oceanic Series End of Posting Compilation

= CORRECT POSITIONING

3. State the five features of ‘E’ of CE technique.

(a) E SHAPE

(b) FORMED BY MIDDLE, RING AND LITTLE FINGER

(c) PUT BELOW THE BODY OF MANDIBLE

(d) E BEGINNING OF ELEVATION

(e) LIFT UP THE JAW

4. State the five features of ‘C’ of CE technique

(a) C SHAPE

(b) FORMED BY INDEX THUMB

(c) PUT AROUND MASK

(d) C BEGINNING OF COMPRESSION

(e) COMPRESS DOWN

5. After gripping the mask and jaw with CE technique, mention the next important step you

would do.

= MAINTAIN HEAD TILT CHIN LIFT POSITION

6. Mention the duration of your single squeeze.

= 1 SEC

7. Mention the meaning of CO-CPR.

= COMPRESSION ONLY CPR

121
122 Oceanic Series End of Posting Compilation

OSPE

You must have three things and study them


1. Laryngoscope.
2. ET Tube.
3. Magill’s Intubating forceps. The curve
You must know
4. Anatomy Oral cavity, Three axes, Epiglottis and Vallecula, Vocal cords, Trachea,
Bifurcation
5. Airway assessment Mallampati, Cormack Lehane, 332 Technique, Neck Head tilt
Chin lift,
6. LEMON
Check in OT how it is done
1. Positioning Pad under head
2. Pre oxygenation
3. IV anaesthetics Propofol, Thiopentone, Ketamine, Fentanyl, Midazolam, 4. Muscle
relaxant Succinyl choline Till respiration stops, notice the fasciculations
5. Extend the blade, hold with left, tip towards right of oral opening,
6. introduce and push the tongue towards left and
7. slide the tip on surface of the tongue posteriorly till it reaches the junction with the
more posteriorly placed epiglottis at the vallecula.
8. Lift the chin now, lifting the tongue together with the epiglottis revealing the inlet to
the larynx
9. Keep yourself away from the patient. Don’t be too close.
10. While holding the laryngoscope with the left hand put the tip of the endotracheal
tube into the trachea through the vocal cords. The black line just under the vocal
cords. Can use the intubating (Mac Gill’s) forceps.
11. Fill the cuff with air 5-10 ml. Listen to the leaks.

ABCDEFG Auscultation at five areas, Breathing out fogs the tube, Chest expands
with inspiration, Direct visualization while intubating, Esophageal detector, F, Graph
of carbon dioxide, Had an X ray, TEE,
Oesophageal intubation
Right main bronchus intubation
Then either

122
123 Oceanic Series End of Posting Compilation

1. Spontaneous respiration or
2. Controlled respiration by manual (bag squeezing) or mechanical ( ventilator)

About Pre oxygenation


1. Pre or before anaesthesia
2. Giving oxygen 100% via a tight fitting mask for 3-5 minutes, through a non-
rebreathing valve.
3. To fill up the FRC (2000 ml) of the patient with nearly 100% i.e. 98% oxygen. Also
called 98% denitrogenation.
4. Preventing the hypoxia during the 3-5 minute of apnoea following the introduction of
suxamethonium.

About Mallampati

About Cormack Lehane

Rapid sequence intubation


1. Rapidly done to prevent aspiration in a patient with full stomach.
2. Cricoid pressure posteriorly to occlude the oesophagus between it and the body of
cervical vertebra 6.
3. Only when the patient lose his consciousness to prevent the discomfort.

123
124 Oceanic Series End of Posting Compilation

4. Using the index finger in the center and the thumb and the middle fingers stabilizing
the trachea.
5. Quite similar to the BURP procedure on the larynx.
6. the correct position and the cuff inflated.

Awake blind nasotracheal intubation

Indications
1. Ensure airway patency
2. Protect airway from aspiration
Drugs in intubation
Asleep, Analgesic and Muscle relaxation
Suxamethonium
Depolarizing block, wear off by itself, Pseudo choline esterase, congenital absence,
Extubation

LEMON Look (chin, tongue), Evaluate 332, Mallampati, Obstruction, Neck

Cormack Lehane
1. Grade I: full view of the cords
2. Grade II: partial view of the cords
3. Grade III: view of the epiglottis
4. Grade IV: No view of the cords or epiglottis
Laryngoscope
1. Where will you put the tip of the blade of the laryngoscope finally?
2. First at Oral opening, Right or left side? Why?

124
125 Oceanic Series End of Posting Compilation

3. Which blade? Large or smaller


4. Curved or Straight and Why?
ET Tube
1. How much will you put in the endotracheal tube into the trachea?
2. The black line
3. How do you know it is in the trachea?
4. How will you know it is in the oesophagus?
5. Draw the capnographic tracing of oesophageal intubation?
6. Draw a normal capnographic tracing?
Magill’s forceps.
1. Other name
2. Reason of the curve.
Bag Valve Mask Ventilation Self inflating bag
1. What is it?
2. CE Technique
3. Head tilt chin lift
4. Jaw thrust
5. Gag reflex
6. Oro pharyngeal airway
7. Naso pharyngeal airway
Mouth to mouth. Kiss of life
1. Oxygen percentage in the expired air 15.3%
2. Carbon dioxide percentage in the expired air 4.5%
3. CO2 Good?

Indications
a. Cardiopulmonary Arrest
b. Patient in deep coma or unresponsive
c. Shallow or slow respirations (less than 8 per minute)
d. Progressive cyanosis
e. Gastric lavage / gavage
f. Surgical patients where body positioning or facial contours preclude the use of a mask
g. To prevent loss of airway at a later time, i.e. a burn patient who inhales hot gases may be
intubated initially to prevent his airway from swelling shut
Indications
1. Ensure airway patency

125
126 Oceanic Series End of Posting Compilation

Prolonged operations
Difficult access to airway
Excessive movement of head and neck
Difficulty achieving clear airway with laryngeal mask airway
Situations, major intraoperative complication (haemorrhage, malignant
hyperthermia)
3. Protect airway from aspiration
Risk of regurgitation
Extensive bleeding
Indications
1. Decreased consciousness and loss of airway reflexes
1. Failure to protect airway against aspiration - Decreased consciousness that
leads to regurgitation of vomit, secretions, or blood
2. Failure to maintain airway tone
1. Swelling of upper airway as in anaphylaxis or infection
2. Facial or neck trauma with oropharyngeal bleeding or hematoma
3. Failure to ventilate
1. End result of failure to maintain and protect airway
2. Prolonged respiratory effort that results in fatigue or failure, as in status
asthmaticus or severe COPD
4. Failure to oxygenate (ie, transport oxygen to pulmonary capillary blood)
1. End result of failure to maintain and protect airway or failure to ventilate
2. Diffuse pulmonary edema
3. Acute respiratory distress syndrome
4. Large pneumonia or air-space disease
5. Pulmonary embolism
6. Cyanide toxicity, carbon monoxide toxicity, methemoglobinemia
5. Anticipated clinical course or deterioration (eg, need for situation control, tests,
procedures)
1. Uncooperative trauma patient with life-threatening injuries who needs procedures
(eg, chest tube) or immediate CT scanning
2. Stab wound to neck with expanding hematoma
3. Septic shock with high minute-ventilation and poor peripheral perfusion
4. Intracranial hemorrhage with altered mental status and need for close blood
pressure control
5. Cervical spine fracture with concern for edema and loss of airway patency

126
127 Oceanic Series End of Posting Compilation

Indications for ENDOTRACHEAL INTUBATION in the operating room include:


1. the need to deliver positive pressure ventilation
2. protection of the respiratory tract from aspiration of gastric contents
3. surgical procedures involving the head and neck or in non-supine positions that
preclude manual airway support
4. almost all situations involving neuromuscular paralysis
5. surgical procedures involving the cranium, thorax, or abdomen
6. procedures that may involve intracranial hypertension
Some non-operative indications are:
1. profound disturbance in consciousness with the inability to protect the airway
2. tracheobronchial toilet
3. severe pulmonary or multisystem injury associated with respiratory failure, such as
sepsis, airway obstruction, hypoxemia, and hypercarbia
Objective measures may also be used to help determine the need for intubation:
1. respiratory rate > 35 breaths per minute
2. vital capacity < 15 ml/kg in adults and 10 ml/kg in children
3. inability to generate a negative inspiratory force of 20 mm Hg
4. PaO2 (arterial partial pressure of oxygen) < 70n mm Hg
5. A-a gradient (Alveolar-arterial) > 350 mm Hg on 100% oxygen
6. PaCO2 (arterial partial pressure of carbon dioxide) > 55 m Hg (except in chronic
retainers)
7. dead space > 0.6 L
Contraindications
1. Obstruction of the upper airway due to foreign objects
2. Cervical fractures
3. The following conditions require caution before attempting to intubate:
a. Esophageal disease
b. Ingestion of caustic substances
c. Mandibular fractures
d. Laryngeal edema
e. Thermal or chemical burns
Advantages
1. Provides an unobstructed airway when properly placed
2. Prevents aspiration of secretion(blood, mucous, stomach / bowel contents) into lungs
3. Can be easily maintained for a lengthy period of time
4. Decreases anatomic dead space by approximately 50%

127
128 Oceanic Series End of Posting Compilation

5. Facilitates positive pressure breathing without gastric inflation (compared with


BVMask)
6. Facilitates body positioning and movement of the patient
7. May be utilized to pass medications
a. Naloxone
b. Atropine
c. Epinephrine
d. Lidocaine
Disadvantages
Need advanced training to properly perform procedure
1. Bypasses the nares function of warming and filtering the air
2. Increased incidence of trauma due to neck manipulation, spinal cord injury suspected
3. May increase respiratory resistance
4. Improper placement
Pre oxygenation
1. Oxygen consumption in one minute
2. Carbon Dioxide production in one minute
3. What is respiratory arrest?
4. De nitrogenation percentage
5. Minutes
6. FRC
7. When will you remove the face mask?
Correct placement confirmation
1. Auscultation of breath sounds 5 point
2. Breathing associated haze/ fog
3. Chest m/m
4. Direct visualization
5. Esophageal detector
6. Feel of bag
7. Graph of CO2
Rapid sequence intubation
1. Cricoid Anatomy
2. Cricoid pressure
3. When to apply
4. When to stop
5. When to stop as an emergency
6. How it prevents What.

128
129 Oceanic Series End of Posting Compilation

7. Why is full stomach dangerous?


8. What is elective anaesthesia and what is emergency anaesthesia?
9. NPO ?
10. Mendelson’s or Acid aspiration syndrome
Complications
1. Oesophageal intubation How will you know?
2. Endobronchial intubation
3. Impaction
4. Herniation of the cuff
5. Compression of the lumen Stretching of the tracheal wall Fires
Position of head
1. Why do you usually put a pillow under the occiput before intubation?
2. Head extension where? Neck flexion How?
3. Why lift chin, in head tilt chin lift?
4. Jaw thrust
What will you see with the Laryngoscope?
1. What is the distance from the vocal cord to the entrance of right main bronchus?
2. How many centimetres (in detail) are there in one inch?
What is suxamethonium?
1. How long is the respiratory arrest produced by suxamethonium?
2. Why recovery does occur?
3. What is suxamethonium apnoea?
4. What will you do?
5. What is pseudo choline esterase?
Agonist Antagonist
1. What is an Agonist?
2. Suxamethonium Agonist or else
3. What is an Antagonist?
4. Is Atracurium an antagonist? If so how?
5. Antagonist for opioids, diazepam.
6. Affinity and intrinsic activity
Acetylcholine
1. Where is it secreted
2. Nicotinic receptors
3. Muscarinic receptors
4. Atropine
5. How it acts

129
130 Oceanic Series End of Posting Compilation

6. Where it acts? Receptors


Clinical methods for detection of recovery from paralysis
1. Sustained head lift
2. Good grip Handshake
ENDOTRACHEAL INTUBATION REQUIRED EQUIPMENT:
1. Gloves
2. Laryngoscope handle
3. Laryngoscope blade
4. Miller blade (straight blade)
5. Macintosh blade (curved blade)
6. Stylet – a wire inserted into the endotracheal tube in order to stiffen it during passage
7. Endotracheal tube
8. Size of tube is dependent on size of patient
9. 7.5 mm is the “Universally Accepted” size for an unknown victim
10. Men are usually larger, therefore an 8.0 mm tube may be appropriate
11. Females are usually smaller, therefore a 7.0 mm tube may be appropriate
12. Water soluble lubrication – KY Jelly
13. Magill forceps –used to help guide an endotracheal tube from pharynx into the larynx
14. 10 cc Syringe – used to fill the cuff at the end of the endotracheal tube
15. Ambu-bag – to facilitate positive pressure ventilations
16. Oropharyngeal airway (bite block), to prevent from biting down on endotracheal tube
17. Stethoscope – to check for proper placement of the endotracheal tube
18. Tape – to secure the endotracheal tube in place
19. Suction Device – to clear the airway of debris (blood, mucous, saliva)
PROCEDURAL STEPS:
1. A folded towel placed under the patients shoulders and neck to assist with
positioning
2. Head extended Position the patient’s head – three axes, those of the mouth, the
pharynx, and the trachea must be aligned to achieve direct visualization of the vocal
cords Sniffing Position – the head is extended and the neck is flexed
3. Explain & Suction the patient (no longer than 30 seconds)
4. Explain & Oxygenate patient for 3 minute with 100% Oxygen
5. Drugs
6. End of fasciculation
7. Laryngoscope handle is held with the left hand
8. Insert the curved laryngoscope blade in right side of mouth and sweep to center
of mouth

130
131 Oceanic Series End of Posting Compilation

9. When a straight blade is used, the tip of the blade is inserted under the epiglottis
10. Slide the laryngoscope blade to the Vallecula
11. Lift the laryngoscope blade in an upward motion 45 degree Along the handle
12. The handle must not be used with prying motion, upper teeth must not be used as
fulcrum
13. Visualize the vocal cord
14. Using the right hand, insert the endotracheal tube until you see the cuff pass through
the vocal cords. Advance the tube an additional ½ to 1 inch for proper placement.
15. Remove the laryngoscope carefully from the patients mouth
16. Remove the stylet from the endotracheal tube
17. Ventilate the patient with two breaths
18. Check for proper placement with these first two ventilation’s by:
a. Observing the chest rise and fall with each ventilation
b. Listen apex base axilla (Epigastrium)
c. Look Capnograph
d. Look Pulse oximeter
19. Inflate the endotracheal tube’s cuff with 10 cc’s of air:
a. Holds tube in place
b. Acts as a barrier and prevents fluids from entering the lungs
20. Ventilate the patient with two breaths
21. Insert oropharyngeal airway
22. Ventilate the patient with two breaths
23. Tape endotracheal tube securely in place Anchor
24. Continue to ventilate patient (1 breath every 5 seconds) and suction as necessary
a. NOTE: The insertion of the endotracheal tube should be no longer than 30 seconds
from the time you stop ventilating the patient until the time you remove the stylet. If you are
unable to place the endotracheal tube within 30 seconds, withdraw the endotracheal tube
and laryngoscope, ventilate the patient and start again
Extubation steps
1. Determine that endotracheal intubation is no longer required
2. Patient begins spontaneous respiration’s
3. Medical Officer orders removal of endotracheal tube
4. Remove tape from endotracheal tube
5. Remove oropharyngeal airway from patient’s mouth
6. Suction endotracheal tube, patient’s mouth, and patient’s posterior pharyngeal area
7. Deflate the endotracheal tube’s cuff
8. Withdraw the endotracheal tube with one smooth motion

131
132 Oceanic Series End of Posting Compilation

9. Monitor the patient for signs / symptoms of respiratory distress or difficulty


AWAKE BLIND NASAL
1. An awake patient
2. With conscious sedation and
3. Local anaesthesia to the airway or
4. Anaesthetised patient breathing spontaneously.
5. The head is positioned as for direct laryngoscopic intubation and
6. Endotracheal tube is gently passed through the nostril of choice reaches pharynx.
7. Then the chin is lifted forwards and the other nostril is occluded.
8. If the patient is awake he is asked to close his mouth and breathe deeply.
9. The tube is advanced slowly while listening for breath sounds
10. Capnography is extremely useful in this situation.
11. Breath sounds or a capnography trace, the tube has passed into the trachea.
12. Blind nasal intubation is a very useful it does not need expensive equipment and
13. Can be performed anywhere.
Awake
Indications
1. Upper airway obstruction
2. A known or suspected difficult intubation
3. Patient with an unstable cervical spine fracture where any traction on the neck should
be avoided
4. Full stomach. This technique is used in the United States of America.
5. Respiratory failure in extremis where anaesthetic induction may bring about the
patient’s immediate demise
The procedure for anaesthetising the airway
1. Give oxygen to the patient throughout the procedure eg.nasal prongs and ensure that all
routine monitoring is in place with intravenous access secured.
2. Administer a drying agent intravenously, for example atropine 400-600mcg or
glycopyrrolate 200-400 mcg.
3. Administer sedation to make the patient comfortable without compromising patient
safety. For example, a benzodiazepine eg.midazolam 0.5-2mg along with a short acting
opioid eg.fentanyl 50mcg. Although both drugs have specific antagonists available, care
should be taken not to depress respiration too much.
4. Local anaesthesia of the upper airway is achieved as follows:
1. Surface analgesia is achieved with 2-4% lignocaine (maximum 3mg/kg)
applied to the mouth, tongue, pharynx and nasal passages by spraying,

132
133 Oceanic Series End of Posting Compilation

gargling or inhaling a nebulised form. Cotton tipped pledgets soaked in the


same solution may be used for analgesia of the nasal passage.
2. A translaryngeal injection through the cricothyroid membrane provides
analgesia to the area below the vocal cords. To perform this injection, identify
the cricothyroid membrane and confirm that the tip of the needle is in right
place by free aspiration of air in a saline-filled syringe before the injection is
made (Figure 5). 2-4ml of lignocaine 4% is used because the higher
concentration penetrates the mucosa more efficiently. As soon as the
injection is performed the patient will cough and the needle should be
withdrawn swiftly to prevent any damage.
5. Have the equipment ready for the chosen technique eg.fibreoptic bronchoscope or
retrograde intubation set.
6. Plan for the procedure and have a rescue plan in case it fails.

How to intubate
Check equipment
Prior to positioning the patient:
1. Make sure that your laryngoscope is locked into position and that the incandescent
light on the blade tip functions. Also make sure that you have several alternate
blades available in case the one you have chosen does not allow for visualization of
the cords.
2. Examine the endotracheal tube. Make sure that the cuff inflates by using a 10-mL
syringe to inflate the cuff and then detach the syringe to ensure that the cuff pressure
is maintained. Be sure to deflate deflate the cuff after testing it.
3. Attach the connector to the proximal end of the tube. Push it in as far as possible to
lessen the likelihood of disconnection.
4. If you are going to use a stylet, it should be inserted into the ET tube and bent to
resemble a hockey stick to facilitate intubation of an anteriorly positioned
larynx. Even if you do not plan on using a stylet, one should be within easy access in
case the intubation proves to be more difficult than anticipated.
5. Ensure a functioning suction unit to clear the airway in case of unexpected blood,
emesis or secretions.
6. Ensure that you have tape within your reach to secure the tube once it is in place.
Proper patient positioning can be the difference between a successful and failed intubation.
1. The patient’s head should be level with the physician’s xiphoid process.
2. To achieve the sniff position (which allows for optimal visualization of the glottic
opening), elevate the patient’s head and extend the atlanto-occipital joint. This can

133
134 Oceanic Series End of Posting Compilation

be achieved by sliding your free hand (right hand if you are right handed, left hand if
you are left handed) beneath the patient’s head and gently lifting it up and towards
you. Or, you can gently position the chin up and mouth open before attemting
laryngoscopy.
3. The "scissor technique" can also be used to further open the patients mouth. Cross
your right forefinger and thumb and insert into the right side of the patient's
mouth. Apply pressure to the upper teeth with your forefinger and the lower teeth
with your thumb to open the mouth. Be sure to position your hands so as NOT to
obstruct your view.
Mask Ventilation is often used in the operating room after induction, prior to intubation. If
you are able to achieve signs of ventilation using this technique, you are afforded the
knowledge that, if intubation fails, you are able to achieve ventilation using the bag-mask-
valve device. Further, it allows for pre-oxygenation. Preparation for induction and
intubation in the operating room also involves pre-oxygenation with several (eight) deep
breaths of 100% oxygen. Preoxygenation provides an extra margin of safety in case the
patient is not easily ventilated after induction.
After preoxygenating the patient and positioning the patient in the Sniff position, with the
patient’s mouth widely open, carefully introduce the blade, held in your LEFT HAND, into the
right side of the mouth. Regardless of which blade is used, IT MUST NEVER PRESS
AGAINST THE TEETH or dental trauma will result. The tongue is then swept to the left and
up into the floor of the pharynx by the blade’s flange.
The curved Macintosh blade is inserted past the tongue into the vallecula (at the base of the
tongue). Providing sufficient lifting force in parallel with the handle, yet avoiding posterior
rotation that causes the blade to press against the teeth, pressure is applied deep in the
vallecular space by the tip of the blade immediately anterior to the epiglottis, which flips out
of the visual field to expose the laryngeal opening.
The straight Miller blade is inserted deep into the oropharynx, PAST the epiglottis. Providing
sufficient lifting force in parallel with the handle, yet avoiding posterior rotation that causes
the blade to press against the teeth, under direct vision, the blade is slowly withdrawn. It will
slip over the anterior larynx and come to a position at which it holds the epiglottis flat against
the tongue and anterior pharynx, exposing a view of the larynx.
With either blade, the handle is raised up and away from the patient in a plane perpendicular
to the patient’s mandible. Avoid trapping a lip between the teeth and the blade and AVOID
using the teeth as leverage and avoid posterior rotation of the blade.
Once a view of the larynx is obtained via laryngoscopy, the ETT is introduced with the
RIGHT HAND through the right side often mouth. Directly observe the tip of the tube
passing into the larynx, between the abducted cords. Pass the tube 1 cm through the

134
135 Oceanic Series End of Posting Compilation

cords. The ETT should lie in the upper trachea but beyond the larynx (3 to 4 cm proximal to
the carina). If the patient is going to be repositioned, the cuff should be closer to 2 cm
beyond the cords.
Remove the laryngoscope, careful not to displace the ET tube and not to cause trauma to
the teeth, lips or mucosa.
Inflate the cuff with the least amount of air necessary to create a seal during positive
pressure ventilation (usually 4-8 mL of air).
Remove the mask from the bag-valve device and attach the 15 mm connector on the
proximal end of the ET tube to the bag-valve device (into which oxygen is flowing and to
which the carbon dioxide detector is attached). Provide positive pressure and immediately
(and quickly):
• ausculatate the chest for breath sounds
• check the capnoraphic tracing on the monitor to ensure end tidal CO2
• check the connector for fog
• look at the chest for expansion with each breath
If there is any question as to whether the tube is in the esophagus or trachea, remove the
tube, ventilate with a mask and try again, this time attempting to adjust anything that may
have interfered with your first attempt. You might reposition the patient, use a different
blade, decrease tube size, or add a stylet.
If you are sure that your intubation is successful, turn on the mechanical
ventilator. Continuously provide positive-pressure ventilation at a volume of 350-700 ML per
70 kg (5-10 mL/Kg) and at a sufficient rate to maintain normal end tidal CO2 (8-12
respirations per minute).
Proceed to tape or tie the tube to secure its position. Do not tape or tie the cuff. To prevent
the patient from biting and occluding the ETT during emergence from anesthesia, a roll of
gauze can be placed between the teeth or an OPA can be inserted.
Document the view of the larynx obtained during laryngoscopy using the following criteria:
• Grade I: full view of the cords
• Grade II: partial view of the cords
• Grade III: view of the epiglottis
• Grade IV: No view of the cords or epiglottis
OTHER WAYS TO INTUBATE
Nasotracheal Intubation: Nasal intubation is similar to oral intubation except that the ETT is
advanced through the nose into the oropharynx before laryngoscopy. If the patient is awake,
local anesthetic drops and nerve blocks can be used. A lubricated ETT is introduced along
the floor of the nose, below the inferior nasal turbinate, perpendicular to the face. Often, a
nasopharyngeal airway can be used. The tube is advanced until it can be visualized in the

135
136 Oceanic Series End of Posting Compilation

oropharynx. Via laryngoscopy, the tube is then advanced in between the abducted vocal
cords.
Nasal instrumentation (with ETTs, NPOs, or nasal catheters) is contraindicated in all patients
with severe midfacial trauma.
Bougie: The Bougie is a straight, semi-rigid stylette-like device with a bent tip that can be
used when intubation is (or is predicted to be) difficult – often helpful when the tracheal
opening is anterior to the visual field. During laryngoscopy, the bougie is carefully advanced
into the larynx and through the cords until the tip enters a mainstem broncus. While
maintaining the laryngoscope and Bougie in position, an assistant threads an ETT over the
end of the bougie, into the larynx. Once the ETT is in place, the bougie is removed.
Light Wand: Lightwands, when inserted into an endotracheal tube, may be useful for blind
intubations of the trachea (when the laryngeal opening cannot be visualized). The end of
the ET tube is at the entrance of the trachea when light is well transilluminated through the
neck (the jack o’lantern effect). The tube can then be threaded off the light wand and into
the trachea in a blind fashion.
Flexible Fiberoptic Bronchoscopy: Laryngoscopy may be contraindicated in a patient who
requires intubation and mechanical ventilation. This is often the case in trauma patients who
may have an unstable cervical spine or in patients with poor range of motion of the temporo-
mandibular joint. In such patients, flexible fiberoptic bronchoscopy allows for indirect
visualization of the larynx. The endoscope is introduced through the mouth or nose. Once
anatomic structures are recognized, and the larynx or trachea are entered under direct
visualization.
COMPLICATIONS OF INTUBATION
Complications of laryngoscopy and intubation are most frequently secondary to airway
trauma, tube malpositioning, tube malfunction or physiologic responses to airway
instrumentation. Trauma such as tooth damage, lip/tongue/mucosal laceration, sore throat,
dislocated mandible, retropharyngeal dissection can occur during laryngoscopy and
intubation. Mucosal inflammation and ulceration and excoriation of nose can occur while the
tube is in place. Laryngeal malfunction and aspiration, glottic, subglottic or tracheal edema
and stenosis, vocal cord granuloma or paralysis during extubation.
Malpositioning of the endotracheal tube can result in esophageal intubation and
unintentional extubation.
Physiologic responses to intubation include hypertension, tachycardia, intracranial
hypertension, and laryngospasm. Laryngospasm, which occurs during induction and
recovery from anesthesia or, rarely, in an awake patient, is a forceful involuntary spasm of
the laryngeal musculature caused by sensory stimulation of the superior laryngeal

136
137 Oceanic Series End of Posting Compilation

nerve. Treatment includes positive pressure ventilation via a bag-mask device using 100%
oxygen or administration of IV lidocaine.

HOW TO EXTUBATE
Knowing when to extubate is also an important knowledge set. In general, it is best to
extubate when a patient is still deeply anesthetized (but with adequate spontaneous
respirations) or when the patient is awake and responsive with stable vital signs, good grip
and sustained head lift. Adequate reversal of neuromuscular blockade must be
established. A patient must also demonstrate adequate spontaneous respiratory function
with a vital capacity of greater than 15 mL/kg and a negative inspiratory force of greater than
20 mm Hg. Extubation while the patient is in a light plane of anesthesia or still emerging
from anesthesia is avoided because of an increased risk of laryngospasm, the most dreaded
complication of extubation.
Regardless of whether a patient is extubated while deeply anesthetized or awake, begin by
thoroughly suctioning the patient’s pharynx and mouth in order to decrease the risk of
aspiration or laryngospasm. Also, “preoxygenate” the patient with 100% oxygen in case it
becomes difficult to establish an airway after the ETT is removed. Untape the ETT and
deflate its cuff. Apply a small degree of positive pressure on the air bag to help blow out any
secretions you may have missed on first suctioning and suction again. Withdraw the tube
on end-inspiration or end-expiration in a single, smooth motion. Apply a face mask to
deliver 100% oxygen.
Intubation is easy by Dr Aung Myat Thwin 30-11-10
Yes, it is easy. Like every other things, if you know the basics you can do it. I will try to tell
them all, one by one.
So, are you ready to intubate? Do you know that intubation means Endotracheal Intubation?
Yes it is. So now you got the word. Let us go into detail. Endo means inside. Tracheal
means the trachea. Intubation means putting in the tube. So, now we know that
endotracheal intubation is just putting in a tube into the trachea.
We need something to help us in putting in the tube. To put in the tube, you must be able to
see the open end of the tube. The trachea is connected to the larynx at its upper end and
there is an opening in the larynx known as the laryngeal aperture or the glottis. If someone
wants to put in a tube into the trachea, he must be able see the larynx and to view the larynx
we need an instrument known as Laryngoscope or a viewer of the larynx. We need to know

137
138 Oceanic Series End of Posting Compilation

something about it. Then we also need to know a bit about a suitable tube to be introduced
into the trachea.
For someone to be able to see the laryngeal inlet from the mouth opening, we need to align
the axes of the oral cavity, pharyngeal tube and the larynx into a straight line. We must
position the head and neck of the patient to be able to do this.
So we now know that before intubating the patient we need to know
b. The Laryngo tracheal anatomy.
c. The positioning of head and neck of the patient,
d. The Laryngoscope and finally
e. The Endotracheal tube.
The Larynx and its associates
You can feel the larynx in front of your neck. It is also known as the Adam’s apple to the
laymen. The larynx has got three pained and three unpaired cartilages. The three unpaired
ones are the laryngeal, the cricoid and the epiglottis. The three paired cartilages are the
arytenoid, corniculate and the cunieform. Please look at the pictures below to get a little bit
of idea. The epiglottis originates from the back of the thyroid cartilage. The epiglottis and the
posterior end of the tongue meet at the valleculla. This is an important point.
At the inlet of the larynx, we can see the two vocal cords like an inverted V, just behind the
centrally lying epiglottis through which you can sometimes see the tracheal rings.
You must be able to feel the hyoid above, thyroid middle and the cricoid below in your own
neck.
Remember the side view of the epiglottis above, larynx middle and the cricoid below.
Flexion at the neck attenuates the tracheal axis towards the pharyngeal axis.
Extension of the head at the atlanto occipital joint attenuates the oral axis towards the
laryngeal axis.
Just remember the flexion of the neck and extension of the head at the atlanto occipital joint.
The Laryngoscope
Plastic, curved, bevelled, cuffed and marked PVC disposable tube.
The laryngeal aperture
The view of the laryngeal inlet is important.
The tip of the blade is in the vallecula.

138
139 Oceanic Series End of Posting Compilation

139
140 Oceanic Series End of Posting Compilation

140
141 Oceanic Series End of Posting Compilation

THORACOCENTESIS PROCEDURE
LUMBAR PUNCTURE OSCE

BASIC LIFE SUPPORT


30-year-old man collapsed in front of you and no previous contact with you (non-hospital
setting)

1. What would you do? Explain D-R-S-A-B-C-D

Patient had abnormal breathing.

2. What would you do next? Proceed to do chest compression.

3. What is the name of the device? Automated External Defibrillator

141
142 Oceanic Series End of Posting Compilation

4. Give 4 characteristic of chest compression.

- 5-6cm deep

- Rate 100-120 compression in 60 seconds

- Allow full chest recoil

- Minimum interruption

EPIDURAL NEEDLE

1. What is this needle called?

Epidural needle

2. Describe its feature

3. What are its other function besides anesthesia

4. How does medication is given via this needle

5.List all the layers that the needle must pass?

6. What are the other types of needles that can pass through the arachnoid layer?

Spinal needle

142
143 Oceanic Series End of Posting Compilation

st
Oceanic Question Bank 1 Edition
DERMATOLOGY

143
144 Oceanic Series End of Posting Compilation

EMQ
1. 35years old lady presented with recurrent crop of itchy vesicles Dyshidrotic
on both hands for 6 months. She works as account clerk, and Eczema
she was not known to have any allergy
2. A 63 years old lady presented with itchy scaling on face, neck, Photodermatitis
and extensor of both forearms for 3 weeks. She was just started
on Tab Hydrochlorothiazide by her GP for her hypertension two
months ago
3. 35 year old woman complained of multiple painful non blanching Cutaneous
papules over both lower limbs for 1 week. She had history of Vasculitis
taking Bactrim for URTI for a week
4. 60 year old lady underlying OA on analgesics complains of Fixed Drug
painful hyperpigmented patches over the forearm and back for Eruption
the past 6 months. She recalled of having similar 3 episodes of
the lesion at the same site in the past 6 months
5. 28-year-old woman presented with multiple painful erythematous Erythema
nodules on both shins for a week associated with fever and Nodosum
cough
6. 16 year old boy with intellectual disability and epileptic presented Adenoma
with asymptomatic skin coloured papules and nodules on his Sebaceum
face since his childhood
7. 50 y/o woman which is osteoarthritic and on analgesic Fixed Drug
complains of painful hyperpigmented skin lesion on forearm and Eruption
back, similar lesion at the same location have occurred 3 times
in the past 6 months
8. 33 year old man with underlying epilepsy presented with fever Drug
and generalised itchy skin rash on his body, face and limbs for Hypersensitivity
one week. Started on carbamazepine 2 months ago. On Syndrome
examination, he was febrile with generalised erythematous rash
involving 60% of body surface area No blisters or erosion on his
skin. Oral mucosa, eyes n genitalia were spared. Investigation
showed leucocytosis with eosinophilia, acute renal failure n
raised liver enzymes

144
145 Oceanic Series End of Posting Compilation

9. 33 years old Indonesian man presented with multiple Leprosy


asymptomatic erythematous patch over body n face. On
examination, there is reduced sensation to pin prick within skin
lesion
10. 7 y/o girl, patchy hair loss, scaly lesion on scalp. 3 cats at home Microsporum
Canis
11. a 6 years old child had recurrent itchy erythematous rash on Atopic Eczema
neck, antecubital and popliteal fossa, it was aggravated by
sweating and heat
12. A 15-year-old boy, studying in a boarding school, presented with Scabies
multiple itchy papules on both hands, feet, buttocks and
genitalia. The itch was severe at night
13. Painless lesion at penis Treponema
Pallidum
14. Woman with vaginal discharge. Gram stain shows diplococci Neisseria
organism- Gonorrhoea
15. Initially develop itchy papules on scalp, elbow. Examination Guttate
show scaly thick lesion with nail pitting Psoriasis
16. A woman complain of nodular erythematous tender lesion on Erythema
shin and feet after consumption of tablet (Diane) for Nodosum
contraceptive purpose
17. Child presented with soft, red compressible lesion on shoulder Infantile
Haemangioma
18. Non healing ulcer at cheek with rolled border Basal Cell
Carcinoma
19. 76 years old with dermatitis and Parkinson’s had multiple tensed Bullous
itchy lesions on upper limb and lower limb for 3 months Pemphigoid
20. 50 years old female underlying diabetes and gouty arthritis Tens
develop 40% epidermal detachment with erosion of oral mucous
and conjunctivitis
21. 22 years old footballer had multiple non itchy hypopigmentation, Pityriasis
well defined lesion on shoulder and back
22. erythematous involve more than 90% BSA with nail pitting AN Erythrodermic
Psoriasis
23. Bilateral itchiness purplish papule, polygonal shape and flat Lichen Planus

145
146 Oceanic Series End of Posting Compilation

24. Swelling at the right cheek, well demarcated margin, tender & Erysipelas
warm on palpation
25. 10 years old boy with a hairless patch. He has 3 cats at home. Tinea Capitis
26. 6 months old girl, presented with erythematous skin at perineal Candidiasis
region with multiple tiny vesicles. The beefy like lesion scattered (Diaper’s Rash)
along the line of the perineum
27. 50 y/o have face, neck and extensor rash after taking medication Photodermatitis
28. 69 years lady, itchy skin lesion on face, neck & extensors of both Photodermatitis
forearms. Recently started on hypertension medication.
29. 38 years old lady presented with multiple painful erythematous Erythema
nodules on shin of both legs. Associated with fever and cough Nodosum
for 1 week.
30. A 16 year old boy w intellectual disabilities and epilepsy. Ash Macule
Presented w asymptomatic well defined hypo pigmented lesion Leaf
on his back since childhood.
31. A man with multiple pigmented plaque on face. The plaque is Seborrheic
verrucous and has stuck on appearance. Keratosis
32. 60 years old man, develop non healing ulcer in his left cheek, Squamous Cell
gradually progressive for 9 month. Irregular everted border. Carcinoma
33. 33y/o Indonesian man comes with asymptomatic patch. Leprosy
Examination finds annular erythematous patch with reduce skin
prick sensation
34. Man with painful ulcer at penis. Notice vesicular erosion at penis Genital herpes
gland
35. 25 years old homosexual presented with generalized skin Secondary
erythematous rash at body, limbs, palms and sole for 1 week. syphilis
Has history of shallow ulcer on glans penis 2 months ago
36. A 11 months old baby with yellow greasy lesion on the head. Seborrheic
The lesion is not itchy. Otherwise, the baby is normal Dermatitis
37. 8 years old boy presented with eczematous skin rash just below Allergic Eczema
umbilicus. Response to steroids but recurred after stopping
steroids. The child likes to wear jeans
38. 7 years old presented with skin coloured papules, have central Mollucium
umbilication Contangiosum
39. Athlete presented with well demarcated hypopigmentation of Pityriasis
skin. On wood lamp, yellow green fluorescence Versicolor

146
147 Oceanic Series End of Posting Compilation

MEQ 1 psoriasis

1. 50 y/o gentleman with underlying DM and HPT, came to skin clinic with non-pruritus rash
over the trunk and legs.
a. Describe the lesion in the legs
Multiple erythematous, silvery scale plaque with well-defined margin, symmetrical
on both lower limbs
b. Describe 2 lesion of the nails
Nail pitting, onycholysis, subungual hyperkeratotic, total dystrophy
c. State the diagnosis
Plaque psoriasis
d. Give 1 example of potent topical steroid
Betamethasone valerate, mometasone furoate, fluticasone propionate
e. State 2 side effects of steroid
Acne, striae, skin thinning
f. State 2 other topical treatment
Emollient, tar preparation, dithranol in Vaseline, vitamin D 3 analogue

There is no improvement with the medication

g. State 2 other treatment option for the patient


Systemic therapy – methotrexate, systemic retinoid, cyclosporine
Phototherapy

147
148 Oceanic Series End of Posting Compilation

MEQ 2 psoriasis

2. 40 y/o gentleman, presented with multiple non itchy, scaly lesions on scalp and trunk,
associated with nail pitting for 2 months.

a) Describe the figure shown


Raised erythematous scaly papules with well defined margin
b) State the most likely d(x)
Guttate psoriasis
c) State 4 precipitating factors:
Trauma, infection, medication, pregnancy, stress, smoking
d) State 2 important complications:
CVS, hyperlipidemia, obesity, depression
e) State 4 topical agents
Emollient, tar preparation, dithranol in Vaseline, vitamin D 3 analogue

f) Patient not response to topical t(x). What other t(x) option?

Psoralent UVA, UVB

148
149 Oceanic Series End of Posting Compilation

MEQ 3 eczema

3. Pruritic rash at axilla for 2 weeks, hx of deodorant use for 1 month


a. State the diagnosis
Irritant eczema
b. Describe the pathophysiology of the disease

c. State the investigation that you want to do?


Patch testing test

MEQ 4 ECZEMA

4. Child with skin lesion over the scalp, upper and lower extremities, and popliteal fossa for
2 years.
a. State 2 history you want to ask
History of bronchial asthma, allergic rhinitis
Family history of bronchial asthma, allergic rhinitis, atopic eczema
Triggering factor
b. Describe lesion (4 marks)

Erythematous, pruritic, ill defined, crusted lesion


c. State 2 exacerbating factors
Extreme weather and allergens such as pollen, dust
d. State the Final diagnosis
Atopic eczema
e. State 2 managements
According to the intensity of the disease
Step 1: skin hydration, emollient, avoidance irritants
Step 2: low mid potent topical corticosteroid or topical calcineurin inhibitors
Step3: mid-high potent topical corticosteroid or topical calcineurin inhibitors
Step 4: systemic therapy (cyclosporine) or UV therapy

149
150 Oceanic Series End of Posting Compilation

MEQ 5 ECZEMA

5. Mariam, 11 year old, hx of itchiness at the neck for 12 months. Use glucocorticoid itchiness
recover but recur after that. Been wearing tudung for the past 9 months. No other skin
disease or atopy disease.

a. Describe the lesion.


Erythematous, scaly patch with well-defined margin.
b. State your diagnosis
Allergic contact dermatitis
c. State the possible allergen
Nickel
d. State 2 other object made up from nickel
Necklace, belt,
e. State investigation to confirm
Allergic test (patch test)
f. State 2 topical treatment
Calcineurin inhibitor, emollient, steroid
g. Other thing that she need to do apart from taking medication?
Avoid using things associated with nickel for the time being.

150
151 Oceanic Series End of Posting Compilation

MEQ 6 FUNGAL SKIN INFECTION


6. The differences between Pityriasis Versicolor and Pityriasis Alba

a. State the characteristics of each disease

Pityriasis Versicolor: hyperpigmented or hypopigmented finely scaled macules

Pityriasis Alba: first characterized by red, scaly patches. These patches resolve
leaving areas of scaling hypopigmentation, or lighter coloration.

b. State the causative organism

Pityriasis Versicolor: Malassezia furfur

Pityriasis Alba: unknown

c. Explain how to take the sample for both infection

Skin scraping

d. After taking the sample and stained it, what appearance that you would expect to
see?

Spaghetti and meatballs

151
152 Oceanic Series End of Posting Compilation

MEQ 7 TENS

7. The scenario is about Toxic epidermal necrolysis vs SJS


a. The patient was prescribed with medication however the condition deteriorates
over the weeks, state what is happening to the patient
Drug hypersensitivity syndrome
b. State the symptoms of TEN
Involve more than 30% of BSA
Epidermal detachment
Extensive confluent lesions
c. State the complication of TEN
Barrier function: Dehydration. Electrolyte imbalance, protein loss
Temperature regulation: Hypothermia and hyperthermia
Immune function: Increase risk of infection
Metabolic and endocrine function: Hypercatabolic and hyperglycemia

MEQ 8 DERMATOMYOSITIS
8. The scenario is about dermatomyositis

Figure 1

Figure 2

152
153 Oceanic Series End of Posting Compilation

a. What is the diagnosis based on picture given?


Dermatomyositis
b. State the signs for each picture
Picture 1: Gottron sign
Picture 2: Heliotrope sign
c. State 2 investigations that can be done
Serum muscle enzyme
MRI
Electromyography
Muscle and skin biopsy

153
154 Oceanic Series End of Posting Compilation

Oceanic Question Bank 1st Edition


EAR, NOSE & THROAT

154
155 Oceanic Series End of Posting Compilation

EMQ

102. 72-years-old woman presented with left-sided ear pain, Herpes Zoster
followed by weakness of face including eye closure. Oticus (Ramsay
Vesicles are evident on pinna. Diagnosis? Hunt syndrome

103. 60-years-old diabetic patient presented with 4-weeks history Necrotizing


of left-sided earache. Pain radiates down his jaw and keeps (malignant) otitis
him awake at night. externa

104. 20 years old man presented with 3 days worsening of sore Peritonsillar
throat and fever. He complained of otalgia, difficulty opening abscess
mouth and change in speech. On examination, he has
trismus, tonsil swelling and soft palate inflammation on the
right side. The uvula is deviated to the left. Diagnosis?
105. 50 years old man having dysphagia and hoarseness of Squamous cell
voice. He smokes cigarette 20 sticks per day. Diagnosis? carcinoma of
tonsil
106. Theme: Treatment of Otological Sepsis Mastoidectomy
Atticoantral perforation and cholesteatoma. What is the
definitive treatment?
107. Theme: Treatment of Otological Sepsis Analgesic with
12 years old boy presented with pyrexia, systemic upset systemic
and painful ear by frequent pulling ear pinna. Treatment? antibiotic
108. 50-year-old man presented with nasal blockage 2 month Inferior turbinate
associated with nasal discharge and itchiness in both nasal hypertrophy
cavities. On nasoscopy, there was boggy oedematous red
swelling. Patient is sensitive to nasal probing on these
swelling. Most likely diagnosis of this case?
109. 40 years old man, right sided nasal blockage and discharge. Inverted
On examination, he was noted to have irregular friable papilloma
mass. Diagnosis?
110. Theme: Treatment of Otological Sepsis Analgesics with
6-year-old boy presented with ear pain, pyrexia, systemic systemic
upset and a red ear drum. Treatment? antibiotic

155
156 Oceanic Series End of Posting Compilation

111. Theme: Treatment of Otological Sepsis Mastoidectomy


50-year-old man presented with cholesteatoma and
persistent foul-smelling suppurative ear discharge.
Treatment?
112. Theme: Paediatric Airway Obstruction Acute laryngo-
4-year-old girl presented with a 24-hour history of general tracheo-broncitis
malaise and has developed a barking, noisy cough with
difficulty in breathing. There was no drooling of saliva.
Diagnosis?
113. Theme: Paediatric Airway Obstruction Foreign body
2-year-old girl presented with a brief history of chocking and airway
cyanosis for about 10 seconds a few hours earlier. She is
now completely asymptomatic.
114. 45-year-old woman presents with a two-month history of Vocal cord
hoarseness of voice. She has history of cerebral granuloma
haemorrhage due to motor vehicle accident requiring
admission to ICU, intubation and craniotomy. Diagnosis?
115. 40-year-old lady presented with reduced hearing. She gave Sensorineural
history of high-grade fever one month ago. On examination, hearing loss in
Rinne’s test is positive on both ears and Weber test is both ears (Left
lateralized to right ear. Absolute bone conduction is reduced more severe than
on both sides. This is the most likely diagnosis of this case. right)
116. Theme: Hearing loss Cholesteatoma
65-year-old lady presented with a history of chronic ear
infection. She complained of an offensive discharge from
her right ear with associated hearing loss for months.
Examination of the ear revealed an attic perforation of ear
drum with abnormal skin collection. Name of that finding?
117. Theme: Hearing loss Otosclerosis
35-year-old man presented with progressive bilateral
hearing loss over the last year. Rinne’s test was negative on
both sides and on examination the tympanic membrane was

156
157 Oceanic Series End of Posting Compilation

normal. His father lost his hearing at a similar age and


required a hearing aid. Diagnosis?
118. Theme: Hoarse voice Vocal cord
45 years old woman presented with a hoarse voice. She granuloma
had a history of drug overuse and requiring admission to
intensive care unit and intubation. Diagnosis?
119. Theme: Hoarse voice Vocal cord palsy
51-year-old woman complained of a hoarse voice following
hemithyroidectomy. Diagnosis?
120. A two-month-old baby presented with gradually worsening Laryngomalacia
noisy breathing which is especially noticeable when she
eats. She is on a lower centile for weight gain and has poor
food intake. This is the most likely diagnosis.
121. A 40-year-old man presented with a history of bilateral nasal Rhinitis
blockage. He has temporary relied with an over-the-counter medicamentosa
decongestant that he has been using for the last two
months. This is the most likely diagnosis.
122. 3-year-old boy presented with 1 week history of foul Foreign body
smelling and unilateral nasal discharge. Diagnosis?
123. 40 years old asthmatic patient presented with gradual nasal Nasal polyp
blockage and pale swelling in both nasal cavities. He is
known to be sensitive to aspirin.
124. 3-year-old boy presents with sudden onset noisy breathing Acute epiglottitis
and drooling. He had a preceding URTI and pyrexia. On
examination, the child appears frightened, flushed in the
face, with inspiratory and expiratory stridor. There is in-
drawing of neck muscles. Diagnosis?
125. 3-year-old girl with history of 2 months hoarseness presents Laryngeal
with worsening stridor over the last 72 hours. The child is papillomatosis
afebrile. She had similar admission for 3 times this year for
similar problem. Diagnosis?
126. 12-years-old boy complained of hearing loss for 1 month. Left conductive
Rinne’s test right positive, left negative. Weber lateralized to hearing loss

157
158 Oceanic Series End of Posting Compilation

left side. Absolute bone conduction test was equal on both


ears. Diagnosis?
127. 40-year-old woman with history of fever and reduced Sensorineural
hearing. Rinne test was positive on both ears. Weber test hearing loss.
lateralized to left. Absolute bone conduction test reduced on Right side is
both sides. Diagnosis? severe
128. 65-year-old man presented with reduced hearing for 5 years Right
duration. Rinne test was positive on both ears. Weber test sensorineural
was lateralized to the left. Absolute bone conduction was hearing loss
reduced on right side and equal on left side
129. 24 y/o woman, 3-day hx of increasing sore throat, general Acute tonsillitis
malaise and temperature. Examination show inflamed
enlarged tonsil with white spots with bilateral
lymphadenopathy. FBC shows neutrophil leucocytosis. Paul
Bunnel test negative. Diagnosis?
130. Child presented with pulling of ear, systemic upset. Analgesic and
Treatment? systemic
antibiotic
131. Theme: Investigation of sinus Non contrast CT
35 years old lady, simple nasal polyp, not resolve with scan
medical treatment. What preoperative assessment to
perform?
132. Theme: Investigation of sinus Biopsy under
25 years old present with history of unilateral nasal bleeding nasal endoscopy
and exophytic mass at fossa of Rosenmuller. What is the
definitive investigation?
133. Theme: Diagnosis of voice disorder Vocal cord palsy
Young women present with decrease in voice quality after
hemithyroidectomy?
134. Theme: Diagnosis of voice disorder Chronic
Old man hoarseness of voice, taking antacids for laryngitis.
indigestion. History of smoking for years.

158
159 Oceanic Series End of Posting Compilation

135. Theme: Diagnosis of voice disorder Laryngeal


24 y/o woman presents with inspiratory stridor, dyspnoea papillomatosis
and hoarseness. On direct laryngoscopy, extensive, pink-
coloured warty growths were seen in laryngeal inlet.
136. 53 y/o wood worker, left side nasal blockage, blood Adenocarcinoma
discharge, pain, paraesthesia of cheek. Diagnosis? of maxillary
sinus
137. 32 y/o bilateral facial pain with mucous discharge. Sinusitis
Diagnosis?
138. 5 y/o with runny nose, snoring, and obstructive sleep apnea. Adenoid
Diagnosis? hypertrophy
139. 80 y/o woman, with ill-fitting denture wakes up in the middle Foreign body
of the night with sudden onset of pain in the neck and throat
difficulty swallowing her saliva. No symptoms prior to
episode. Diagnosis?
140. 75 y/o woman presents with three months history of Hypopharyngeal
increasing dysphagia to solid, weight loss and pain on Carcinoma
swallowing. Diagnosis?
141. Tragus sign positive. Pinna oedema, canal filled with debris. Acute otitis
Diagnosis? externa
142. 15 y/o boy presents for the fourth time to casualty with Local anaesthetic
minor bleeding from Little’s Area. Attempts at direct spray and
pressure to arrest the bleeding prove unsuccessful. An chemical/
obvious bleeding point can easily be identified. electrical cautery
143. 50 y/o woman presents to casualty with a 6-day history of Emergency
ongoing bleeding from both nostrils. She tells you that she examination
has a history of Osler Weber Rendu syndrome. Although under
the casualty has packed the nose with a Merocel (nasal anaesthesia
tampon), the patient is continuing to bleed. Examination (EUA) and
reveals brisk bleeding from both nostrils and a significantly packing in
deviated nasal septum. The patient has a haemoglobin of theatre +/-
7.3 g/dl. submucous
resection (SMR)

159
160 Oceanic Series End of Posting Compilation

144. 30 y/o man presents with a 2-day history of severe right Acute otitis
sided earache after a recent holiday abroad. On externa
examination, there is marked tragal tenderness. The ear
canal is swollen and filled with debris
145. 5 y/o boy with an URTI had a 24-hour history of severe pain Acute
in his right ear followed by a pus like discharge with suppurative otitis
resolution of pain media
146. 75 y/o woman presents with a 3-month history of increasing Hypopharyngeal
dysphagia to solids, pain on swallowing and weight loss carcinoma
147. 70 y/o man present with a 5-week history of right sided sore Squamous cell
throat and worsening pain on swallowing. He has carcinoma of
associated otalgia. He smokes 20 cigarettes every day and tongue
is known to be a heavy drinker
148. 3 y/o boy presents to outpatient clinic with a history of Hearing test and
delayed speech development. Examination reveals a tympanoplasty
lustreless tympanic membrane with several visible
“bubbles”. Management?
149. 29 y/o man post radical mastoidectomy presents with a Microsuction and
three-month history of discharge from ear. Examination topical
reveals a chronic perforation, although his original surgery antibiotics
preserved the tympanic membrane.
150. 75 y/o woman presents with a 3-month history of increasing Hypopharyngeal
dysphagia to solids, pain on swallowing and weight loss carcinoma
151. 30 y/o man presents with a 2-day history of severe right Acute otitis
sided earache after a recent holiday abroad. On externa
examination, there is marked tragal tenderness. The ear
canal is swollen and filled with debris
152. 5 y/o boy with an URTI had a 24-hour history of severe pain Acute
in his right ear followed by a pus like discharge with suppurative otitis
resolution of pain media
153. A 20-year-old man presents with a 3-day history of a Peritonsillar
worsening sore throat and raised temperature. He is abscess
complaining of right otalgia, difficulty opening his mouth

160
161 Oceanic Series End of Posting Compilation

and a change in his speech. On examination he has trismus


(difficulty opening mouth), tonsil inflammation and
soft palate swelling on the right side. The uvula is deviated
to the left.
154. 18-year-old woman presents with a 7-day history of general Glandular fever
malaise and increasing throat pain, causing her difficulty in
eating and drinking. On examination the tonsils are
enlarged and inflamed with a sloughy exudate on the
surface. There is marked cervical lymphadenopathy. Full
blood count shows a lymphocytosis. Paul Bunnel test is
positive.
155. 35 y/o woman presents with a 3-month history of a Globus
sensation of a lump in the throat. There is no dysphagia, pharyngeus
and the sensation improves when she swallows.
156. 48-year-old man presents with a 2-month history of hoarse Chronic
voice. He takes oral antacid therapy for indigestion and laryngitis
denies any dysphagia.
157. 40-year-old known asthmatic patient presents with gradual Deviated nasal
blockage of both nostrils. The GP examined the nose and bone
noted pale swellings in both nasal cavities. The patient is
known to be sensitive to aspirin.
158. 2-year-old child presents with a 1-week history of an Foreign body
offensive unilateral nasal discharge. nose
159. Following are the risk factors for laryngeal carcinoma except Epstein-Barr
virus
160. All the following are early complications of a Tracheo-
tracheostomy EXCEPT oesophageal
fistula
161. Commonest site for lodgement of oesophageal foreign body Just below the
is cricopharyngeus
162. Which of the following is not a site of normal esophageal Above the
constriction? diaphragm
163. Flat tympanogram is found in Otitis media with

161
162 Oceanic Series End of Posting Compilation

effusion
164. All the following are causes of a conductive hearing loss Meniere’s
EXCEPT disease
165. Commonest site of impaction of FB in the ear is At the junction of
the
cartilaginous part
and the bony part
of EAM.
166. Easiest method of removal FB from EAM is Irrigation by
water
167. Which FB does not need urgent removal? Plastic bead.
168. The ethmoid sinuses drain into Middle meatus
and superior
meatus
169. All the following arteries share in Keisselbach's plexus Posterior
except ethmoidal artery
170. The following paranasal sinuses drain into the ostiomeatal Posterior
complex except ethmoidal
sinuses
171. The nasolacrimal duct opens into Inferior meatus
172. The teeth related to the floor of the maxillary sinus are Second premolar
and first and
second molar
173. The sphenoid sinus drains into Spheno-
ethmoidal recess
174. The visible structures by anterior rhinoscopy are the Cribriform plate
following except of ethmoid
175. A 4-year-old child presented with left offensive nasal Foreign body
discharge. You should suspect impaction
176. Which of the following nasal foreign body irritating and Bean
cause inflammatory reaction
177. Of the nasal foreign bodies one of the following may cause Calculator
nasal septal perforation battery

162
163 Oceanic Series End of Posting Compilation

178. A patient presented with bilateral nasal obstruction after Septal abscess
nasal trauma. The patient temperature is 38. There is
throbbing nasal pain. Your diagnosis is
179. 25 y/o patient with fever, mucopurulent nasal discharge and Acute maxillary
pain over the cheeks is suffering from sinusitis
180. 40 y/o male presented with left nasal obstruction and fleshy Inverted
reddish nasal mass. There is a history of recurrence after papilloma
previous surgery 2 years ago. Diagnosis?
181. Juvenile nasopharyngeal angiofibroma is characterized by Affects only
the following except females
182. The following are causes of unilateral offensive nasal Nasal allergy
discharge except
183. Keisselbach's plexus accounts for the following percentage 90%
of epistaxis
184. A patient with epistaxis showing a bleeding point in Little's Cautery
area is best managed by
185. In a teenager male with recurrent severe left epistaxis, Nasopharyngeal
pallor and conductive deafness of the left ear, you should angiofibroma
suspect
186. Which of the following structures does not pass through the Chorda tympani
internal auditory meatus nerve
187. Retracted ear drum is characterized by all the following Central ear drum
except perforation
188. The cricothyroid muscles has its nerve supply from External
laryngeal nerve
189. The commonest site of laryngeal carcinoma is Glottis
190. Hoarseness is an early symptom in Glottis
carcinoma
191. Dyspnea is an early symptom in Subglottic
carcinoma

163
164 Oceanic Series End of Posting Compilation

MEQ

QUESTION 1 (ACUTE OTITIS EXTERNA)

A 25-year-old man comes to accident and emergency department (A&E), with complaints of
severe left sided ear pain for five days duration and reduced hearing on the left side.

Upon further questioning, he had habit of using cotton bud for ear cleaning and complaints of
left pinna tenderness whenever he accidentally touches his left ear.

1. Mention four complaints you would ask to reach your diagnosis:

• Ear discharge • Vertigo


• Ear fullness or pressure • Itching (ear)*** classic of AOE
• Tinnitus • Fever
2. Expected physical findings:

• Narrowing of external auditory • Impacted wax in EAC


canal (EAC) *** • Injury to ear drum or perforation
• Erythema and oedema of skin • Cellulitis of the face or neck
lining EAC*** • Lymphadenopathy of the
• Clear/ purulent and foul-smelling ipsilateral neck
ear discharge • Tragal sign-positive***
• Fungus in EAC
3. Differential diagnoses:

• Otomycosis • Ear canal trauma


• Furunculosis • Foreign body in EAC
• Acute otitis media • Ramsay Hunt Syndrome
• Acute myringitis • Acute otitis externa
4. Name the procedure (physical examination) to confirm your diagnosis:

Name of procedure-Otoscopy

5. State your final diagnosis:

Left acute otitis externa

164
165 Oceanic Series End of Posting Compilation

6. Possible complications:

• Abscess formation • Cellulitis


• Narrowing of ear canal • Malignant otitis externa
• Inflamed or perforated ear drum
7. Mention four principles of management:

• Ear toileting/ frequent canal cleaning: Dry mopping, irrigation, ear suction
• Topical antibiotics (control oedema, infection)
• Anaelgesics (relieve pain)
• Prevention by health education (don’t la too frequent use ear cotton bud!)

165
166 Oceanic Series End of Posting Compilation

QUESTION 2 (OTITIS MEDIA EFFUSION)

Nora is a 6-year-old girl who was brought to the audiology clinic because she has been having
academic trouble in school. According to her classroom teacher, Nora has difficulty in following
instructions. She appears to stare blankly when the teacher is speaking to the class and never
answers questions. She had previous history of frequent attacks of URTI.

Note: common presentation of otitis media effusion: school going age, blockage sensation in
ears, poor school performance, reduce hearing, ear pain, ear discharge, tinnitus, vertigo, fever,
h/o of URTI (important), h/o of allergic rhinitis (important).

1. Symptoms you would ask to reach your diagnosis:

• Ear Pain • Vertigo


• Ear Discharge/ Fullness • Hearing Loss
• Tinnitus • Fever
2. Expected Physical / Otoscopic/ Tuning Fork Findings

• Retracted ear drum • Conductive hearing loss


• Yellow/ Dark orange/ grey • Foreign body in EAC: TRO ear
coloured ear drum obstruction
• Air-fluid level, bubbles behind • Ear drum perforation: TRO
intact ear drum Chronic Suppurative OM
• Handle malleus shortened
3. Differential diagnoses:

• Otitis media with effusion • Adenoid hypertrophy


• Adhesive otitis media • Eustachian tube dysfunction
• Tympanosclerosis • Nasopharyngeal carcinoma
4. Name the procedure (physical examination) to confirm your diagnosis: Otoscopy

5. Investigations:

• Examination under microscope • Tympanometry


(EUM) • Language testing
• Pneumatic otoscopy • Tympanocentasis
• Pure tone audiometry (PTA)

166
167 Oceanic Series End of Posting Compilation

6. State your final diagnosis:

Right/ left (kalau tak mention in scenario, takyah tulis), Acute (<6 weeks) / chronic (>12
weeks) (kalau tak mention in scenario, takyah tulis) Otitis Media Effusion

7. Possible complications:

• Delayed speech and language • Retraction pocket of ear drum


development • Cholesteatoma
• Poor academic performance • CEOM/ CSOM
• Tympanosclerosis
8. Treatment:

• Treat underlying cause


• Nasal decongestant
• If persist of fluid + hearing loss: Myringotomy + Gromet insertion

167
168 Oceanic Series End of Posting Compilation

QUESTION 3 (SAFE/TUBOTYMPANIC CHRONIC SUPPURATIVE OTITIS MEDIA)

A 30-year-old gentleman presented with history of discharge from right ear with hearing loss for
five years.

1. Symptoms you would ask to reach your diagnosis:

• Ear discharge • Tinnitus


• Hearing loss • Vertigo
• No pain • Fever: TRO infection
2. Expected physical findings:

• Retracted ear drum • Perforation ear drum: Central


• Discharge in external auditory • Mild/ Moderate Conductive
canal: Profuse, Mucoid, Hearing loss
Odourless • No cholesteatoma
3. Differential diagnoses:

• Unsafe/ attichoantral CSOM • Chronic Effusion Otitis Media


4. Name the procedure (physical examination) to confirm your diagnosis: Otoscopy

5. Investigation:

• Swab for C+S • Tympanometry


• Pure tone audiometry (PTA) • Imaging CT/MRI
State your final diagnosis: Right Safe/Tubotympanic Chronic Suppurative Otitis Media

6. Treatment:

Medical (interval surveillance for 3 months)

• Ear toileting/ frequent canal cleaning: Dry mopping, irrigation, ear suction
• Topical antibiotics +/- corticosteroid
• Analgesics (if needed)
• Prevention by health education
Surgery

• Mastoidectomy+ Myringoplasty+Meatoplasty (if indicated)

168
169 Oceanic Series End of Posting Compilation

QUESTION 4 (UNSAFE/ATTICHOANTRAL CHRONIC SUPPURATIVE OTITIS MEDIA)

A 25-year-old man presents with right sided foul smelling ear discharge for 3 years duration
associated with severe headache, neck rigidity and vomiting of 4 days duration.

1. Symptoms you would ask to reach your diagnosis:

• Ear discharge • Tinnitus


• Hearing loss • Vertigo
• Pain • Fever
2. Expected physical findings:

• Retracted ear drum • Perforation ear drum: Attic/


• Discharge in external auditory Marginal
canal: Scanty, Purulent, Foul • Conductive Hearing loss/ Mixed
Smelling • Have cholesteatoma
3. Differential diagnoses:

• Unsafe/ attichoantral CSOM


• Chronic Effusion Otitis Media
4. Name the procedure (physical examination) to confirm your diagnosis: Otoscopy

5. Investigation:

• Swab for C+S • Tympanometry


• Pure tone audiometry (PTA) • Imaging CT/MRI
State your final diagnosis:

Right Unsafe/ Attichoantral Chronic Suppurative Otitis Media (with complication;


Meningitis)

6. Possible complications:

• Meningitis • Facial Nerve Plasy


• Labyrinthitis • Petrositis
• Mastoiditis

169
170 Oceanic Series End of Posting Compilation

7. Treatment:

Medical (interval surveillance for 3 months)

• Ear toileting/ frequent canal cleaning: Dry mopping, irrigation, ear suction
• Topical antibiotics +/- corticosteroid
• Anaelgesics (if needed)
• Prevention by health education
Surgery

• Mastoidectomy+ Myringoplasty+Meatoplasty (if indicated)

170
171 Oceanic Series End of Posting Compilation

QUESTION 5 (NASAL POLYPS)

A 50-year-old male patient came to hospital with gradual bilateral nasal blockage for 3 weeks
and history of sneezing nasal discharge for 12 years. On examination, pale swellings were
found in both nasal cavities.

Note: nasal polyp pathophysiology: chronic sinusitis/ allergic rhinitis/ asthma→ chronic
inflammation→ fluid builds up in mucosal cells→ over time, gravity pulls on fluid filled cells→
forming Nasal Polyps!!

1. Symptoms you would ask to reach your diagnosis:

• Runny nose • Decrease sense of smell/ taste


• Mouth breathing • Nasal bleeding
• Persistent stuffiness • H/o allergy
• Facial discomfort • H/o asthma
• Snoring • H/o aspirin used
2. Expected physical Findings

Inspection: normal.

Airway Patency test: Nasal blockage

Anterior Rhinoscopy:

• Unilateral/ Bilateral • Pale in colour


• Smooth, glistening • Not sensitive to probing
• Grape-like masses • Not bleed to touch
3. Differential diagnoses:

• Nasal Foreign Body


• Nasopharyngeal Carcinoma (NPC)
4. Investigations:

• Nasal Endoscopy
• Coronal CT scan

5. State your final diagnosis:

171
172 Oceanic Series End of Posting Compilation

• Bilateral nasal polyps (or Bilateral Eithmoidal Nasal Polyps)


Note: Eithmoidal polyps: Bilateral, grape-like, shiny glistening appearance, grow
gradually, a/w asthma, asprin

Note: Antrochoanal polyp: Unilateral, trilobed shape

6. Definitive Treatment:

• Steroids (primary main treatment)


• Polypectomy (with indication)
7. Possible Complications:

• Obstructive sleep apnoea (OSA)


• Sinusitis
• Middle ear infection
• Altered facial structures

172
173 Oceanic Series End of Posting Compilation

QUESTION 6 (NASOPHARYNGEAL CARCINOMA (NPC))

A 50-year-old Chinese gentleman came to hospital with right sided blood-stained nasal
discharge, painless enlarged lymph node on his right neck and pain in right ear off and on for
two months.

1. List four other relevant symptoms to achieve your diagnosis:

Nasal symptoms: Neuro symptoms:

• Fresh bleeding from the nose • Headache


• Nasal obstruction • Diplopia (CN4)
• Hyponasal speech (sengau) • Facial Pain (CN5)
Aural symptoms: Neck:

• Conductive Hearing loss • Mass in neck area


• Tinnitus • Lymph node swelling
• Otalgia/ pain
2. List four clinical signs you would elicit to help in diagnosis:

• Cranial nerve dysfunction, most commonly of VI / III, IV, VI/ V/ IX-XII


• Persistent middle ear effusion
• Retracted TM
• Tuning fork finding: conductive hearing loss from the effected side
• Exophytic mass arising from roof/Rossen Muller fossa of nasopharynx
3. State your clinical diagnosis:

Nasopharyngeal carcinoma

4. Differential diagnoses:

• Nasopharyngeal lymphoma • Rhabdomyosarcoma


• Hodgkin's disease • Hemangioma
• Non-Hodgkin's lymphoma

173
174 Oceanic Series End of Posting Compilation

5. Mention two specific investigations with one finding each to establish diagnosis:

• Naso-endoscopy - abnormal growth at nasopharynx (Fossa of Rossenmuller, roof


of nasopharynx)
• Biopsy/ FNAC of neck swelling - Squamous cell carcinoma
• Imaging- CT/MRI - Isodense/isointense shadow in nasopharynx (to know
extension of lesion)
6. List four treatment options according to your diagnosis.

Treatment options: Radiotherapy alone/Radiotherapy & chemotherapy/Radiotherapy &


chemotherapy plus neck dissection/Palliative care/Radio/ radio-chemo or surgery
(nasopharyngectomy +/- neck dissection)

Treatment:

• Early stage: Radiotherapy alone


• Advanced: Radiotherapy + Chemo
• Metastasis/ Recurrent disease: Radiotherapy + Chemo + Surgery
(nasopharyngectomy +/- neck dissection)
• Recurrence after radiotherapy within 1 year: Surgery (nasopharyngectomy +/-
neck dissection)
7. Pathology/ WHO classification

• Class I: Keratinized Sq. Cell Ca. (Poor prognosis)


• Class II: Non- Keratinized Sq. Cell Ca.
• Class III: Undifferentiated (Good prognosis)
8. Prognosis: (poor to good → Class I to III)

• Poor→ Class I (Keratinized SCC) because insensitive to radiotherapy


• Good→ Class III (undifferentiated) bcoz radiosensitive, better control BUT have
higher rate distant metastasis.

174
175 Oceanic Series End of Posting Compilation

QUESTION 7 (ALLERGIC RHINITIS)

A 25-year-old woman came to clinic with history of frequent attacks of bilateral nasal blockage,
nasal congestion, itchiness, and watery eyes whenever she goes to dusty and crowded area.

1. Symptoms you would ask to reach your diagnosis:

• Eyes: Watery, itchy, puffy


• Nose: Runny nose, nasal congestion, Frequent sneezing, Postnasal drip • Throat/
roof of mouth: Itchy, sore throat
• Lung: Cough
• Family h/x atopy (asthma/allergic rhinitis/eczema)
• Aggravating factor: smoking, allergen exposure i.e., smoke, dust, pets.
2. Expected physical Findings:

• Eyes: edema of lids, congestion and cobblestone appearance of the conjunctiva,


dark circle under the eyes (Allergic shiners)
• Nose: thin watery discharge of eyes or nasal, transverse crease/ black lines
across middle nasal bridge.
• Ear: retracted tympanic membrane or serous otitis media because of Eustachian
blockage
3. Differential diagnoses:

• Nasal Polyps
• NPC
4. Investigations:

• Skin Prick Testing


• Blood test for allergen specific test, eosinophils (IgE) count
• Nasal Endoscopy
5. State your final diagnosis:

Allergic Rhinitis

175
176 Oceanic Series End of Posting Compilation

6. Mention 4 types of treatment:

• Antihistamine
• Corticosteroid (oral/ topical)
• Anticholinergic (Intranasal spray)
• Mast cell stabilizers (sodium cromoglycate)
• Leukotriene receptor antagonist (Montelukast/ Zafirlukast)
Principles of treatment:

• Lifestyle modification (avoid allergens, hygiene care)


• Medications
• Immunotherapy if indicated
7. Possible Complications:

• Chronic rhinosinusitis • Orthodontic problem (due to


• Sleepiness, fatigue and irritability prolonged mouth breathing)
• Nasal Polyp • Bronchial asthma
• Serous Otitis Media in children
only.

176
177 Oceanic Series End of Posting Compilation

QUESTION 8 (OBSTRUCTIVE SLEEP APNOEA)

A 54-year-old obese office staff presents with noisy and difficulty in breathing while sleeping at
night and easily falling asleep during the day for past six months.

1. Symptoms you would ask to reach your diagnosis:

Nocturnal symptoms:

• Snoring- loud, habitual, and bothersome to others


• Witnessed apneas - often interrupt the snoring
• Gasping + chocking during sleep until arouse from sleep
• Recurrent awakening from sleep
• Daytime symptoms:
• Unrefreshing sleep
Daytime fatigue

• Morning headache, or sore throat


• Excessive daytime sleepiness (EDS)-that are not explained by other factors
• Cognitive (memory and intellectual) deficits
• Impaired concentration
• Personality, mood change; Depressed, anxiety
• Sexual dysfunction, decreased libido
2. Expected physical findings:

• Obese • High arch palate


• Large neck circumference • Other med conditions: CHF,
• Abnormal Mallampati score Stroke, DM type 2, Metabolic
• Narrow lateral airway walls syndrome
• Enlarged Tonsils

177
178 Oceanic Series End of Posting Compilation

3. Risk Factors:

Anatomical Factors Structural Factors Non-structural Factors Medical Conditions


-Enlarged tonsils -Facial elongation -Age -Hypothyroid
-Volume of tongue -Mandibular hypoplasia -Male -Neurological
-Soft tissue -Down’s syndrome -Obesity: BMI>30 syndromes
-Lateral pharyngeal -Marfan’s Syndrome -Central fat (muscular
walls -Prader- Willi syndrome. distribution dystrophy)
-Length of soft -Adenotonsillar -Sleep in supine -Stroke
palate hypertrophy position -Acromegaly
-Abnormal position -High arch palate -Alcohol use
of maxilla & -Nasal obstruction (nasal -Sedative use
mandible polyps, deviation, -Smoking
tumors, trauma, -Habitual snoring with
stenosis) daytime somnolence
-Retroglossal obstruction -Rapid Eye Movement
-Retropalatal obstruction Sleep (REM)
-Postmenopausal
state

4. Gold standard investigation: Polysomnography

5. Definitive treatment: Continuous positive airway pressure (CPAP)

6. Provisional diagnosis: Obstructive sleep apnea

7. Treatment:

Non-pharmacological Pharmacological

• Lifestyle modification: diet, • Modafinil


weight loss • SSRI
• Sleep positional therapy: sleep on Surgical
the side
• Nasal Surgery (Septoplasty/Sinus)
• Intraoral devices
• Tonsillectomy
• CIPAP
• Uvalopalatopharyngoplasty

178
179 Oceanic Series End of Posting Compilation

OSCE

OSCE 1 (ACUTE MASTOIDITIS)

A 3-year-old boy presented with a history of discharge from right ear for 3 weeks. On
presentation he has profuse mucopurulent discharge and tenderness behind right ear

1. Give four history to help with your diagnosis

• Hearing loss
• Hx of ear problems (acute otitis media)
• URTI problems
• Fever
2. State four clinical sign you would find in this patient

• Smooth iron like swelling at mastoid area


• Mastoid reservoir sign (immediate filling with pus after cleaning)
• Sagging of posterior superior meatal wall
• Nipple like Inflamed tympanic membrane
• Light house sign (pulsatile ear discharge)
3. State your diagnosis and two points to support your diagnosis

Acute mastoiditis

• Tenderness and inflamed behind the ear


• Sagging of posterior superior meatal wall

179
180 Oceanic Series End of Posting Compilation

4. State one investigation and one finding to document your diagnosis

• X Ray mastoid. Finding: clouding of the mastoid air cells


• Audiometry. Finding: conductive HL
5. Complications

• Labyrinthitis
• Meningitis
• Brain abscess
6. State two treatment for this patient

• Systemic therapy – broad spectrum antibiotic


• Cortical mastoidectomy
7. Complication of mastoidectomy

• Injury to facial nerve


• Dislocation of incus
• Rupture of sigmoid sinus (hah bukan sigmoid colon…..)

180
181 Oceanic Series End of Posting Compilation

OSCE 2 (MAXILLARY SINUSITIS)

A 36-year-old lady came to ENT clinic with complaints of pus discharge from both nasal cavities
for 6 month duration. She also complained of bilateral facial pain and right periorbital swelling
for 3 days.

1. List four points to ask in history

• Nasal obstruction
• Postnasal drip
• Fever
• History of atopy
• Family history
• Any pets at home
2. List four physical sign you can find in this patient

• Tenderness over the cheek (maxillary sinus area)


• Oedema of nasal mucosa
• Pus at middle meatus
• Inferior turbinate hypertrophy
3. State two differential diagnosis

• Sinusitis
• Nasopharyngeal carcinoma

181
182 Oceanic Series End of Posting Compilation

4. State two investigation and finding for each investigation

• Nasal endoscopy: pus at middle meatus


• CT scan of the paranasal sinus: air fluid level & mucosal thickening of maxillary
sinus
5. State your diagnosis

Chronic bilateral maxillary sinusitis

6. List two complication of this problem

• Meningitis
• Periorbital cellulitis
• Brain abcess
7. Name one surgical procedure that can be done in this patient

Functional endoscopic sinus surgery (FESS)

182
183 Oceanic Series End of Posting Compilation

OSCE 3 (ENT MALIGNANCY)

A 65-year-old man came to ENT outpatient clinic with a complaint of persistent ulcer on right
lateral border of tongue for 3 months without any improvement by medical treatment. He also
has history of tongue pain, loss of appetite and loss of weight.

1. List four complaints you can ask in the history

• Unexplained bleeding in mouth


• Difficulty in swallowing (dysphagia)
• Localised tongue pain
• Bad breath
• Speech problem
• Otalgia
2. State four risk factor

• Smoking
• Chewing betel nuts
• Alcohol consumption
• Family history
3. List four physical findings in this patient

• Red lesion on tongue


• Swelling of tongue
• Teeth problem
• Difficulty to move jaw
4. Give two differential diagnosis

183
184 Oceanic Series End of Posting Compilation

• Tongue ulcer
• Glossitis
5. State two investigation to confirm your diagnosis

• Tissue biopsy
• Radiology (ct scan of head and neck)
6. State your final diagnosis

Carcinoma of anterior ⅔ of lateral tongue

7. List two treatment option you can offer for this patient

• Surgical resection
• Radiotherapy, chemotherapy

184
185 Oceanic Series End of Posting Compilation

OSCE 4 (CHRONIC SUPPURATIVE OTITIS MEDIA)

RIGHT EAR LEFT EAR

A 6-year-old boy come to ENT clinic with complaints of right sided reduced hearing for six
months duration associated with poor academic performance. Upon further questioning, he also
has a history of frequent attacks of upper respiratory tract infection.

1. Mention 4 complaints you would ask.

• Any discharge • Vertigo


• Pain • Feeling of fullness
• Feeling of blockage • Balancing problem
• Tinnitus
2. Mention 4 relevant physical/otoscopic findings you would expect.

• Eardrum is oval • Altered light reflex


• Handle of malleus present and • Collection of fluid
shortened • No presence of discharge
• TM is retracted, red and bulging
3. List 2 differential diagnosis.

• Right ear chronic suppurative otitis media


• Right ear chronic catarrhal otitis media
4. State 2 appropriate investigation to confirm your diagnosis.

• Tympanometry
• Audiometry
The otoscopic findings for this patient are provided.

5. State your final diagnosis.

185
186 Oceanic Series End of Posting Compilation

Chronic right ear suppurative otitis media

6. Enumerate 2 possible complications.

Mastoiditis

Labyrinthitis

7. Mention 2 principles of management.

• Treat URTI
• Ear toileting- dry mopping, clean suction

186
187 Oceanic Series End of Posting Compilation

OSCE 5

A 25-year-old man presented to ENT clinic with bleeding from left nasal cavity.

1. List 4 important causes for the above scenario.

• Trauma
• Malignancy
• Medical condition (blood disorder/HPT)
• Consumption of drugs (NSAID/ anticoagulant)
2. Enumerate 2 important physical findings you must look for.

• Site the bleeding


• Any presence of mass/foreign body
3. Mention 2 principles of management.

• Assess the blood loss


• Stop the bleeding
4. State 4 steps involved in Trotter’s method (first aid).

• Patient need to sit and lean a little forward


• Over a basin to spit any blood any blood and
• Breath from mouth
• Cold compressed applied to nose to cause reflex vasoconstriction

187
188 Oceanic Series End of Posting Compilation

OSCE 6

A 56-years-old obese office staff presented with noisy breathing while sleeping at night and
easily falling asleep during the day for the past six months.

1. Mention 4 complaints you would ask.

• Snoring • Gasping
• Choking • Nocturia
2. Mention 4 physical findings you would expect.

• High volume of tongue • BMI >30


• High arched hard palate • Mandible hypoplasia
• Enlarged tonsils
3. List 4 non-structural risk factors for this patient.

• Obese • Alcohol drinking


• Smoking • Supine sleeping position
The attending physician assesses the patient by using ESS, his score is 18/24 and was arrange
the sleep study to determine the severity of this medical condition.

The test reveals total events of Apnea and Hypopnea is 240 during six hours of sleep.

4. Calculate the AHI (Apnea Hypopnea Index).

240/6 = 40 episodes per hours

5. State the final diagnosis and severity based on the sleep test.

Obstructive sleep apnea, severe stage

6. Mention the gold standard treatment for this patient.

Continuous positive air pressure (CPAP)

188
189 Oceanic Series End of Posting Compilation

OSCE 7 (CHRONIC SUPPURATIVE OTITIS MEDIA)

A 12-year-old girl was brought to the clinic by her mother with complaints of foul-smelling left ear
discharge with left facial muscle weakness, reduced hearing in left ear for the past few weeks.
The girl has past medical history of scanty foul-smelling left ear discharge for three years.

1. List four other complaints you would ask to help for your diagnosis. (2 marks)

• Ear pain • Vertigo / dizziness


• Post-auricular pain • Headache
• Tinnitus
On examination, the girl was afebrile with normal vital signs. Examination of the right ear was
normal. Examination of the left ear showed purulent discharge in the external auditory canal.
After cleaning the external auditory canal, otoscopic examination was done and findings were
provided.

2. List three other important findings from her otoscopic examination. (3 marks)

• Perforations in the pars flaccida / attic region


• Bits of shiny white flakes / debris through the perforation
• Partly congested TM
• Dull TM
3. State your clinical diagnosis. (2 marks)

Left attico-antral CSOM with cholesteatoma

4. Mention two complications of this clinical condition. (1 mark)

Subperiosteal abscess/ Facial nerve palsy/ Labyrinthine fistula / Labyrinthitis Meningitis /


Brain abscess

189
190 Oceanic Series End of Posting Compilation

5. Mention two investigations to evaluate / assess the clinical condition. (1 mark)

• Pure tone Audiogram (PTA)


• CT / MRI scan of the mastoid and brain
6. State the definitive treatment. (1 mark)

Left modified radical mastoidectomy with tympanoplasty

190
191 Oceanic Series End of Posting Compilation

OSCE 8 (NASAL POLYP)

A 50-year-old male patient came to hospital with gradual bilateral nasal blockage for three
months with history of sneezing and nasal discharge for 12 years. On examination, pale
swellings were found in both nasal cavities.

1. List four points you would ask to reach diagnosis. (2 marks)

• Persistent cold.
• Postnasal discharge.
• Dull headaches.
• Hyposmia or anosmia.
• Mouth breathing / snoring / H/O inhalant allergies.
• H/O bronchial asthma / H/O aspirin sensitivity.
2. List four clinical signs you would elicit to help in diagnosis. (2 marks)

• Bilateral multiple smooth shiny pale greyish nasal masses.


• Anosmia/hyposmia.
• Blocked nasal airway.
• Postnasal drip.
• Facial deformity – broad nasal base.
3. State your clinical diagnosis and mention two physical signs to justify it. (2 marks)

• Bilateral nasal polyps / Ethmoidal polyps.


• Signs - Bilateral multiple smooth shiny pale greyish nasal masses.
Anosmia/hyposmia.
4. Mention one specific imaging and its reason. (2 marks)

191
192 Oceanic Series End of Posting Compilation

• Imaging: Coronal CT scan of paranasal sinuses and skull base.


• Reason: To know anatomical landmarks & variations and extension of disease.
5. State the mainstay of treatment according to your diagnosis. (2 marks)

Steroid Therapy

192
193 Oceanic Series End of Posting Compilation

OSCE 9 (ACUTE TONSILITIS)

A 24-year-old woman presented with three-day history of an increasing throat pain with general
malaise.

1. Mention four complaints you would ask to reach the diagnosis. (2 marks)

• High fever • Mouth breathing / Snoring /


• Dysphagia Earache /Headache
• Odynophagia
2. Enumerate four physical findings you would expect. (2 marks)

• Enlarged and inflamed tonsils • Tender cervical lymph nodes /


• Exudate over tonsils cervical LN enlargement
• Uvula in midline • Neck stiffness
3. Mention two differential diagnoses for this patient. (1 mark)

Acute pharyngitis / acute tonsilitis / lymphoma of tonsils / carcinoma of tonsils

4. State your final diagnosis. (1 mark)

Acute tonsilitis

5. Enumerate two possible complications. (2 marks)

Peritonsillar cellulitis / peritonsillar abscess / otitis media / OSA / Glomerulonephritis /


Rheumatic fever / Septic arthritis

6. Mention two treatment measures for this patient. (2 marks)

• Maintain adequate hydration • Paracetamol to control fever


• Analgesic to control pain • Antibiotic therapy

193
194 Oceanic Series End of Posting Compilation

OSCE 10 (OTITIS MEDIA WITH EFFUSION)

7 years old boy come with a complaint of right sided reduced hearing for 6 months duration. It is
associated with poor studying performance. Upon further questioning, his father complaint that
his son has frequent onset of URTI.

1. List 2 complaints to help for diagnosis

Ear pain/Ear discharge/Vertigo

2. Physical findings

Red, absent of light reflex, air-fluid level

3. 2 differential diagnoses

Otitis media with effusion/ Eustachian tube dysfunction/ Acute otitis media

4. Investigation to confirm diagnosis

• Tympanometry
• Pure tone audiogram
5. Final diagnosis

Right sided otitis media with effusion

6. Potential complication

Perforation of eardrum/Cholesteatoma/Reduced hearing

7. Principal of management

• Treat URTI
• Myringotomy + Grommet insertion

194
195 Oceanic Series End of Posting Compilation

OSCE 11 (RETROPHARYNGEAL ABSCESS)

Lateral view of X-ray of retropharyngeal abscess was given.

46 years old man come to ED with very severe sore throat and fever for 2 days duration. History
of accidental ingestion of chicken bone a week ago. No obvious neck swelling is seen.

1. Other complaints you want to explore

• Difficulty in swallowing • Change in quality of voice


• Painful swallowing
2. Expected physical finding

• Cervical lymphadenopathy • Drooling of saliva


• Posterior wall of pharyngeal wall • Limitation in mouth opening
swelling, congestion, edema • Neck tenderness
3. Differential diagnosis

• Epiglottis
• Laryngitis
• Pharyngitis
• Pharyngeal abscess
• Retropharyngeal abscess

195
196 Oceanic Series End of Posting Compilation

4. Name of imaging and finding

• Soft tissue neck X-ray lateral view


• Finding: Widening of prevertebral shadow, loss of cervical spine curvature,
presence of air fluid level
5. Final diagnosis

Retropharyngeal abscess

6. Possible complications

Airway obstruction, septicemia, acute respiratory distress, esophageal perforation

7. Principal of management

• Admit to hospital
• IV antibiotic
• IV fluid
• Incision and recession
OSCE 12 (CA LARYNX)

1. What additional information you would want to take from the history?

• Any difficulty in breathing?


• Any noisy breathing?
• Any neck pain?
• Any difficulty in swallowing?
• Any pain during swallowing?
• Any neck swelling?
• Any passing of friable tissues / blood?
2. State the findings.

• Presence of mass
• Occupy at right glottis level
• Extend to surrounding structure
• Mass partially block the airway

196
197 Oceanic Series End of Posting Compilation

3. State the clinical diagnosis.

Right sided glottic carcinoma

4. Investigations which can be done.

• Direct laryngoscopy
• CT Scan
• MRI Scan
OSCE 13 (CHRONIC SUPPURATIVE OTITIS MEDIA)

1. Mention a clinical procedure to reach your diagnosis.

• Otoscopy
• Examination under miscroscope
• Pure Tone Audiometry (PTA)
2. State the clinical diagnosis.

Chronic suppurative otitis media of the left ear

3. List the expected physical findings.

• Pus in the ear canal / middle ear cavity


• Eardrum perforation
• Conductive hearing loss
• Absence of light reflex
• Negative Tragal sign

197
198 Oceanic Series End of Posting Compilation

4. List the potential complications of the condition.

• Meningitis
• Encephalitis
• Facial nerve palsy

198
199 Oceanic Series End of Posting Compilation

st
Oceanic Question Bank 1 Edition
OPHTHALMOLOGY

199
200 Oceanic Series End of Posting Compilation

EMQ

1. A gardener was hit by a tree leaf in his eyes. He


presented with eye discharge. On slit lamp examination,
white flake with fluffy edge on the cornea

2. Pt have hx contact with pt above, the came with red eye


and was given prednisolone eye drop. 1 week later still
did not resolve and have blanching of cornea.
3. 56 years old woman, complain of poor vision right eye. Vitreous
Known diabetic for 6 years with poor compliance hemorrhage
medication and follow up. Visual acuity achievable is
counting finger one metre right eye, 6/18 left eye.
Ocular media is clear, but fundoscopy have absence
red reflex
4. 6 months old kid. Mom realised abnormality at one of
her son’s eye. White patchy at left eye. Absent of light
reflex.

5. 10 years old boy with history of dental caries, has Preseptal cellulitis
periorbital redness and swelling, discomfort of right eye.
Visual acuity is 6/9 and extra ocular movement full for
both eyes.

6. 65y/o paddy planters with hypopyon, lesion with Fungal infection


irregular fudging edges at macula area.

7. 50 years old male with acromegaly and visual field Bilateral temporal
changes hemianopia

8. Upon fundoscopy, there was inferior pole scarring on Right altitudinal


her right eye superior/inferior
defect
9. Old age, disciform scar, drusen at posterior pole of eye Central scotoma

200
201 Oceanic Series End of Posting Compilation

10. 23 y/o, Loss of vison w hx of mva. Recovered on itself Vitrous


after 6 weeks Haemorrhage
11. Diabetic patient, hard exudates on funduscopy Background
diabetic
maculopathy
12. Hyperlipidaemia, hypertension, tortous and dilated CRVO
vessels
13. Mother noticed white patches on center of left eye of Retinoblastoma
her child
14. Pt with sinusitis came with pain ocular movement, Orbital cellulitis
periorbital swelling
15. 20 y/o man, hx of MVA, lens aspiration, but no IOL d/t
retinal detachment. 6 months later IOP 50 mmhg
16. Teenagers comes to ED w hx of cement injury into the
eyes. Irrigation was done immediately w/out taking any
history
17. 70y/o was diagnosed w left central vein occlusion. He
had rubeosis iridis on the left eye. What kind of
glaucoma it is
18. 70 years old lady sudden loss of vision, papillary dilated
on light reflex , exam show difffuse white & cherry red
spot
19. 55y/o woman presented w eye pain, redness, headache
and reduce vision on the left eye. Hx of iv injection /
retrobulbar injection of dexamethasone for diabetic
maculopathy. She had conjuctival injection, mild corneal
edema and vitritis
20. 65 y/o women underlying dm, poor compliance with
medication complaint of right eye loss vision..uoon
visual acuity examnination, right eye-hand movement,
left eye 6/18..what do u expect from fundus findings
21. 6months old baby was brought to Eye department. Her
mom notice abnormality in her baby’s eye. There was

201
202 Oceanic Series End of Posting Compilation

white patch at the central of the eye. EUA, there was


white opacity at the central eye and reduce light reflex

22. Pituitary adenoma Bitemporal


hemianopia
23. Macular hole Central Scotoma

24. Papilloedema- Increase blind spot

25. Young boy, red eyes 1 week Viral conjuctiva?

26. Superior tarsal sclera injection, Phenylephrine

27. Child contant rubbing eyes Vernal conjunctiva

28. Iron deposit at iris

29. Black curtain, flashes

30. 20 y/o has a history of MVA and develop secondary Angle recession
cataract. After a few years, presenting with visual glaucoma
blurriness

31. 60 y/o, mature cataract, increased intraocular pressure Phacolytic


of 40mmHg glaucoma
32. Paddy leaves hit the eye of a farmer. Develop whitish Fungal
eye and sticky eye discharge conjunctivitis
33. Sudden onset of loss of vision at 6pm, nausea and Acute angle closure
vomiting, anterior shallow chamber, left glaucoma
eye pupil semidilated.

34. 8 years old son with bilateral pink eye, frequent rubbing Allergic
eye conjunctivitis
35. 3 microaneurysms Mild non-
proliferative diabetic
retinopathy

202
203 Oceanic Series End of Posting Compilation

36. Venous beadings in 2 quadrants Severe non-


proliferative diabetic
retinopathy
37. Retinal thickening or edema in <500 microns from Clinically significant
fovea maculopathy
38. 20 y/o has a history of MVA and develop secondary Angle recession
cataract. After a few years, presenting with visual
blurriness

39. 60 y/o, mature cataract, increased intraocular pressure Phacolytic


of 40mmHg glaucoma

40. Paddy leaves hit the eye of a farmer. Develop whitish Fungal
eye and sticky eye discharge conjunctivitis

41. Sudden onset of loss of vision at 6pm, nausea and Acute angle closure
vomiting, anterior shallow chamber, left eye pupil glaucoma
semidilated

42. 8 years old son with bilateral pink eye, frequent rubbing Allergic
eye conjunctivitis

43. Theme 4: Cause of Uveitis

A Ankylosing spondylitis F Posner Schlossman


syndrome

B Bencet disease G Rheumatoid arthritis

C Fuch uveitis H Systemic lupus erythematosus

D Juvenile rheumatoid arthritis I Systemic sclerosis

E Psoriatic arthritis

20 y/o man come with bilateral red eyes and confirmed


evidence of uveitis. O/E: chronic back

203
204 Oceanic Series End of Posting Compilation

Ankylosing

pain and lower back stiffness. spondylitis

9 y/o girl presenting with bilateral joint pain of both Juvenile rheumatoid
hands associated with uveitis. arthritis

44. 45 y/o patient with acromegaly has this visual defect

45. 60 y/o man with multiple drusens on macula has visual


defect
46. Optic disc 0.8, laminar dots
47. A 35 year old man with dental caries for the past 2 week Orbital cellulitis
came with periorbital redness, eye pain and reduced
visual acuity on his left eye for 1 week. He also have
limited eye movement and complete ptosis on the same
eye
48. 25 y/o rheumatic, left eye redness and discomfort.
Localized redness at superotemporal conjunctiva which
disappeared with phenylephrine eye drop.
49. Female had MVA. Complaint of progressive vision loss
over 1 month period. Visual acuity was hand movement
50. 65 y/o women underlying dm, poor compliance with
medication complaint of right eye loss vision..uoon
visual acuity examnination, right eye-hand movement,
left eye 6/18..what do u expect from fundus findings
51. 6months old baby was brought to Eye department. Her
mom notice abnormality in her baby’s eye. There was
white patch at the central of the eye. EUA, there was
white opacity at the central eye and reduce light reflex
52. 50 y, female, painless gradual visual loss for both eyes, Primary open angle
iop increased, cdr 0.6, 0.7, goniometry shows iii & iv glaucoma
360 deg of both eyes

204
205 Oceanic Series End of Posting Compilation

53. 2. Loss of vision of left eye. Has nausea n vomiting. Primary close angle
Right pupil constricted with light but left pupil was glaucoma or
semidilated. Shallow anterior chamber Primary angle
closure suspect
54. Thyrotoxicosis feel foreign body sensation superior limbic
keratocunjctivitis
55. 5 year old, periorbital edema is purple bluish Verneal
discoloration, rapd negative conjunctivitis(xsure)
MAYBE ORBITAL
CELLULITIS
56. woman with RA have red eyes for 2 weeks, relieved by episcleritis
lubrications.
57. Long standing hypertension & hyperlipidemia. Hypertensive
Funduscopy: Flamed-shaped haemorrhage RAPD retinopathy (not
negative sure) boleh jadi
CRVO
58. Undelying DM found retinal thickening 1 disc diameter clinically significant
from fovea macular edema ( not
sure)
59. 56 years old woman, complain of poor vision right eye. Vitreous
Known diabetic for 6 years with poor compliance hemorrhage
medication and follow up. Visual acuity achievable is
counting finger one metre right eye, 6/18 left eye.
Ocular media is clear, but fundoscopy have absence
red reflex.
60. 6 months old kid. Mom realised abnormality at one of
her son’s eye. White patchy at left eye. Absent of light
reflex.
61. 10 years old boy with history of dental caries, has Preseptal cellulitis
periorbital redness and swelling, discomfort of right eye.
Visual acuity is 6/9 and extra ocular movement full for
both eyes

205
206 Oceanic Series End of Posting Compilation

62. 65y/o paddy planters with hypopyon, lesion with Fungal infection
irregular fudging edges at macula area
63. 50 years old male with acromegaly and visual field Bilateral temporal
changes hemianopia
64. Upon fundoscopy, there was inferior pole scarring on Right altitudinal
her right eye superior/inferior
defect
65. Old age, disciform scar, drusen at posterior pole of eye. Central scotoma
66. 50 year old man with left optic tract traction due to mva Right homonymous
hemianopia
67. Funduscopy presence of flame hemorrhage, exudates, Superior altitudinal
cotton wool spots at the inferior half of the left retina of left eye
68. 60 year old woman complained of pain and reduced endophthalmitis
vision in left eye for 2 days, went for left extracapsuler
extraction 4 days ago, cornea was hazy, presence of
hypopyon and viritis
69. 8 years old presented with periorbital redness, eye periorbital cellulitis
discomfort and slight ptosis. VA and EOM movement
are normal
70. Contact lens wearer complained of pain, vision loss for Bacterial keratitis
1 week. On slip lamp examination, there’s 2 corneal
infiltration and hypopyon
71. 70 year old man with sudden visual loss, papillary reflex Central retinal artery
is dilated. On fundoscopy, there is homogenous retinal occlusion
whitening with fovea cherry red spots
72. 50 years old Chinese lady complained of sudden left Primary (acute)
eye visual loss associated with conjunctiva hyperemia. Angle closure
On ocular examination the right eye was normal but left glaucoma
eye has fixed dilated pupil and non reactive. Both of the
anterior chambers were shallow
73. 50 yo woman bilateral progressive loss vision. Right left
Optic disc ratio 0.7 n 0.8. Iop right left 38 n 28. Angle
graded as III-IV 360 degree

206
207 Oceanic Series End of Posting Compilation

74. 50 y/o chinese lady with sudden left eye visual loss
assoc with headache and nausea. On ocular
examination the right wye was normal but left eye was
partially dilated and non reactive. Both of the anterior
chamber were shallow

75. Underlying DM found retinal thickening 1 disc diameter


from fovea - clinically significant macular edema

76. pt had thyrotoxisosis with bilateral eyes redness & superior limbic
foreign body sensation - superior limbic keratoconjunctivitis
keratoconjunctivitis

77. 40 yo woman with known history of RA had localized


left eye pain, resolved with lubricants after two week

78. Protrusion of eye, lid retracted , tachycardia, red eye -Superior limbic
and sensation of foreign body keratoconjunctivitis
(in thyroid eye
disease)

79. Child presented with bluish purple swelling of eye, -Preseptal cellulitis
RAPD negative
If RAPD+ orbital
cellulitis

80. A blacksmith, iron piece get into the stroma of his intraocular foreign
anterior chamber body (toxic)

207
208 Oceanic Series End of Posting Compilation

81. History of MVA, presented with loss of vision. On viterous


examination, visual acuity is hand movement and haemorrhage
anterior chamber is clear

82. Nail trauma causing injury to the eye, fluorescence corneal abrasion
staining in cornea and siedel test negative

83. History of cataract and swollen lense phacomorphic


glaucoma

84. History of CRVO presented with pain red eye complication of


CRVO is
neovascular
glaucoma

85. 25 years old tennis player presented to Eye Department Hyphaema


after injury to right eye. Examination revealed 1.2mm
blood in anterior chamber of right eye. Left eye normal

86. MVA recently. Decrease in vision but everything okay Traumatic optic
neuropathy

87. 36 years old male teacher trying to hang a photo frame, corneal laceration
when the nail recoils and hit his left eye. He complained
red eye. Seidel’s test positive

88. 58 years old lady with underlying 18 years of history of Diabetic retinopathy
diabetic come for medical check up due to right eye
vision problem. Normal bilateral IOP, right eye 6/60, left
6/18

89. Mother complained her 2 months old child has white congenital cataract
patch in right eyes. Past history revealed the baby was
delivered via forceps delivery. Red reflex was abnormal

90. A man has uveitis and stiffness of the back and lower ankylosing
back pain spondylitis

91. 9 years old with chronic joint pain and having uveitis Juvenile RA

208
209 Oceanic Series End of Posting Compilation

92. a 60 years old man with type 2 dm of 20 years duration Annually


had routine fundoscopy. There are no
diabetic retinopathy findings.

93. 54 years old gentleman with type 2 DM for 15 years 2 to 4 months check
duration went to routine eye check up. up
Mild nonproliferative diabetic retinopathy seen

94. A man has uveitis and stiffness of the back and lower Ankylosing
back pain spondylitis

95. A 9-year old girl came down with uveitis. She also has Juvenile rheumatoid
joint pain in her hands. arthritis

96. 8 years old was presented with periorbital redness, eye Periorbital cellulitis
discomfort and slight ptosis. VA and EOM movement
are normal

97. On fundoscopy, there was presence of flame Superior altitudinal


hemorrhage, exudates, cotton wool spots at the inferior of left eye
half of the left retina

98. On fundoscopy, there was presence of flame Superior altitudinal


hemorrhage, exudates, cotton wool spots at the inferior of left eye
half of the left retina

99. 5 years old girl was referred by her optometrist for Anisometric
suspected decreased vision in her left eye. On further amblyopia
examination, there was a difference in refractive error
between both eyes.

100. Congenital cataract;


A six-month-old child was brought to the Eye
bukan
Department, the mother complaining that the eyes of
retinoblastoma sbb
the child looked abnormal. There was a white patch in
usually kalau
the centre of the left eye. On examination under
retinoblastoma it is
anaesthesia, a whitish patch was seen behind the
hereditary and lahir
pupil and a diminished red reflex was noted on
lahir dah ada.
funduscopy

209
210 Oceanic Series End of Posting Compilation

101. Vitreous
A 56-year-old female is examined at the Eye
haemorrhage
Department, following the complaint of poor vision in
her right eye for the last one month. She is a known
diabetic patient for six years with poor compliance for
follow up and medications. Her best corrected vision is
right eye counting finger at one metre, left eye 6/18.
On right eye examination, ocular medias are clear
except the fundus, giving rise to an absence of red
reflex on funduscopy.
102. Posterior
A 30-year-old woman complaining of progressive
subcapsular
blurring of vision in both eyes. Her visual acuity is 6/12
cataract
and 6/60 for right and left eye respectively, not
improving with pinhole test. She is under medical
follow-up for her multiple joint pain and swelling and on
regular oral therapy. Slit lamp examination shows lens
opacities.
103. Oil droplet cataract
An infant presented with failure to thrive, lethargy,
vomiting and diarrhoea. Systemic examination
revealed hepatosplenomegaly, anaemia and deafness.
The baby was also referred to ophthalmologist for full
ocular assessment. Examination showed presence of
bilateral lens abnormalities.
104. Relax
A 42-year-old woman is reading a book. She is
complaining of difficulty to read small prints for the
past one year. Otherwise, she is healthy. State the
condition of her lens zonules when she is reading her
book.
105. To kill ischemic
A 40-year-old man with type-1 diabetes presents to
retina
eye clinic complaining of decreased vision in his both
eyes. He has not seen any eye doctor in years. On

210
211 Oceanic Series End of Posting Compilation

ocular examination, there are numerous dot-blot


haemorrhages, hard exudates and area of abnormal
vasculature in the retina. Pan-retinal photocoagulation
(PRP) is planned for him. This is the reason for doing
PRP.
106. Maxillary
A 40-year-old man with type-1 diabetes presents to
eye clinic complaining of decreased vision in his both
eyes. He has not seen any eye doctor in years. On
ocular examination, there are numerous dot-blot
haemorrhages, hard exudates and area of abnormal
vasculature in the retina. Pan-retinal photocoagulation
(PRP) is planned for him. This is the reason for doing
PRP.
107. Non-ischemic
A 65-year-old man complained of sudden reduction in
central retinal vein
vision on the right eye. Visual acuity on the right eye
occlusion
was 6/36, not improved with pin hole. The left eye was
6/9. On pupillary examination, relative afferent
pupillary defect was negative. Fundus examination
showed scattered flame-shaped haemorrhages and
few cotton wool spots. Fundus fluorescein
angiography was two disc diameters of capillaries non
perfusion area.
108. Clinically significant
50-year-old man with type II diabetes mellitus for five
diabetic macula
years complained of difficulty to read newspapers for
oedema
three months duration. Fundus examination showed
bilateral hard exudates at half disc diameter from
fovea.
109. Commotio retinae
A 30-year-old man had an accident. He complained of
cloudy vision in the left eye. Visual acuity was 6/60,
relative afferent pupillary defect was negative and the
macula looks white. A few weeks later his vision was
almost normal.

211
212 Oceanic Series End of Posting Compilation

110. Basic chemical


A teenager presented in casualty department with
injury
history of cements injury to the both eyes. Copious
irrigations of the eyes were immediately done without
asking further history.
111. Orbital fat prolapse
A young patient was brought to you in the Emergency
is a sign of orbital
department following an accident. An initial
septum damage.
examination showed that the patient has an ocular
trauma.
112. Painful ocular
A 45-year-old woman complaining of a painful red eye
movement
in the left side. She noticed the symptoms on waking
up in
113. Anterior uveitis
A 23-year-old male patient presents with a two days
history of painful red right eye and photophobia. On
examination, there is posterior synechiae in the right
eye, but no similar finding anterior chamber of the right
eye. On review systems, he reports a six months
history of progressive lower back stiffness
114. Reduced Ishihara
A 25-year-old lady presents to the Ophthalmology
plate score
clinic complaining of a one week history of severe pain
on movement on her left eye. Over the first four days
she noticed her vision was much reduced in that eye,
but feels that this has somewhat improved over the
last three days. Her doctor suspects an acute optic
neuropathy.

212
213 Oceanic Series End of Posting Compilation

MEQ 1 Retinoblastoma

60-year-old man, presented with decrease vision (progressive). VA also reduced. When using
pinhole, no effect.

1. 3 diff dx
AMD, cataract, glaucoma

2. 3 hx to ask to lead to diagnosis


Amd- do u smoke, family hx of amd
Cataract- do u have glare, change glasses frequently
Glaucoma – do you have any pain in ur eyes

3. 2 specific ocular examination


Ophthalmoscope, OCT

4. Result of funduscopy show drusen.


What is drusen? – tiny yellow or white accumulations of extracellular material that build
up between Bruch's membrane and the RPE

5. Diagnosis? – Age Related Macular Degeneration

50 y/o underlying HT and DM (fundoscopy show immature type)

1. state 4 step to measure the visual acuity

2. Type of cataract
-subcapsular
-cortical
-neuritic sclerotic

3. Full dx

213
214 Oceanic Series End of Posting Compilation

4. 2 preop investigation
-A scan to measure the size of cornea
-keratometer to measure the cornea power

5. The best treatment


Phacoemulsification
6. Complication
Intra op:
-
-
Post op
-
-

MEQ 2 (TUBERCULOUS OCULAR)


50 years old with history of chronic cough for 2 years and progressive blurring of vision.
1. 2 other symptoms you would like to ask
2. 4 ocular signs to make a diagnosis
3. What is the etiological organism for this disease?
4. 2 investigations to make diagnosis
5. 2 treatment
6. What are the complication of the disease or from the medication?

214
215 Oceanic Series End of Posting Compilation

MEQ 3
20 years old man came to the eye clinic with the complaint of progressive blurry vision on both
eyes. He had the history of diabetes mellitus since teenage. He also complained of unable to
read in near distance.

1. 3 risk factors

Long duration of having DM, poor diabetic control, nephropathy, hypertension, smoking,
hyperlipidemia

2. 3 benefits of good diabetic control (any relevance answer is accepted)

• Able to slow down the progression of diabetic retinopathy


• Able to prevent the complication of nephropathy
• Able to reduce the risk of having heart disease

3. 2 changes in the features of blood vessel in persistent diabetes

Intramural pericyte damage due to sorbitol, outpouching of capillary wall, loss of vascular
smooth muscle, proliferation of endothelial cells, vessel leakage, neovascularization

4. Reason for patient to have central scotoma

Maculopathy

5. 3 indication for urgent referral to ophthalmologist (based on CPG)

• Sudden severe vision loss


• Symptoms or signs of acute retinal detachment

1. Mother brought 6mo child with excessive crying since birth. During examination child showed
irritability when examined with torch light and bilateral contracted eyelids

1. 3 Ocular signs based on scenario

2. Examinations that can be done

3. Abnormality seen in the photo ( congenital glaucoma)

4. Diagnosis

215
216 Oceanic Series End of Posting Compilation

5. Aetiology

6. Management

-pharmacology

-surgical

Amblyopia *could not remember much as Doctor discuss without shows the question

Hx to elicit

RF of amblyopia

Causes of amblyopia

Name of the phenomena of amblyopia

How to dx amblyopia

How to treat amblyopia (cover the bad eye, force the good eye to become amblyopic)

MEQ 4 (CORNEAL ULCER)

54 years old woman, works as labour worker, presents with history of severe left eye pain and
tearing for five days, after plucking tea at plantation. Visual acuity for left eye 6/18, right eye was
normal. On examination, hypopyon was seen with circum cornea injection.

1. Get 2 more information from history that will help in management of this patient.

2. what is your provisional diagnosis. Ans: Left corneal ulcer

3. what are the proper investigation for the patient?

4. List four management for this patient.

75 years old Chinese lady presented with sudden severe pain and loss of vision of the righ eye
associated with nausea and vomiting. On examination, her left visual acuity was 6/12 with
evidence of cataract. Her right visual acuity was counting fingers and presented
with hyperemia of the conjuctiva. Both of the anterior chamber are shallow. Right eye was

216
217 Oceanic Series End of Posting Compilation

oval, semidilated and non-reactive to the light while left eye was normal. The intraocular
pressure was 55mmHg in the right eye and 25mmHg in the left eye.

1. State the probable differential diagnosis

Primary angle closure glaucoma - (Left)

2. State two risk factors in this patient

i. Age: The average age of relative pupillary block is about 62 years at presentation. Non-
pupillary block forms of primary angle closure tend to occur at a younger age. (*formation
of pupillary block involved in the pathogenesis of angle closure glaucoma)

ii. Refraction: Eyes with ‘pure’ pupillary block are usually hypermetropic. Non-
pupillary block mechanism can occasionally occur in myopic eyes.

iii. Axial length: Short eyes tend to have a shallow AC secondary to a relatively
anterior lens position. Eyes with nanophthalmos (axial length less than 20 mm) have a
very short eye and are at particular risk.

3. State on IV drug that can be used to reduce the intraocular pressure in this patient and explain
the mechanism of action.
Acetazolamide 500 mg is given intravenously if IOP >50 mmHg and orally (not slow-
release) if IOP is <50 mmHg.

MOA: It works by blocking the action of an enzyme called carbonic anhydrase.


Blocking this enzyme reduces the aqueous humour and helps to lower the IOP.

4. List two topical drugs that can be used and state the indication for each drugs.
i. Pilocarpine 2% four times daily to the affected eye and 1% four times daily to the fellow
eye.
ii. Topical steroid (prednisolone 1% or dexamethasone 0.1%) four times daily if the eye is
acutely inflamed

217
218 Oceanic Series End of Posting Compilation

5. State two ocular examination for this patient.

i. Anterior segment OCT

ii. Anterior chamber depth measurement

iii. Biometry

iv. Posterior segment ultrasonography

Ocular examination was done when the cornea was clear, angle right eye 0-I and left eye I-
II 6. State two structure that normally can be visualised in this examination.

i. Trabecular meshwork
ii. Schwalbe’s line

MEQ 5
A 60 year old man presented with painless blurring of vision in both eyes for six months
duration.
1. Mention 4 details in the history you would like to ask to help with diagnosis
• (Ask risk factors)
• Hypertension
• DM, well controlled or not
• Hx of trauma / surgery
• Progressive?
• Onset

On further questioning, the patient complaint of polyuria and nocturia for the past
three years. You performed systemic and fundus examination and noted there was

218
219 Oceanic Series End of Posting Compilation

evidence of dot-blot haemorrhages, cotton wool spots (CWS), hard exudates (HE) and
flame shaped haemorrhages in the both eyes. There was also present of one
intraretinal microvascular abnormalities (IRMAs) in the left eye.
2. State the definition of the
lesions. CWS : Infarction of
nerve fibers
HE : leakage of plasma

IRMA : collateral, abnormal branching, sinuous shunt vessels that typically develop adjacent
to areas of capillary nonperfusion or cotton wool spots.

3. Describe two pathogenesis of this condition


• decrease oxygenated blood flow
• microvascular occlusion & leakage

4. List four stages of this ocular disease.

Mild Non-proliferative Diabetic Retinopathy

Moderate NPDR

Severe NPDR

Proliferative Diabetic Retinopathy

5. State the most likely ocular diagnosis/stage for the left eye.

Severe NPDR

This patient complained of sudden loss of vision in the left eye after about 6 months
of the initial presentation.
State the most likely cause of this sudden loss of vision and its management.pars planar
vitrectomy

219
220 Oceanic Series End of Posting Compilation

Retinal detachment // PPV

MEQ 6

40-year-old women came to OPD, presented with sudden onset of painless loss of vision

in the left eye 3 month ago. Visual acuity (VA) of the affected eye was hand movement with

positive RAPD. VA of the right eye was 6/9. An ocular examination using a handheld

instrument was done by a general practitioner doctor.

a. Mention 4 possible cause of sudden loss of vision for this patient.

• CRAO

• CRVO

• Advanced glaucoma

• Retinal detachment

• Vitreous hemorrhage

• Arteritic/non-arteritic anterior ischemic optic neuropathy

b. Mention 3 question you would ask to reach the diagnosis.

• Any history of hypertension, DM, hyperlipidemia

• History of smoking

• History of ocular trauma

• Does she have high myopia

c. Describe the definition of positive RAPD.

• During swinging light test, there is dilatation of the pupil when the light is

shone to the eye due to the retinal afferent nerve abnormality or due to

220
221 Oceanic Series End of Posting Compilation

retinal disease.

d. State the most possible finding during this examination.

• Optic disc atrophy (3 month- chronic)

221
222 Oceanic Series End of Posting Compilation

OSCE
1. How to use fundoscopy, visual acuity in 3M osce room
2. Central retinal vein occlusion

65 y/o man with hx of HPT came to emergency for sudden blurry vision for 3 days

Q1 state 3 findings from the pic shown

Q2 state the diagnosis

Q3 2 possible systemic condition other than HPT

Q4 Management for this patient

Q5 indication of using laser therapy?

3. Toxoplasmosis

Q1 findings from funduscopic image

Pigmented clumps in macula area

Scarring of macula area

222
223 Oceanic Series End of Posting Compilation

Pallor of the optic disc

Q2 what are the ocular signs to elicit

Q3 diagnosis

Toxoplasmosis

Q4 definitive host

Cat

Q5 Diagnostic test

PCR

Q6 Indication for treatment

Immuno compromised patients

Macular to optic nerve distance

Q7 What is the 1st line treatment

Azithromycin/prednisolone

Q8 list out 2 possible complications

4. Diabetic retinopathy

Q1 pathogenesis of the condition


Q2 fundoscopic findings
Q3 relevant investigation

5. Anatomy of fundoscopy

223
224 Oceanic Series End of Posting Compilation

6.

7. Right eye strabismus

Q1 diagnosis right eye strabismus


Q2 type of ocular muscle and innervation SO4 – inward and upward LR6- eye inward
Q3 investigation hischeburg test
Q4 management eye patch, botulinum toxins, surgical resection and recession

224
225 Oceanic Series End of Posting Compilation

8. u/l dm, sudden loss of vision of left eye. RAPD+, right eye visual acuity 6/6

Q1 finding from the fundus image – flame shape hemorrhages, dot blot
Q2 hx/ symptoms to support the diagnosis hx of headache, HPT
Q3 diagnosis CRVO
Q4 list 2 investigation – Fundus, fluroscene angiography, goniotomy

9.

225
226 Oceanic Series End of Posting Compilation

Papilloedema

Q1 give 2 clinical signs in the fundoscopy

Blurring and raised margin of optic disc

No CDR ratio

The funduscopy of other eye shows the same findings

Q2 What are etiology for this finding

Raised intracranial pressure (space occupying lesion)

Q3 2 causes that cause this condition

Space occupying lesion

Malignant hypertension

226
227 Oceanic Series End of Posting Compilation

Q4 what are primary investigation?

Urgent CT scan of the Brain

Q5 what is management?

OSCE 1 – Coronal view of CT Scan of right sided blow out orbital fracture

10 years old kid with orbital trauma during playing sport, associated with periorbital swelling.

1. Interpret the image and state the dx


left blow out fracture of the orbital floor

Q2 list three possible injuries of the eyeball in this condition


Globe rupture, hyphema, traumatic iritis, retinal detachment, choroidal tear

You palpate the affected eyelids and there is crepitations under the skin.

- State the medical terminology for this sign. (0.5 marks)


Subcutaneous emphysema

- List three other signs (outside eyeball) you will assess for this patient. (3 marks)
Restricted eye movement, enophthalmos, subcutaneous emphysema, fracture at
the roof (other than orbit)

227
228 Oceanic Series End of Posting Compilation

- State the X-ray view that you will order when instructing the radiographer, for a
plain X-ray. (1 mark)
Occipitomental view (Water’s view)

- Mention an instruction to the patient during early stage of management. (1 mark)


Do not blow the nose
Ice pack 24-48 hour

1. state 3 abnormalities

Chemosis, conjunctival swelling, redness, watery eyes, sclera show, thyroid stare, periorbital
swelling

2. 3 ocular signs

Lid lag sign, Kocher sign, EOM restricted, visual field changes

3. one relevant blood investigation

Thyroid function test

4. Provisional diagnosis

Thyroid eye disease

2. 2 major ocular complications

Exposure keratopathy/corneal ulcer, secondary glaucoma, compressive optic nerve

228
229 Oceanic Series End of Posting Compilation

Q1 ocular changes on fundus


Q2 risk factor?
Q3 pathogenesis?
Q4 diagnosis – wet age-related macular
degeneration
Q5 specific ocular examination

Q1 ocular changes on fundus

Q2 clinical manifestation

Left tractional retinal detachement, left exudative


retinal detachment

Q3 treatment

Q4 Diagnosis Left rhegmatogenous retinal


detachment

bilateral congenital glaucoma

Q1 based on photo in medical term

blepharospasm, photophobia, epiphora

Q2 3 findings

Q3 treatment medicine and surgery

229
230 Oceanic Series End of Posting Compilation

A 25 year old lady presented with history of decreased right eye vision for 5 days. On
examination, her right visual acuity is counting fingers. The exhibit shows the anterior segment
photo of her right eye.

1. State three clinical findings from the photo


i. Watery eye
ii. Eyelashes sticky to each other
iii. Hypopyon
iv. Corneal opacity
v. Hazy pupil
vi. Inflamed/injected conjunctiva with white patches (if inflamed only:
subconjunctival haemorrhage)

2. State two risk factors


i. Trauma with vegetative material
ii. Patient taking steroid eye drop

3. State your provisional diagnosis


Corneal ulcer

4. List two investigation to help your diagnosis


i. Corneal scrapping

230
231 Oceanic Series End of Posting Compilation

ii. Corneal c&s


iii. Aqueous tap

5. List three measures to manage this patient


i. Admit patient
ii. Give topical antibiotic
iii. Never give steroids
iv. Cycloplegic (atropine)

A 61 years old male presented with loss of vision in his left eye that began yesterday morning
and has progressively worsen

1. State two ocular changes on the fundus


i. Horse-shoe tear
ii. Subretinal fluid accumulation

2. State other two manifestations


i. Failing visual acuity
ii. Presence of flloaters
Iii. curtain-like view

231
232 Oceanic Series End of Posting Compilation

3. State two risk factors to have this problem


i. High myopia
ii. Lattice degeneration
iii. Family history

4. List two treatment


i. Cryocoagulation/photocoagulation
ii. Scleral buckling
iii. Pars plana vitrectomy (PPV)

5. State your diagnosis


Superior bullous rhegmatogenous retinal detachme

A 19 years old woman presented to the hospital with complaints of progressive


shortness of breath with manifestation of butterfly rash on her face. She also
complained of eye dryness. Kerato-conjunctivitis sicca

1. State one ocular manifestation in the picture


Corneal haziness

2. State the systemic disease experienced by this patient


Systemic lupus erythematosus
Acute cutaneous lupus erythematosus (explained dlm derma)

232
233 Oceanic Series End of Posting Compilation

3. State the drug that causes the ocular manifestation


Steroids

4. State other systemic side effects of the drug mentioned


i. GI upset
ii. Osteoporosis
iii. Weight gain
iv. Elevated blood sugar / blood pressure

5. State the surgical management for this patient


Phacoemulsification (cataract surgery)

6. State two complication of above surgery


i. Stitch abscess
ii. Corneal oedema
iii. Iris trauma
iv. Endophthalmitis
v. Retinal detachment
vi. Hyphaema

You are provided with a photograph of a six-months-old boy.

a. State abnormalities seen.

233
234 Oceanic Series End of Posting Compilation

white patch in the centre of the left eye

b. List three questions you would like to ask to help in your diagnosis.

how about her kid’s vision (expect has poor vision/ irritability)?

does any in her family has the same problem?

does the kid has any other medical condition (non ocular tumor- osteosarcoma,
melanoma…)
Investigation was done. This picture illustrates its finding.

a. Name the above investigation.

B-scan / Ultrasound

b. State another important imaging modality for this condition and its reason.

CT scan / MRI to assess extent of tumor invasion involving optic nerve

c. State your provisional diagnosis.

Retinoblastoma

d. List two other differential diagnoses for this condition.

congenital cataract

congenital glaucoma

234
235 Oceanic Series End of Posting Compilation

e. State the histopathological feature of the disease.


1. composed of small basophilic cells with large hyperchromic nuclei and scanty
cytoplasm many undefferentiated retinoblastoma
2. if differentiated forms rosettes (flexner wintersteiner, homer wright, fluorettes)

clinical presentations
leukocoria
strabismus
painful red eye
bupthalmos
iris nodule
secondary glaucoma
poor vision
orbital invasion

tx
chemotherapy
brachytherapy
enucleation

The baby was treated successfully through medical and surgical means. However at
the age of 16 years old, he has a right thigh progressive swelling for six months.
f. State the most probable cause of this swelling.

Secondary tumour (soft tissue sarcoma)

235
236 Oceanic Series End of Posting Compilation

This is the fundal picture of a 30-year-old lady who had blurring of vision associated
with pain on ocular movement in the left eye for one week. The right eye was normal.
(in this case, it is optic neuritis not papilledema because it’s unilateral meanwhile
papilledema tend to be bilateral)

a. List 2 abnormal findings.


fundoscopy : flamed shape hemorrhage
optic disc is edematous and hyperaemic
blurring of optic disc margin
retinal veins are congested and tortuous

b. List any 4 ocular examinations (other than mentioned answer in (a)) you would like to
perform and its expected findings.

visual acuity- decreased visual acuity

pupillary reflex - RAPD positive

coloured vision (ishihara chart) - impaired

slit lamp biomicroscopy- reveal inflammatory changes in the vitreous

236
237 Oceanic Series End of Posting Compilation

This patient was admitted to Ophthalmology Ward and was given regular intravenous
medication for three days and was monitored for the side effects.
c. Name the most appropriate drug given to her.
treat the causes (sebab usually pt typical yang kena ni ada MS jugak)
IV Methylprednisolone (iv steroid)
interferon therapy for MS

d. List any 2 side effects of this drug.

weight gain

nausea and vomitting

hyperglycemia

She defaulted follow up for a year and came back with hand movement vision. The
left optic disc noted to be pale.
e. List any 2 possible causes of pale optic disc.

CRVO/CRAO/Advanced glaucoma
arteritic anterior ischemic optic neuropathy

237
238 Oceanic Series End of Posting Compilation

A 41-year-old male presented with a chief complaint of blurring of vision, redness and
floaters in his left eye that had persisted for 3 weeks. His medical history was remarkable for
type 2 diabetes, which was controlled with oral medication. His best corrected visual acuity
was 20/200 for left eye. IOP for right and left eye was 12 mmHg and 24 mmHg respectively.
Anterior chamber of the left eye was deep with cells seen. Fundus photography was taken.

Describe 2 abnormalities seen in the retina.


-juxtapapillary lesion
-macula inflammatory lesion

List 2 information in the history you would like to ask to determine the cause.
• ada demam tak
• ada bela kucing tak
• control diabetes tak

An aqueous tap was done and sent for microbiological test.

c. Name the stain for this microbiological test.

Giemsa stain

d. Identify the organism and its stage (life cycle).

Toxoplasma gondii (trophozoite)

e. State your provisional diagnosis.

Ocular toxoplasmosis

f. Name the definitive host of this organism.

Feline (cats)

State the cause of high IOP of the left eye.

Uveitis causing trabeculitis and block pores, thus obstructing aqueous flow

238
239 Oceanic Series End of Posting Compilation

2 indications for treatment.

1. immunocompromised

2. develop life threatening disease like uveitis

Name one first line treatment you would prescribe to the patient.

pyrimethamine and sulfadiazine plus corticosteroid

This is the optic nerve head picture of a 30-year-old lady who had blurring of vision

associated with pain on ocular movement in the left eye for one week. The right eye was

normal. (In this case, it is optic neuritis not papilledema because it’s unilateral meanwhile

papilledema tend to be bilateral) (malignant hypertension and SOL don’t have blurring of

vision)

a) List two abnormal findings. (2 marks)

Optic disc is swollen / loss of cup-disc ratio / Blurred optic disc margin

Retinal veins are congested and tortuous

b) List any 4 ocular examinations (other than mentioned answer in (a) you would like to

perform and its expected findings. (4 marks)

239
240 Oceanic Series End of Posting Compilation

Visual acuity - reduced

Visual field – reduced

Light contrast – reduced

EOM – pain on ocular movement

Colored vision – impaired

This patient was admitted to Ophthalmology Ward and was given regular intravenous

medication for three days and was monitored for the side effects.

c) Name the most appropriate drug given to her. (1 mark)

IV Methylprednisolone

d) List any 2 side effects of this drug. (1 mark)

Weight gain

Facial puffiness

Skin acne

Gastritis

Flushing

She defaulted follow up for a year and came back with hand movement vision. The left

optic disc noted to be pale.

e) List two other possible causes of pale optic disc. (2 marks)

CRAO / traumatic ischemic neuropathy / retinitis pigmentosa

Anterior ischemic neuropathy (AION)

Advanced glaucoma

CRVO (less common)

240
241 Oceanic Series End of Posting Compilation

st
Oceanic Question Bank 1 Edition
PSYCHIATRY

241
242 Oceanic Series End of Posting Compilation

EMQ
1. 23-year-old Newly enrolled college student was anxious Separation anxiety
about living in the campus, as this was his first time away
from his parents.
2. A 38-year-old single woman presented with breast H. Risperidone
abscess. Prior to that she has a full breast engorgement
for the past one month.
3. A 30-year-old man with schizophrenia complaint of sore E. Clozapine
throat with fever for one
week. His white blood count was 3000/mm³.
4. Medical students narrated: “What happened today that Open-ended
brought you to the hospital? questioning
What else have you experienced?”.
5. Medical students narrated: “Oh you lost your job about 3 Linking
months ago which led you
to your money problem. Since then, you became
distressed and started to worry
about your future”.
6. A 53 years old woman complaint of dyspareunia after her Lubricant - KY Jelly
menopause. She had no
other menopausal symptoms
7. A 38 years old man was feeling down with a loss of libido C. CBT
after he had lost his job
following the COVID-19 pandemic.
8. An 11 years old girl was abused by her stepfather on Child Protection
several occasions. Both of her Procedure
parents were taken under custody by the police. She was
then placed temporarily at
a welfare home.
9. A 14 years old girl was molested by her maternal uncle. Interim Protection
The uncle was taken into police Order

242
243 Oceanic Series End of Posting Compilation

custody and subsequently was bailed out. He was


refrained from visiting the girl and
her family.
10. A 25 years old emergency room doctor working on night Circadian-Rhythm-
shift for the past one month. Sleep-Wake Disorders
He did not feel refreshed after he sleeps and was
experiencing excessive sleeping
when working.
11. A 20 years student was prescribed Ritalin Hypersomnolence
(Methylphenidate) to improve his Disorder (Narcolepsy)
concentration in class.
12. MOA of olanzapine dopamine d2 and
serotonin type 2
(5HT2) antagonists
13. Moa Donepezil Acetylcholinesterase
inhibitors
14. A lady who is seductive, attention seeker and displays Histrionic
temper, tantrum when fail to get
attention:
15. 4. A lady found to have bruises on her body when visit a Dependence
doctor due to health problem, it was
due to her husband abused her however she still didn't
want to make a report and keep on
saying her husband cares and loves her:
16. 56-year-old woman is irritable and insomnia for the past Hormonal therapy
6 months. Experiencing
dyspareunia associated with vaginal dryness.
17. Primary ejaculation. still have problem after do SSRI
intervention given by doctor.
18. A 21 years old woman went to eat 3 box of cereals and Bulimia nervosa
8 chocolates bar. She felt guilty
after eating too much and vomited. Her BMI 25.

243
244 Oceanic Series End of Posting Compilation

19. 9y/o girl with mild mental retardation. Presented to clinic PICA
with abdominal colic anaemia.
Microscopic result shows numerous ova of roundworms.
20. A 30 years old schizophrenic man was found guilty for a
murder. He was found unguilty
because of unsound mind. He was found unguilty
because of insanity.
21. 10. 70-year-old woman received property from deceased Power of attorney
husband. She is aware of that and
ask family lawyer to manage.
22. A 40-year-old unemployed man brought to psychiatric schizo is associated
clinic for the first time. which of the social with high
statement is incorrect socioeconomic status
23. A person with mental illness usually experiences A person experiences
psychological phenomenon. Which of depersonalization
the phenomenon is incorrect believes that he/ she
is physically detached
from the outside
world
24. Female with depression has underlying SLE - Depressive due to
Depressive due to medical condition. medical condition

25. 21 years old salesgirl stop working because feeling


anxious in the supermarket and taking public transport. Agoraphobia

26. A 40 years old businessman was apprehensive and can’t Generalized Anxiety
do his work. He felt giddiness and abdominal discomfort Disorder (GAD)
at all time.

27. Drug of choice for bipolar disorder with depressive Lamotrigine


phase.

244
245 Oceanic Series End of Posting Compilation

28. 40 years old female , irritable and anxious with unknown Hyperthyroidism
reason and cant comb his hair.

29. Students been offered tablets by her friends for study Amphetamine
week for upcoming examination.

30. Change from smoking cigarettes to vaping. Nicotine

31. 8 years old boy with slow learning, IQ 80. L.Wechsler for
Children / Norm-
referenced test

32. 30 years old man with history of self-harm on many Minnesota


occasions multiphasic
personality inventory

33. 40 years old teacher presented with depression after Bereavement therapy
death of her husband 6 months ago. She was given anti
depression and her mood become alleviated.

34. Child with pica comes with her parents. Through mirror Family therapy
observation by the psychiatrist, the parents were
arguing.

35. 40 years old single man, no intention to get married, no Homosexual


sexual attraction towards opposite gender.

36. 20 years old lady, anxious on her wedding night, was Vaginismus
tensed when husband tried to make love to her.

245
246 Oceanic Series End of Posting Compilation

37. 13 years old boy, always arguing with his parents Conduct disorder
,disobey his parents' instructions and always involved in
misdemeanour at school

38. 8 years old girl complains of abdominal discomfort every School refusal
time she goes to school and will be very distress about
it. She always throw a tantrum.

39. 40 years old man, stupors, do not eat anything ECT

40. A 60 years old man presented with loss of memory. He Biochemical


was found wandering alone. He was not able to answer investigation

question during interview session

41. A 30 year old lady is presenting with recurrent headaches


Systemic lupus
for 2 years. She was depressed for the last 2 months. erythematous
She also developed rash in her face and pain at the joint
of her fingers

42. 20 year old man smells odd smells and act abnormally. Temporal lobe
epilepsy

43. A 16 year old boy presented to A&E department in Opioid intoxication


euphoria with aggressive behaviours. On examination
there is no abnormalities found.

44. A 30 year old man presented with rhinorrhoea and body Opioid withdrawal
ache. On examination, his blood pressure is 140/90
mmHg and pulse is 100/min.

45. A 24 year old medical student wanted to have more Amphethamine


energy to study late night. He seeks
advice from a friend.

246
247 Oceanic Series End of Posting Compilation

46. A 30 year old man smoked cigarette for 15 years decided Nicotine
to quit cigarette and start vaping. He was able to quit
cigarette without withdrawal effects

47. A 15 year old student scored poorly in his academic Aptitude test
standing. He was noted to have difficulty in learning
Science and Mathematics but skilful in handyworks.

48. A 20 year old man was reported to behave abnormally Direct observation
for one week. He is not forthcoming during the interview test

49. A 30 year old man was reported to harbour a paranoid Psychodynamic


ideation towards his colleagues. He remained a loner therapy

and refused to get closer to anyone in the office. He


was directed to see a psychiatrist for intervention.

50. A 35 year old lady presented with acute anxiety in the Behavioral
place where the Halloween celebration was on. She felt modification
distressed with the crowd and subsequently she was
afraid to go out at night.

51. A 25 year old man was apprehended by security guards Voyeurism


after they caught him hiding in and peeking in the ladies
toilet.

52. An 18 year old student has a habit of collecting ladies Fetishism


undergarment. He confessed that he like to smell panties
to increase his arousal

53. A 10 year old boy presented with inability to socialize


Autism spectrum
than usual. He remains aloof in his own world and was disorder
not able to interact with his cousin.

247
248 Oceanic Series End of Posting Compilation

54. A 9-year-old girl was noted to be slow in learning. She Dyslexia


was not able to read a paragraph and have difficulty in
writing sentences.

55. A 30-year-old schizophrenic has episodes of hurting Put in a safe isolated


himself in the ward. He stood in a corner and banged room
his head against the wall

56. A 25 years old man, handcuffed, was agitated towards Aggressive risk
the policemen. He appeared hostile towards the staffs at behaviour
assessment
the clinic.

57. This is the drug of choice for treatment of anxiety with Lorazepam
depressive features.

58. This is the drug of choice for negative symptoms in Clozapine


schizophrenia.
59. A 19 year old student finds it difficult to control hisObsessive
recurrent blasphemous thoughts. So for weeks he had to compulsive disorder
recite a special prayer.
60. A 38 year old lady presented with several episodes of Panic disorder
palpitation and shortness of breath after her mother’s
death one month ago
61. 65 year old man presented with personality change and Neurosyphilis
ataxia for 6 months. He experienced episodes of
headache with dizziness. On exam of eyes, he had
Argyll-Robertson pupils
62. A 60 years old man had depression for 1 year. His friend Thiamine deficiency
noticed he had personality changes.He had loss of
memory
63. 30 years old man needed larger amount opiod with time. Opioid use disorder
He had significant impairment of his social function. He
attempted to cut down usage but was unsuccessful.

248
249 Oceanic Series End of Posting Compilation

64. Patient has depressed mood and suicidal ideation Opioid induced mood
disorder
65. A 26 year old medical student wants to have high mental Caffeine
alertness. He takes high amount of this substance. Then
he experienced headache, palpitation, heartburn
66. A 30 year old man started to feel energetic and active Kratom/ketum
after taking this substance
67. A 15years old student was scored poorly throughout his Achievement Test
academic year. His intelligent quotient was within normal
68. A 25 years old man undergo personality test and the Eysenck test
result shows psychotism, extrovernism, and narcoticism
69. Her mother passed away 12 months ago, patient been Bereavement Therapy
having depressed symptoms for 8 months. What is the
suitable therapy.
70. A 28 year old woman break up with her boyfriend for 8 Interpersonal therapy
months and she does not want to fall in love again
71. 20 years old man feeling dysphoric since adolescent. He Transgenderism
does not like his gender. He dresses like a girl
72. 18y/o girl has intimate feeling towards an older woman Lesbianism
who is her physical instructor
73. A 5 year old boy presented with acute abdominal pain in Pica
emergency room. Abdominal x ray shows a large opaque
image in stomach
74. 13 year old girl who refused to listen to her teacher and Conduct Disorder
verbally aggressive towards her (If oppositional , age <
13y/o)
75. A 30 year old schizophrenic had episodes of verbal Aggressive risk
aggressive in the ward. He threatened to physically assessment
assault anyone who come close to him.
76. 25 year old patient who is angry and shouting to other Calm the behaviour
patient that disturbed his belonging
77. Drug of choice for anxiety with depression Lorazepam

78. Drug of choice for treatment of positive symptoms of Haloperidol


chronic schizophrenia

249
250 Oceanic Series End of Posting Compilation

79. Young girl, weak proximal hip muscle, have anxiety Hyperthyroidism

80. 28 year old has nightmares about an accident which PTSD


caused her mom died 6 month ago. She also
experienced anxiety when she pass through the road
81. Thyroid Function Test necessary to do after prescribing Lithium
this drug
82. 45 years old lady was admitted for acute mania due to ECT
non-compliance to treatment. She was noted to be very
agitated, hostile and violent even after a week of
pharmacotherapy.
83. An additional form of therapy that may be helpful in Psychoeducation
improving her insight and compliance to medication in
order to prevent relapses of illnes
84. A 13 year-old girl has been enrolled in a boarding Adjustment disorder
school one month ago. She has not been sleeping well
at night. She has not been able to focus in her class.

85. Molested by cousin, after returning home mother noticed Acute stress disorder
crying, change in behaviour
86. A 26 year-old single female teacher feels insecure and Dependent
not able to make her own decision. She requires constant Personality
reassurance for all her undertakings
87. 25 year-old lady had always been a timid person. She Avoidant Personality
had the fear of being criticized by public and afraid to
socialized because she feels herself inadequate
88. 70 year-old man presented with bradykinesia, resting Parkinson’s Disease
tremor, rigidity and problem with gait; that progressively Dementia

got worse over the last 8 years. Recently, he developed


impaired memory with poor verbal fluency and executive
dysfunction.
89. 62 year-old lady presented with progressive lost of short Alzheimer Dementia
term memory over the last five years, resulting in
impaired activities of daily living. She was not known to

250
251 Oceanic Series End of Posting Compilation

have any medical problems. Her mother and one of her


sister had similar problem.
90. Mr. Tan was admitted for septicemia, in ward he was Illusion
noted to be afraid and restless because he mistook IV
drip tube for a snake on his bed.

91. Mr. Tan was regularly given his medication at 8 am. Memory deficit
During morning ward round, he complained to his doctor
nurses did not give him medication and foods at all
92. 27y/o man fall from tree due to him seeing angel on the Mania
top of tree and want to meet him there. Engage with
many activities but failed to finish any task. Treated for
similar symptoms before
93. 23y/o soldier says he has hearing loss. On ENT Malingering
examination here was no finding. His condition
drastically improved after his posting to war was
cancelled.
94. A novel antipsychotic was given to a college student. The Olanzapine
medication was stop because he later developed weight
gain and diabetes mellitus
95. A 25 years-old man newly diagnosed with schizophrenia Haloperidol
was given rapid neuroleptization for his aggressive
behavior. The next day he developed muscle rigidity,
high fever and confusion.
96. A previously healthy 50 year-old bank manager Frontal lobe tumour
developed personality changes which include apathy,
lack of motivation and spontaneity. The laboratory
investigations of urine and blood were normal.

97. A 65 year-old lady admitted for multiple medical Syndrome of


illnesses. She was noted to be depressed and she was Inappropriate
Aldosterone Hormone
prescribed a SSRI. Following that her blood sodium was
(SIADH)

251
252 Oceanic Series End of Posting Compilation

found to be low and she was advised to go on with fluid


restriction.
98. A 40 year old man presents at the emergency Alprazolam
department with a sudden onset of intense anxiety
symptoms. The doctor prescribed him a drug which
advised to be used only when necessary as it has the
highest risk of causing dependence.
99. Anti-anxiety for repetitive hand behaviour SSRI

100. This drug should be stopped at least two days before Lithium carbonate
giving Alectroconvulsive Therapy (ECT) as it may cause
‘toxic delirium’
101. This drug has long half-life and it should be avoided Diazepam
during Electroconvulsive Therapy (ECT) as it may
increase the seizure threshold.

102. A 42 years old man was reported to behave abnormally Bipolar Mood
disorder
for last 10 days. He slept only for 1- 2 hours at night. He
had been painting a lot in 1 week, no remarkable award
before
.
103. Post Traumatic Stress
A 28 years old soldier just returned from his services Disorder
war at Afghanistan 3 months ago. He presented with
agitation. He had intrusive thought and recollect back
memory from the war. He was unable to sleep and
became withdrawn from his family.

104. 75-year-old man was prescribed with haloperidol for Parkinsonism


hallucination 3 weeks ago. He presented with tremors,
slow, shuffling gait and drooling of saliva.
105. A 40-year-old man was started on Haloperidol and he Akathisia
presented with restlessness. He was unable to sit still in
one place.

252
253 Oceanic Series End of Posting Compilation

106. 19 years old student had just started university. She wasSocial anxiety
a shy person and rarely went out with friends and rather disorder
prefer to spend time alone at home. She had to drink
alcohol whenever she has to go out with friends
107. This disorder present with symptoms of anxiety in crowd Agoraphobia
places.
108. A 32 years old man was highly anxious and his hands Pheochromocytoma
feel shaky. He thought he was having a heart attack. He
was sweating and felt pins and needles in his extremities.
He has tachycardia and hypertension.
109. 52 years old housewife complaint of frequent episodes of Hypoglycaemia
palpitation, tremor, sweating. She became irritable at
times.
110. 29 years old woman with a history of childhood abuse Personality disorder
presented with multiple old scars on her left forearm.
Recently, she cut her hand using a razor after her
boyfriend decided to break up with her.
111. A 42 years old man presented to emergency department Substance induced
with recent scars on his arm. He had no past psychiatric psychosis
history. He felt like sensation of bugs crawling in the skin
which he then tried to remove with the knife.
112. A 68 years old woman with breast cancer metastasize to Give naloxone
bone. She was prescribed with paracetamol, oral
morphine, and gabapentin. She was brought to ED with
pinpoint pupils, respiratory distress and palpitation.
113. A 27-year-old woman left a suicide note. She was found Involuntary
at a train station trying to jump in front of an oncoming admission
train. During interview, she seems depressed and tearful.
Her parents refused hospitalization
114. 27 years old man have no remorse for his old action. Antisocial personality
Psychotherapies are not effective in this personality
disorder

253
254 Oceanic Series End of Posting Compilation

115. A 25 years old lady have unstable relationship and Borderline


frequent suicide attempt. This disorder is best treated Personality Disorder

with combined individual and group psychotheraphy


116. A 27 years old man desire to be a woman. He loves to Gender identity
dress as a woman since teenager. He have intimate disorder

relationships exclusively with men


117. 23 years old female experienced pelvic pain whenever Functional
her husband tried to make love with her. No dyspareunia
abnormalities found during pelvic examination.
118. Neurofibrillary tangles and plaque are pathological Alzheimer
finding in brain in post mortem of this disorder
119. The neurotransmitter that are most affected in this Major Depressive
disorder are 5HT and noradrenaline neurotransmitter Disorder

120.

A 23 years old woman often regrets after binge eating


Bulimia nervosa
and vomiting immediately after eating.
121. A 8 years old child with abnormal diet intake. She was
Restrictive food
snacking instead. She was found to have nutritional intake disorder
deficiency
122. 21 years old male with sudden paralysis multiple times Conversion Disorder
but spontaneously resolved after 24 hours. Lab
investigations are normal.
123. Fearful of making mistakes and multiple times checking OCD
her work. This had affected her
124. 23 years old male presented with shortness of breath and Agranulocytosis
hypotension. Has pneumonia and
low neutrophil count

254
255 Oceanic Series End of Posting Compilation

125. 40 years old man was given increase dose of Acute dystonia
fluphenazine. He developed sudden onset of neck
extension, jaw clamped and eyes forced upward
126. 30 years old man cannot focused at work. He has Voyeuristic disorder
thought of having sexual with his female collegues. He
feels shameful about it.
127. Presented with shortness of breath and palpitation. Has Panic disorder
family history of myocardial
infarction
128. Hand shaking when trying to do things. This resolved Benign essential
when she jogging. tremor

129. 64 years old man has increase shaking of the hand. His Parkinson Disease
right hand seems worse than his
left. Mumble and rarely smile.

130. 24 years old woman diagnosed with drug induced Schizophrenia


psychosis one year ago. After receiving
treatment, she still experience auditory hallucinations.
This caused her to feel distressed
and want to kill herself. She denied taking drug.
131. A man overdosed of 38 tablets of paracetamol and MDD
alcohol intoxication. Wife noted that he
was feeling low recently. Liver function test was normal.
132. 28 years old woman contemplated to jump out but called Discharge with
her parents for help. No previous history of self harm. community homecare
within 48 hours
133. Discharge with community homecare within 48 hours Give N-
acetylcysteine
134. Neurofibrillary tangles found on the post mortem Alzheimer’s disease
examination.
135. Neurotransmitter problem with noradrenaline and 5-HT Major Depression
receptor
136. 25 year old girl has chronic feeling of emptiness, low self Borderline
esteem, unstable self image and personality disorder

255
256 Oceanic Series End of Posting Compilation

impulsivity for past months

137. 40 years old manager unreasonably demands his Narcissistic


subordinates to create a successful working place. He personality disorder
exaggerates his achievements and wants the others to
acknowledge his talent
138. Dress in woman clothes for masturbation and sexual Transvertisism
arousal purpose
139. A man liked to dress in woman’s clothes since Gender identity
adolescent. disorder

140. Teacher report that a child usually makes noise, but Tourette syndrome
polite at school.
141. A child with repetitive behaviour and lack of social Autism spectrum
interaction. disorder

142. 32 years old singer feeling hyperactive, sweating and fast Methamphetamine
breathing. Feels great about himself and high sexual
load.
143. 18 years old lady restricted taking food and taking Anorexia nervosa
laxatives. Weight 33kg and height 1.53m
144. 40 years old lady depression due tu joint pain and SLE
swelling for 2 months. Pale, Raynauld's syndrome
positive
145. 30 years old lady with history of rape a year ago. Develop PTSD
fears and avoid the place
146. 25 years old man presented with arching of the back after Acute dystonia
ingesting medication
147. 30 years old man developed hyperpyrexia & muscle Neuroleptic Malignant
rigidity after been given parenteral antipsychotic. Syndrome)

148. 13 years old given a series of cards and asked to Roschach Test
interpret the meaning
149. Man adds many unnecessary details when asked about Circumstantiality
his address
150. Student is presenting his thesis suddenly stopped and Thought block
cannot continue

256
257 Oceanic Series End of Posting Compilation

151. Woman with depression said his constipation is Somatic delusion


because her intestines are rotting
152. Man looked himself at the mirror claims his hands and
Bizarre
other body part are distorted or something delusion/Nihilistic
delusion
153. 35-year-old man have few friends. He always had Paranoid delusion
unusual belief and perception of his friends talking about
him
154. 15 years old likes to play truant at school and goes out anti-social pd
late at night. He is verbally abusive towards his parents
and frequently gets into fights with his peers. What
personality disorder is most likely going to develop when
he is an adult?
155. 35 years old man explained to his therapist that he is masochism
sexually aroused when his partner inflicts pain him
156. 19yo male caught masturbating while watching female voyeuristic
showering
157. A 33-year-old man feeling deterioration in his self- Family
esteem due to feeling inadequate as a husband to his Therapy/Interpersonal
therapy
wife. He also has problem getting along with his wife.
158. Mark kawen dgn sarah 6 thun. Baru2 ni Mark kne byk
keje n kene balek lambat. Sarah nk mark balek awal n
Family therapy
luang mase jaga anak ini buatkn mereka sllu bertekak.
Sarah tk puas hati then ugut mark nk lari bawak anak
159. this disorder is commonly seen in pregnancy Binge Eating/Obesity

160. this disorder commonly seen with abuse of laxatives Anorexia nervosa
(purging type)
161. Withdrawal causes delirium Alcohol

162. Substances commonly abused by woman Benzodiazepines

163. 50 y/o had developed headache d/t the fall. Hx of right Post-stroke
sided leg weakness before. Currently presentèd with Depression

irritability and depressed after the eps

257
258 Oceanic Series End of Posting Compilation

164. 5 y/o male, noticed to have a sudden blank stare while Petit mal epilepsy
talking to his friends. He claimed of feeling strange before
becoming unaware of the surrounding
165. 21 y/o student did psychological test and was identified MMPI
with hypochondriasis tendency
166. 50-year-old distracted, unable to concentrate during direct observation
interview test/ MMSE

167. This drug have a patch formed. Rivastigmine

168. This medication is used in Behavioural and Memantine


Psychological symptoms of Dementia (BPSD)
169. 16 years old student, accompanied by his mother went Reassurance
to psychiatric clinic. During the interview, she refused to
talk to the doctor in the presence of his mother.
170. 40 years old lady denied having emotional disturbance Persuasion
despite of seeing doctor frequently with complaints of
having bodily distress. She’s reluctant to tell the Dr
regarding her marital problems for the past 6 months.
171. 45 years old man with fluctuating but persistent sadness Persistent depressive
for the past 3 years. However he still can do his normal disorder (Dysthymia)

activities.
172. 22 years old boy with history of depression for 3 months. Bipolar depression
During his late teens, he has history of involve in reckless phase
behaviour and increase self-esteem.
173. 21 years old female stopped working feel extremely Agoraphobia
anxious when going to supermarket and taking public
transport.
174. 25 years old. Feared of being in a company with Social anxiety
unfamiliar people. Fear of being ridiculed by others. disorder

175. 45 years old lady with episodes of irritability for 1 month. Hyperthyroid
Complaint of lose weight, poor sleep and easily anxious
without apparent reason. Has difficulty in comb hair.

258
259 Oceanic Series End of Posting Compilation

176. A 60 years old guy experience change of personality. He Frontal lobe tumour
was being ill mannered at eating table and talk
demeaneringly of his wife. And he complains of frequent
headache
177. 25 years old medical student always go to club, and Amphetamine
always use substance to stay awake to study
178. Abrupt cessation of this substance will cause delirium Alcohol

179. 12 years old boy was noted to be slow in learning. His Achievement Test
intelligent quotient (IQ) was found to be 80.
180. 30 years old man with deliberate self-harm. Minnesota
Multiphasic
Personality Inventory
(MMPI)
181. A 40 years old teacher presented with depression, 4 Bereavement therapy
months ago husband passed away. She was given anti-
depression and her depressed mood become alleviated.
182. A 5 years old referred for nocturnal enuresis. Both Family therapy
parents present for interview. On observation through
one-way mirror, doctor realized father is verbally abusive
towards wife who appeared fearful.
183. 13 years old teenage boy always against his parents. He Conduct disorder
constantly disobey instructions at school.
184. 40 years old woman come to ED with stupor and refuse ECT
to eat. She was given IV fluid. Physical and blood
investigation was normal. What to do?
185. 60 years old man presented with loss of memory. He was Admit to emergency
found wandering in the street. He could not answer the ward, do investigation
first
questions during the interview session. His general
physical examination is normal.
186. Next day I took control.... oh no! I put bleach on my hair loss of association

187. 30 violence neighbor want to steal money paranoid


schizophrenia

259
260 Oceanic Series End of Posting Compilation

188. Walking endlessly and giggling to herself disorganized


schizophrenia

189. Went out with colleague but afraid they might not like her avoidant personality
disorder

190. 30 yo secretary distressed about her project not OCD


completed thinks format is wrong, review
it 6 times, have occurred before and affecting her work
191. A boy repetitive eye blinking and head jerking Tourette

192. 7 yo nausea and vomiting before going to school afraid separation


mother left the house anxiety disorder

193. Easily aroused by sniffing panties Frotteurism

194. 21yo man bradycardia and pin point pupil heroin

195. A boy who has slurred speech after taking this sniffing glue

196. SSRI that have prolong QT citalopram

197. SSRI that have long half life hence least likely to cause Fluoxatine
discontinuation withdrawal

260
261 Oceanic Series End of Posting Compilation

MEQ
MEQ 1

A 40 years old male have irritable mood the for the past week. He buys a lot of birds stuff.
Complaint the shop keeper charges unreasonable. He set fire to the shop that night.

1. List 6 symptoms to make diagnosis

• Distractibility

• Indiscretion

• Grandisosity

• Flight of ideas

• Activity

• Sleep decrease

• Talkative

2. 2 pharmacotherapies to calm patient down

3. Why do we need to admit this patient?

261
262 Oceanic Series End of Posting Compilation

His family request to discharge him early. He was given Lithium 600 mg and later developed
vomiting, tremors, confusion, coma

4. Explain why the patient become like that

Lithium toxicity
5. Biological investigation need to be done based on above situation

Renal fx test
Serum Lithium level
6. Other 2 drugs that can prevent relapse

Carbamazepine
Sodium valproate
Olanzapine
Risperidone

262
263 Oceanic Series End of Posting Compilation

MEQ 2

A 38 year old housewife presented with history of loss appetite, low energy and tearful, waking
early in morning and neglecting her children. She developed symptoms for the last 6 months
since learning her husband’s infidelity

1. State 2 possible diagnoses

I. Major depressive disorder


II. Adjustment disorder with depressed mood

2. State the most likely diagnosis with reason


Adjustment disorder with depressed mood because there is identifiable
stressor

Outline 2 initial managements for this patient

i. Antidepressants
ii. Investigations

The patient had no significant improvement after oral antidepressant therapy for 2 weeks, rather

developed worsening of symptoms and suicidal ideation.

3. Plan 2 next managements for this patient

i. Admit into the ward


ii. Change medications
iii. Plan for ECT

263
264 Oceanic Series End of Posting Compilation

4. State 2 commonly used antidepressants with side effects

i. Fluoxetine - Nausea, vomiting , diarrhea


ii. Imipramine - Cardiotoxic effect

5. List 2 important investigations to be done

i. Full blood count


ii. BUSE
iii. ECG
iv. Thyroid function test
v. Renal profile

6. State 2 common adverse effects of electroconvulsive therapy (ECT)

i. Brief retrograde amnesia


ii. Nausea

7. List 4 other psychological interventions you would offer for this patient

i. Psychoeducation
ii. Marital counselling
iii. Cognitive behavioural therapy
iv. Supportive therapy

264
265 Oceanic Series End of Posting Compilation

MEQ 3

A 25 year old housewife came in with recurrent blasphemous thoughts in her mind that are
intrusive and impaired in her daily activities. She was then convinced that God angry at her and
increased her religious activities but unable to overcome her problem.

1. Write 4 common themes associated with this disorder. (4m)


i. Hygiene
ii. Religion
iii. Sexual
iv. Orderliness

2. Mention 4 common features of this disorder


i. Obsession thoughts
ii. Obsession doubts
iii. Impulses
iv. Rumination

3. State the most likely diagnosis with reason.

i. Obsession compulsive disorder because presence of recurrent intrusive


thoughts about blasphemous ideation.

265
266 Oceanic Series End of Posting Compilation

After that evaluation, the patient complained of being stressed and then she became depressed.
She had persistent low mood, poor appetite and sleep disturbance.

4. Give 2 possible complications

i. Depression
ii. Working problem
iii. Reduced level of functioning

5. Name 2 drugs used for this patient

i. SSRI - Fluoxetine
ii. TCA - Clomipramine

6. The patient did not respond well after 2 weeks of pharmacotherapy. Give 6 reasons for
why this patient are not responding to pharmacotherapy.

i. Poor compliance
ii. Poor insight
iii. Inadequate dose
iv. Wrong medication
v. Misdiagnosis
vi. Side effect of medication

266
267 Oceanic Series End of Posting Compilation

MEQ 4

40 /o, hostile, restlessness for x2/52. Believes he’s the savior of the human kind

1. Ddx and reasons

i. Manic
ii. Brief psychotic disorder (short duration)

2. Other relevant symptoms

MEQ 5

A 32 years old housewife was referred due to anxiety and fearful of contamination with
germs.She was reportedly spent hours in each day cleaning her kitchen wash basin and toilet
with strong detergent. You were asked to take a detail history from her to make a diagnosis.

A. Give 6 salient features of her condition

B. List 2 possible differential diagnosis

C. List 2 complications that she might develop

267
268 Oceanic Series End of Posting Compilation

D. Name 2 drug of choices and why

E. Name 2 behaviour therapies and explain how it is done

F. Explain 2 features to differentiate between OCD and Obsessive Personality

OCD - i) ego dystonic


ii) good insight

OPD- i) Ego Syntonic


ii) poor insight

MEQ 6

A 40 year old male bank officer was brought to the emergency department by his wife for
overdose of paracetamol and pain killer. On upon questioning, his wife reported that he was not
able to function well in bank and take many medical leaves since beginning of the year. His wife
also reported that he took alcohol excessively to combat depression. He is suspected to have
depression.

A. List 2 core symptoms of major depression.

i. Persistent low mood for the past two weeks worse in morning and better in
evening
ii. Anhedonia ( loss of interest in pleasurable activities)

268
269 Oceanic Series End of Posting Compilation

B. List 4 biological symptoms of major depression.

i. Appetite/ weight (kalau jwb utk weight kene precise loss>5% in one month)
ii. GIT features
iii. Sleep
iv. Libido/menstrual disturbances
v. Energy

C. 4 points to determine the severity for major depression

i. strong suicidal ideation


ii. duration of symptoms
iii. frequency of symptom
iv. psychotic features

D. Outline 3 immediate management at the emergency department

i. Stabilize the vital


ii. Treat the overdose
iii. refer to medical

F. the drug n reason

SSRI; less side effect

G. non-pharmacological management

i. crisis intervention

ii. CBT

iii. IPP
iv. supportive therapy/social support

269
270 Oceanic Series End of Posting Compilation

MEQ 7

A 32 years-old housewife was referred due to anxiety and fearful of contamination with germs.
She was reportedly spent hours in each day cleaning her kitchen, wash basin and toilet which
strong detergent. You were ask to take a detailed history from her to arrive at a dignosis.

1. List 2 possible differential diagnosis


Schizophrenia, OCD

2. Provisional diagnosis and give reason


OCD

3. 2 complications that she might develop


Depression, dermatitits, functional decline

4. Name 2 drugs of choice and the side effect


SSRI (fluvoxamine) ; GI disturbance headache
TCA (Imipramine); dry mouth

V. Name 2 behavior therapies


Practice relaxation therapy
Behaviour desensitization therapy (thought stopping)
Exposure & response prevention

270
271 Oceanic Series End of Posting Compilation

MEQ 8

40 years old woman complained of palpitations for one month. She was also easily irritated and
easily irritable towards her friends. Since then she worry about her health. There is no history of
heart disease, supplement or prescribed medications.

1. 2 provisional diagnosis and reasons for each.

• Hyperthyroidism
• Generalized Anxiety DIsorder (GAD)
• Panic Disorder (markah kesian)

2. 2 symptoms u would like to enquire for each diagnosis

Hyperthyroidism:

• Fatigue or muscle weakness


• Hand tremors
• Mood swings
• Nervousness or anxiety
• Rapid heartbeat
• Heart palpitations or irregular heartbeat
• Skin dryness
• Trouble sleeping

GAD :

• Physiological
a. Palpitation
b. Rapid breathing (Choking sensation)
c. Muscle tension and aches

271
272 Oceanic Series End of Posting Compilation

d. Abdominal discomfort
e. Nausea
f. Diarrhea

• Psychological
g. Fear
h. Anger
i. Irritability
j. Poor concentration
k. Worries
l. Negative thoughts

3. 2 laboratory investigations

• Thyroid function test


• ECG

4. 3 Managements and reasons for each

• Beta blockers
• Drug for thyroid (name)
• Psychoeducation
• Benzodiazepine

272
273 Oceanic Series End of Posting Compilation

MEQ 9

A woman was under birth control one year after her third child delivery. Two weeks after, she
developed severe headache

1.Elicit 4 symptoms from pt

• Hormonal history
• How long has she been on OCP
• Irritabilty
• Apathy(loss of interest)
• Long hours of sleeping
• History hypertenson
• Fever
• Loss of weight

Recently, she had been moody and easily irritable. She cannot take care of her children.

2. Elicit other 4 symptoms.

MDD symptoms (SAD A FACE)

• Sleep changes (less slepp, long hours sleep)

• Appetite change

• Dysphoric mood

• Anhedonia

• Fatigue

• Agitation (irritabiity) or Retardation (social withdrawal)

273
274 Oceanic Series End of Posting Compilation

• Concentration impaired

• Esteem low

3.Give your provisional diagnosis and


reasons.

• Medication-induced
depression

4.Give 4 management plan and the reasons (imagine self as a GP)

• Stop medication
• Start anti-depressants (SSRI)
• Psychoeducation
• Psychosocial intervention (family planning issue)
• Refer to psychiatrist if not responding to meds

MEQ 10

23 years old student brought by parents to a&e, d/t disturbed behavior & become
argumentative. Family noticed he spent a lot of money in past one week

1. 4 other symptoms you want to elicit (DIGFAST)

• Distractibility
• Indescretion(inhibtion loss)
• Grandisoity
• Flight of ideas
• Activity increase
• Sleep deficiency
• Talkativeness

274
275 Oceanic Series End of Posting Compilation

• Sleepy pattern
• Appetite
• Dysphonia
• Anhedonia
• Fatigue
• Agitiation
• Concentration decrease
• Self esteem decrease

Admitted to ward, becomes talkative and aggressive, claim himself as owner of hospital

2. List 4 Aggressive risk assessment

• Violent
• Substance abuse
• Unemployment
• Mental disorder

2. Provisional diagnosis and reason

• Bipolar in manic phase -


Grandiosity

• Schizophrenia -
Grandiosity

3. State one prophylaxis & adverse effect

Anti -psychotic(agranulocytosis)
Anti- convulsant
Mood stabilizers

275
276 Oceanic Series End of Posting Compilation

MEQ 11

52 years old male. brought by family to A&E behave abnormally in public, confused, speech
incoherent

1. 2 probable diagnosis

- Schizophrenia (relapse)

- Substance-induced Psychosis

- Delirium d/t organic causes

- Brief Psychotic Disorder

He was afebrile, BP 130/80, pulse irregular , no alcohol smell, no head trauma

2. 4 lab investigations

- ECG

- Urine toxicology

- Renal Function Test

- Random Blood Sugar

- Full Blood Count

At a&e become argumentative, talking & screaming loudly

3. 2 next action of management

- Sedative (Lorazepam etc)

- Anti-psychotic (Haloperidol)

276
277 Oceanic Series End of Posting Compilation

Contact family. History of multiple admission to psychiatric ward, non drug compliance for many
months

4. 2 choices of management

- Hospitalize

- Medication

Discuss with family,

5. 2 long term management

- Psychoeducation

- Psychiatric nursing home

- Rehabilitation therapy

277
278 Oceanic Series End of Posting Compilation

MEQ 12

35 years old man was brought to emergency department. His wife tell that he suddenly

became aggressive and has disinhibited behaviour as he walking naked from bathroom for the

past 3 days. He also has mood swings. His wife describe him as’ totally change’ person.

a) Give 3 differential diagnosis

Substance induced
psychosis

Delirium due to substance


intake

Delirium due to another medical


condition

b) 2 most relevant investigation

CT scan of brain

Complete blood count

Urine toxicology

He noted to be confused and slept on the other patient’s bed. His wife told that he complaint of
constant headache and fever for the past 1 week.

278
279 Oceanic Series End of Posting Compilation

c) Diagnosis : Encephalitis / meningitis

He had fits in the psychiatric ward.

d) 2 immediate management

Give IV diazepam and stabilise patient

Refer him to medical team for further


management.

He then was tranferred to ICU. He became agitated and has vivid hallucination. He see
IV drips as snake.

e) Name one medication to prescribe .

Haloperidol IV

f) 2 non pharmacological intervention.

Educate the nurse how to take care of


patient

Encourage family to accompany

Educate the family about his illness


(psychoeducation)

279
280 Oceanic Series End of Posting Compilation

MEQ 13

65 years old woman brought by her daughter to clinic due to change of behaviour for past
3 months. Recently lost her husband 6 months ago due to MI. Feels sad, keep forgetting
things and unable to perform daily activities. She presented with frequent constipation
and abdominal distress.

a) 2 differential diagnosis

Adjustment disorder

Pseudodementia

Major depression

MSE : tearful. Feels guilty and belive herself to be a burden to her son. Believe that
her stomach is rotting and it is incurable disease. Thinks she is gonna die. She was
convinced she has serious disease.

b) Final diagnosis and give 2 reasons.

Depression with psychosis / MDD with


psychosis

Reasons : appear tearful and feel sad , has delusion of guilt and nihilistic
delusion

c) 2 psychometric assesment

MMSE

Geriatric depression scale

280
281 Oceanic Series End of Posting Compilation

d) 2 medications and examples

Anti depressant – SSRI (fluoxetine,


fluvoxamine)

Anti psychotic –Risperidone, olanzapine,


quetiapine

e) 2 psychological intervention

Psychoeducation

Cognitive behavioural therapy

281
282 Oceanic Series End of Posting Compilation

MEQ 14

15 years old female is mentally subnormal due to epilepsy and has abnormal behaviour for
the past 1 week.

a) Give 3 causes of her mental subnormality

Birth trauma

Infection

Head injury

b) She had psychosis behaviour. Name one medication

Anti psychotics ( risperidone)

c) Epilepsy well controlled and fits develop once over a month. Name one medication to be
prescribed

Anti convulsant – sodium


valproate

d) She was treat as outpatient. What is the advice for her parents ?

Parents must ensure she takes medication to prevent


relapse

If possible, send her to special school or special class in the


school

She has high risk for sexual abuse (need to take extra
care)

282
283 Oceanic Series End of Posting Compilation

MEQ 15

1. Diagnosis: Acute alcohol intoxication

2. Outline the treatment.


3. Diagnosis: Delirium tremens
4. Outline the treatment.
5. Complication of alcohol intoxication – liver problems
6. What advice will you give to patient – Stop drinking alcohol

MEQ 16

Scenario: A woman has been having low mood since upper high school.

1. Provisional diagnosis: Dysthymia

2 weeks ago, she had been seen talking to herself and wanted to do suicide.

2. List 2 symptoms to support the diagnosis

3. Final diagnosis: Major Depressive Disorder

4. List the reasons why patient is not responsive to treatment?

283
284 Oceanic Series End of Posting Compilation

i) Medication is not given at adequate dose for an adequate time

ii) Poor compliance

iii) Side effects of medications

5. Outline the immediate treatment for this patient.

6. Outline the long term management for this patient.

MEQ 17

Scenario: A 60 years old woman came to ED with irrational speech and pacing for 2 days. BP
was 160/90 mmHg. Lab investigations were normal.

1. Provisional diagnosis and reason.

2. Give immediate plan for patient.

Husband told that patient has sustained fall 2 weeks ago.

3. Investigation that should be done and elicit the findings.

Patient was referred to medical ward for management. BP was stabilized but she developed
labile mood and often seen tearful and happy.

4. Your current diagnosis and reason.

Post stroke depression

284
285 Oceanic Series End of Posting Compilation

5. Outline management for patient.

MEQ 18

OSCE 1 Suicidal risk assessment

Interactive session

Suzana, 40 years old women been warded to the medical ward due to overdose of
antidepressant. She wants to go back home. All blood and other investigations are normal.

Take her suicidal risk assessment.

S Sex (Male)

A Age (<20 or >44 years old)

D Depression

P Previous suicide attempt

E Ethanol abuse

R Rational thinking loss (psychosis)

S Social support lacking

O Organized suicide plan

N No spouse (divorced / separated / widowed / single)

285
286 Oceanic Series End of Posting Compilation

S Sickness (presence of chronic or debilitating illness)

* Each risk factors that present is accorded a score of 1 point.

** Maximum points would be 10

Patterson et all recommended

Score 3-4 : Close monitoring

Score 5-6 : Consider hospitalisation

Score 7-10 : Hospitalisation for further assessment

* Regardless of the score obtained , overall clinical assessment is still paramount and
the primary care physician should err on the side of caution.

MEQ 19

A man arrested due to drug possession. After a day in lock up he developed headache,
abdominal discomfort and diarrhea. a) elicit 4 other signs and symptom for diagnosis

• nausea or vomiting

• muscle aches

• lacrimation or rhinorrhea

• pupillar dilation

• piloerection

• diarrhea

286
287 Oceanic Series End of Posting Compilation

• yawning

• fever

• Insomnia

• Dysphoric mood

b) noted there's puncture mark on his arms state the diagnosis and 1 reason

Ans: Opioid Use Disorder at withdrawal state; Reason – heroin usually


through IV

c)2 med Rx for detoxification Ans: (1) treat withdrawal sx

• NSAID

• Antidiarrheal

• Antiemetic

• Clonidine (α2 adrenergic agonist)

• Benzodiazepines (2) substitution treatment

• buprenorphine (partial opiate agonist)

• methadone (long acting synthetic opiate)

d) fit to discharge, what further action.

Ans: • methadone replacement therapy

• rehab center

287
288 Oceanic Series End of Posting Compilation

MEQ 20

Scenario 2 Seorg wanita berusia 30an. Dtg dgn aduan dia berasa nervous dan masalah tidak
selesa perut. Dia dh jmpa byk Dr. dh. Dia sendiri juga tidak mengetahui punca utama yg buat
dia rasa mcm ni.

a) Senaraikn 4 symptoms lain yg boleh didapati dlm pesakit ini.

Ans: • fatigue

• restless

• muscle tension

• irritability

• impaired concentration

• sleep disturbances

b) berikan diagnosis awal anda dan nyatakn 2 sebab.

Ans: General Anxiety Disorder; reason – based on criteria yg dia ada

Mak beliau juga menghdpi bnda yg sama seperti dirinya. Beliau sbnrnye sakit sama seperti ini
sejak berumur 25 thun.

c) senaraikan 2 faktor yg boleh menyebabkn sakit ini.

d) nyatakan 2 terapi bg membantu pesakit ini.

Ans: • Behavior therapy

• Cognitive therapy

• Group therapy

288
289 Oceanic Series End of Posting Compilation

• Exercise (relaxation)

• Psycho education

MEQ 21

Male 23 y/o, talking excessively, irritable mood for 1 week

a) 4 signs and symptoms to ask from parents

Ans: (manic sx DIGFAST + mood swing)

b) Admitted to ward, extra mesra dgn nurse, state dx and reason

Ans: Bipolar Mood Disorder in Manic Phase; reason – explain manic phase

c). Mx patient in acute phase

Ans: • Mood stabilizers (lithium, sodium valproate)

• Symptomatic drugs – benzodiazepine (control agitated behavior/ sleep)

• Antipsychotic (if with psychotic sx)

• ECT

d) 2 medications + investigation

Ans: • lithium (renal fx/thyroid fx)

• sodium valproate (liver fx)

5. Parents nak bwk balik tp x okay lg, 2 Mx

Ans: • ECT

289
290 Oceanic Series End of Posting Compilation

MEQ 22

18 years old always fearful and ada social withdrawal

a) 4 Q's nk Tanya kat parents

Ans: (other psychotic symptoms)

b) ddx

Ans: schizophrenia

c) pharmacotherapy

Ans: • Antipsychotic

d) other type of medications for non-compliant

Ans: • Depot injection

e)2 non-pharmacology punya TX

Ans: • Psychoeducation

• ECT

290
291 Oceanic Series End of Posting Compilation

MEQ 23

21 years old man involved in an accident 1 year ago, had cerebral concussion, however his
father who was riding motorcycle, died due to severe head injury. Has irritability, sleep
disturbances with nightmare and poor concentration.

Give 4 other signs and symptoms for diagnosis

Depressed mood.

Flashback of the scene.

He is having nightmares (sleep affected).

Feel fearful to go back to the scene.

Sister has schizophrenia. List 4 features to rule out psychotic symptoms.

Hallucinations

Delusions

Disorganised speech

Disorganised behaviour

PET scan was done. He has no psychotic symptoms. Provisional diagnosis with reason.

Post-Traumatic Stress Disorder (PTSD)

Because of previous serious traumatic event.

It is more than 1 month.

291
292 Oceanic Series End of Posting Compilation

Give two types of drugs suitable for him with examples

SSRI: Citalopram, Fluoxetine

Benzodiazepine: Lorazepam, Alprazolam, Clonazepam

His symptoms improved, however he still cannot concentrate to his study. List two non-
pharmacological treatment you would like to offer.

Relaxation Therapy.

Cognitive Behavioural Therapy.

MEQ 24

60 years old man, missing from home for 1 day. Found by the road side as his car doesn’t have
fuel. He was very tired. Cannot remember home address. Police brought him back to family.

List 2 possible diagnosis

Alzheimer’s Dementia

Delirium

Alcoholic Delirium

List 2 relevant investigation and give one reason each.

Electrolyte investigation. Detect underlying electrolyte which can cause delirium.

Full blood count. To rule out infection (underlying infection cause delirium)

Random blood sugar. Search for hypoglycaemia.

CT scan. To find out for bleeding, injury to the brain.

Urine investigation. To deduct UTI.

292
293 Oceanic Series End of Posting Compilation

Give others 2 Instrumental Activities of Daily Living

Grocery shopping

Driving

Taking medicine on his own

Paying bills

Preparing a meal

Using telephone

Scored 22/30 on MMSE. Geriatric Depression Scale 4/10. Diagnosis and 2 reasons

Alzheimer’s Dementia.

No comorbidities.

Family history of dementia.

History of short-term memory problems

MMSE only 22/30

List 2 drugs of choice at this stage and mechanism of action

Rivastigmine and Donepezil (because he only had mild dementia)

MOA: Blocks Acetylcholine at the synapse, thereby increase Ach at synaptic cleft and
improve the memory

293
294 Oceanic Series End of Posting Compilation

MEQ 25

45 years old woman diagnosed with breast cancer stage 2, one month ago and become sad
and anxious since then. She demands counsel on treatment.

List 4 possible diagnosis and reasons

Major Depressive Disorder. Because she felt sad (low mood) since one month ago.

Adjustment disorder with depressed mood. The stressor present and duration less than
one month (not more than 3 months).

Acute stress disorder. Sudden onset of serious illness and the duration is less than one
month.

Depression disorder due to medical condition. Having low mood one month ago
secondary to breast cancer.

List 2 clinical features with examples

Biological: Sleep pattern, appetite, libido

Psychological: Low mood, crying, anhedonia

Behavioural: Psychomotor retardation, self-neglect, apathy, lack of self-care

Social occupational functioning

Cognitive: Hopelessness, uselessness, worthlessness, guilt

4 plan of management and example

Pharmacotherapy: SSRIs (Fluoxetine, Citalopram)

Psychotherapy: Cognitive Behavioural Therapy

294
295 Oceanic Series End of Posting Compilation

Social therapy: Find the family support, develop the social network

Psychoeducation: Educate patient and families about medical condition, about the
compliance.

MEQ 26

27 years old man, stuporous and have food refusal for past 2 days. He also feel suspicious
towards his wife’s loyalty for the past few months. No similar episode from the past. Brought to
emergency by his wife.

List 4 other features to ask his wife to confirm the diagnosis

Hallucinations

Mood

Any change in orientation (confusion)

Any history of seizure disorder or infection induced seizure

Physical examination: Conscious, but not respond to command, afebrile and rigidity of limbs.
List two differential diagnosis and reason for each.

Catatonic Schizophrenia. Acute onset, muscle rigidity, history of delusions feature,


conscious but not responding to command.

Organic Brain Syndrome: Acute onset, stupor and refusal to eat.

List 2 investigation for the patients and give the reason for each.

Imaging (CT scan, MRI): Exclude any brain injury

Full blood count/urine test: To rule out infection

Renal function test/ liver function test: Detect any organ failure

Urine toxicology: Exclude any gut related complication.

295
296 Oceanic Series End of Posting Compilation

All investigation found to be normal. The patient refuse to eat and become stupor in the ward.
What is your immediate treatment for the patient and give the immediate complication.

Immediate treatment: ECT

Immediate complication: Retrograde amnesia, severe headache

MEQ 27

Mr A irritable at office, always disturb his friend. His wife complains of him

having abnormal behavior. He has poor sleep. He started to do artistry painting and

convinced that he would win an art competition despite having no artistry inclination.

Duration : 1 month

1) Give two differential diagnosis

-Bipolar Mood disorder (Manic)

-Brief psychotic disorder

- substance induced psychotic disorder

2) Ni tak silap dia tambah ciri belanja spree and had history of bipolar depression state,

what is the final diagnosis and reason

-BMD (Manic phase)

3) 2 medications to prevent relapse:

-Lithium

-Sodium valproate

296
297 Oceanic Series End of Posting Compilation

4) Patient then develop tremor and excessive thirsty after receiving treatment

Investigation to be done and reason:

(patient now develop diabetes insipidus)

1.

- Serum lithium

- Reason : Because develop diabetes insipidus

2.

- Thyroid fx test

- Reason: Lithium induce hypothyroidism

5) Pt is on pharmacotherapy and explain

- CBT

-Psycoeducation

297
298 Oceanic Series End of Posting Compilation

MEQ 28

QUESTON 2: MDD

Housewife.. difficult to initiate sleep for...

1) 4 other quality of sleep:

-multiple awakening

-difficult to sleep again after sudden waking up

-lethargic in the morning

-short duration of sleep

Feeling sad most time. Worst in morning. Husband lost job 2months ago.

2) 4 other core symptoms

(SIGECAPS)

-loss of appetite

-loss of interest

-impaired concentration

Husband had found a low pay job 2 weeks ago. She is very worried about her 3 daughters

who're still studying. However, no family issues.

3) 2 differential diagnosis and reason

-Mdd

-adjustment disorded with depressive mood

298
299 Oceanic Series End of Posting Compilation

4) 2 Class of Medication

- anti depressants - Ssri/snri/tca

-sleeping pills - BZD -

5) Nonpharmaco therapy

-CBT

-Support therapy

-psychoeducation

299
300 Oceanic Series End of Posting Compilation

MEQ 29 OCD

A 36 years old lady want to strangulate her daughters whenever she sees them. So, she feel

guilty, worry and cannot focus doing her work.

1) 2 diagnosis and reason for each.

-OCD – recurrent thought to strangulate her daughters

-Schizophrenia – Paranoid delusion

-MDD – feel guilty and has functional decline

(tak boleh jawab GAD)

She keeps going out to avoid her daughter. She denied having any hallucinations.

2) Provisional diagnosis

OCD

3) Other symptoms to elicit to help the diagnosis

- Any rituals done

- Repeatig washing hands

- Orderliness

- Feeling anxious

300
301 Oceanic Series End of Posting Compilation

4) Pharmacotherapy

-SSRI – Fluoxetine

-TCA – Amitriptylin

5) She need more than pharmacotherapy. Give two elements of behavioural therapies

she need.

-Flooding therapy

-Systematic desensitization

MEQ 30 Opiate Withdrawal

32 years old man brought to the A&E by the police. On examination, he looked agitated, had

muscle cramps and pilo-erection occurred.

1) State the diagnosis

- Opiate withdrawal syndrome

2) 2 other signs

- Rhinorrhea

- Lacrimation

- Puncture scars

3) He was restless and wants to run away. 4 symptomatic drugs that should be given.

- Anti emetic – to treat nausea and vomiting

- Analgesic – to treat cramps and pain

301
302 Oceanic Series End of Posting Compilation

- Anti-spasmolytic – to treat diarrhea

- BZD – acts as sedatives

4) Lab investigations shows positive for infection. What it could be ?

- Hepatitis B

- Hepatitis C

- HIV

5) On 5th day, able to be discharged. Safety precautions given to patient:

- Register for needle exchange programme

- Practise safety sex

6) Long term management

-Refer for methadone clinic

-Drug Rehabilitation centre

302
303 Oceanic Series End of Posting Compilation

MEQ 31

4.30 yo with excessive worry about her health cause her friend was diagnosed with breast

Ca

1) List clinical features

• Palpitation
• Dry mouth
• Muscle tense
• Sleep disturbance

2) She still worried despite investigation been done. Diagnosis?

• Hypochondriasis
• GAD

3) Final diagnosis – hypochondriasis (illness anxiety disorder)

4) Medication?

Benzo

303
304 Oceanic Series End of Posting Compilation

MEQ 32 Liaison case Acute psychosis

1. State 4 relevant history that you want to elicit.

o hx of taking recreational drug,


o family hx of schizophrenia
o hx of alcohol abuse,
o taking according to amount prescribed or not
2. What is your next step in the management
o Hospitalize,
o Give antipsychotics
3. State your Differential Diagnosis.

o substance-induced psychosis
o acute stress disorder with psychosis
o schizotypal disorder
4. Definitive diagnosis

drug induced psychosis


5. State your management.

MEQ 33 Liaison case Thyrotoxic psychosis

MEQ 34 anxiety disorder

30 years old high functional career woman experienced anxiety attack with palpitations,

restlessness, tingling sensation for the past 3 months. She even difficult to go to grocery shop.

She has no underlying medical illness.

1. State 3 differential diagnosis?

Husband noticed she was agitated, irritable, increase appetite but no loss of weight.

304
305 Oceanic Series End of Posting Compilation

2. What is your provisional diagnosis?

3. State the investigations and expected finding

Now she complains of seeing birds.

4. What is your provisional diagnosis?

5. State 2 psychiatric medications and its example.

MEQ 35 Liaison Case Acute psychosis due to hypoglycemia

MEQ 36

A 40 – year old man become anxious after the COVID – 19 virus outbreaks in his neighborhood.
His mind was preoccupied with the possibility of contracting the infections. He developed flu –
like symptoms one week ago and was tested for the infection. He heard voices discussing about
his well – being and was unable to sleep since then.

A. Give FOUR relevant category of symptoms you would ask. (2 marks)

His screening test was reported to be negative. His flu like symptoms was not relieved despite
of medications. His housing area was placed under complete lockdown. He strongly believed
that he had contracted the infection. He become over sensitive to any news related to the
pandemic.

B. Give TWO most probable differential with reasons you would consider now. (2 marks)

Diagnosis Reason

i.

ii.

The following week his condition worsened, and he became agitated. He was worried about his
upcoming repeat test.

305
306 Oceanic Series End of Posting Compilation

C. Give TWO medications of choice with a reason each. (2 marks)

Medications Reason

i.

ii.

The repeat test for COVID 19 infection was negative. The psychological symptoms were fully
resolved in the fourth week from the onset of symptoms.

D. Give TWO important decisions with regard to medication and your reason. (2 marks)

Decision on medications Reason

i.

ii.

However, during the following months he continued to avoid going out of his house as he was
afraid of contracting the virus.

E. Give TWO specific psychological interventions with your reason. (2 marks)

Specific psychological interventions Reason

i.

ii.

306
307 Oceanic Series End of Posting Compilation

MEQ 37

A 23 – year – old schoolteacher had a normal delivery of her first baby two weeks ago. She was
overly worried about supporting her baby. She was not able to sleep because the baby was
constantly crying. She developed these immediately after her delivery.

A. List FOUR causative factors that can lead to the above symptoms. (2 marks)

No. Causative Factors

i.

ii.

iii.

iv.

Her husband lost his job about a month ago before her delivery due to the COVID – 19
pandemic.

B. Give TWO probable diagnoses and your reason for it. (2 marks)

No. Diagnoses Reason

i.

ii.

The following week she continued to have feelings of incompetency as a mother and regretted
giving birth to the baby. She was struggling with breast feeding and house chores. She felt tired
taking care of the baby and looked distressed and tearful.

C. List TWO important questions that you would ask to prevent serious events. (1 mark)

Her husband was worried about her condition and wanted to know about her diagnosis.

D. State the most probable diagnosis and give THREE reasons. (2 marks)

307
308 Oceanic Series End of Posting Compilation

Final Diagnosis Reasons


i.
ii.
iii.

Her husband was not keen on hospitalization; hence she was given out – patient treatment.

E. Give TWO specific pharmacological agents and TWO non – pharmacological


management that you would offer. (2 marks)

No. Pharmacological agents Non – Pharmacological

i.

ii.

Her family was keen on breastfeeding but worried about the medication that she was
prescribed.

F. Give TWO advice that you would give her on breast feeding with regards to medication.
(1 mark)

MEQ 38

A 30 – year – old housewife with episodes of palpitations and inability to do her usual house
chores since her mother was admitted with Corona virus infection two weeks ago. Her family
was screened and quarantined at home.

A. Give FOUR group of symptoms that you would like to elicit in history of presenting
illness. (2 marks)

308
309 Oceanic Series End of Posting Compilation

Fortunately, all the other members in the household were not infected. After two weeks, her
palpitation worsened when her mother was admitted to Intensive Care Unit (ICU) for severe
acute respiratory syndrome.

B. Give TWO differential diagnosis and your reason for considering them. (2 marks)

Differential Diagnoses Reasons

i.

ii.

Subsequently, her mother died after ten days in the ICU. She was devastated after being
informed by the hospital authority. The family was not able to attend to the funeral.

C. Give TWO psychological intervention strategies you recommend and its reason for each.
(2 mark)

Intervention Reasons

i.

ii.

She had frequent crying spells blaming herself for her mother’s death even after two months.
Her sleep was interrupted with infrequent dreams of her late mother.

D. Give TWO differential diagnoses you would consider and give your reasons. (2 marks)

Diagnosis Reasons

i.

ii.

Her family brought her to a psychiatrist and plan of management was discussed.

309
310 Oceanic Series End of Posting Compilation

E. Give TWO specific plan of management for her with your reasons. (2 marks)

Plan of management Reasons

i.

ii.

MEQ 39:

A 30 – year – old single man was brought by the police to the Emergency Room (ER) for
causing grievous hurt to his house mate. He was restless and paranoid towards people around
him for the past two weeks.

A. List TWO group of symptoms that you would elicit in the history of presenting with TWO
examples each. (2 marks)

No. Group of symptoms Examples

i.

ii.

He was caught for stealing on several occasions and was abusing ‘Syabu’ for many years.

B. List FOUR questions you would ask about his issues related with drug abuse. (2 marks)

He preferred to be alone and never had any close relationship in the past.

C. List TWO differential diagnoses with a reason for each. (2 marks)

No. Diagnosis Reason

i.

ii.

310
311 Oceanic Series End of Posting Compilation

He was agitated at the ER and was eventually sedated. The Hospital Director was not willing to
accept the case due to the seriousness of the problem.

D. Give TWO most appropriate decisions that you would consider and the reason for it. (2
marks)

No. Appropriate Decision Reason

i.

ii.

He became mentally stable after one month of treatment and was fit to go for his trial in the
court. The court ordered for a medical report.

State FOUR important points to be mentioned by the psychiatrist in the medical report. (2
marks)

MEQ 40 Drug induced mania

A 22 female, an active college representative, failed her exam and became socially withdrawn
for 1 year where she refused to eat and talk to her parents.

a) Other symptoms that you would like to elicit


-
-
-
-

b) Differential diagnosis with reason(s)


-adjustment disorder with depressed mood
Reason:
-major depressive disorder
Reason:

311
312 Oceanic Series End of Posting Compilation

c) Medicine you would like to prescribe to her and give reason


-
Reason:

d)During follow up after a month, she sang loudly in the clinic and became very talkative.
What is the diagnosis now?
-
Reason:

e) She became aggressive, however refused to be admitted.


What is the next course of management for the current situation? (4)
-
-
-
-

f) Social therapy
-
-
-

312
313 Oceanic Series End of Posting Compilation

MEQ 41 Bipolar in manic/schizophrenia

A 27 years old male was referred to psychiatry department after episode of fighting of his co-
worker, saying that everyone is conspiring against him and try to do bad thing to him.
Additionally, he also felt unsafe in his working environment since 2 years ago as he always feel
like his co-workers try to harm him.

Others histories to elicit (4)

2 differentials with reasons

While in ward, getting aggressive, saying things like the medical attendants try to harm him.
What medication to give?

Management of this patient

313
314 Oceanic Series End of Posting Compilation

MEQ 42 Alzheimer's disease with behavioral modification

A 67 years old female came with the complaint of always forgetting about mundane things in
daily life and self-neglecting herself. Recently said that she feels like there were people
watching her but there was no one there. She sometimes forget her own children and having
inappropriate behaviour.

a) Others histories to elicit


-
-
-
-

b) differential diagnosis with reason (s)


(2 differential, 1 reason each)

This patient was admitted to the ward whilst waiting for her cognitive assessment results. While
in ward, she feels depressed and cannot sleep.

2 medications to help her with reasons


-
-

How to manage this patient?


-
-
-
-

The cognitive results came out as deterioration in her cognitive function (basically, mmse results
decrease). What is your final diagnosis with 3 reasons?

Final dx:
Reasons :

314
315 Oceanic Series End of Posting Compilation

MEQ 43 Liaison case, temporal epilepsy

A 35 year old man, came with chief complaint of always staring into spaces without him
knowing. Furthermore, he always feels like things around him had happened before, but may or
may not be happening (deja vu). Came to the ED.

a. Other histories to elicit


-
-
-
-

b. Relevant investigation for this patient with reasons


-
-
-

c. Your final dx with reasons?


(Final dx with 3 reasons)

d. There was some abnormalities in his neurological investigation. What class of medication to
prescribe with 3 examples?

Class of medication :
3 examples :

(Note: in temporal lobe epilepsy, the patient will come in with absence seizure and feeling of
deja vu)

MEQ 9

27 years old man, stupors and have food refusal for past 2 days. No similar episode from the
past. Brought to emergency by his wife.

315
316 Oceanic Series End of Posting Compilation

A. List other feature to ask his wife to confirm the diagnosis

1. Hallucination

2. Mood

3. Any change in orientation (confusion)

4. Any history of seizure disorder or infection induced seizure

Physical examination: conscious but not respond to command, afebrile and rigidity of limbs.

B. List two differential diagnosis and reason for each

1. Catatonic schizophrenia. Acute onset, muscle rigidity, history of delusions feature,


conscious but not responding to command

2. Organic brain syndrome: acute onset, stupor and refusal to eat

C. List 2 investigation for the patients and give the reason for each

1. Imaging (CT scan and MRI); exclude brain injury

2. FBC; to rule out infection

3. RFT’; detect any organ failure

4. Urine toxicology; exclude any gut related complication

All investigation found to be normal. The patient reduces to eat and become stupor in the ward.
pharmacotherapy was not effective. Parents want to bring patient to home

D. What is your immediate treatment for the patient and give indication

Electroconvulsive therapy (ECT)

1. No progression in ward

316
317 Oceanic Series End of Posting Compilation

2. Pharmacotherapy was not effective

3. Immediate effect (parents want to bring patient home)

MEQ 44

A 30 years old lady came to clinic because she felt nervous, had palpitation and shortness of
breath. She worries she might lose control in the public hence she would avoid going to
crowded place. Her husband always need to always accompany to go to groceries. She had
seen many doctors. She did not know why she had been experiencing that way.

A. List FOUR symptoms that may be presented in this patient

1. Chest pain

2. Fear of dying

3. Choking sensation

4. Derealization / depersonalization

B. Give two differential diagnosis and Reasons


two reason each Differential diagnosis

1. Panic attack i. Palpitation and shortness of


breath

ii. Fear of losing control

2. Social anxiety i. Avoid going to crowded place

ii. Impairment to do daily activities


alone

317
318 Oceanic Series End of Posting Compilation

OSPE

OSCE 1

This patient has been referred to the psychiatric clinic for alcoholic drinking. Assess this patient for his
drinking habit. Take history with particular reference to alcohol dependence and problem drinking.

a) Greetings
b) Introduce yourself and ask patient’s name and age.
c) Explain your purposes
d) Reassure patient about this session and confidentiality.
1. Introduction e) Ask consent
f) Ask chief complaint

a) Type of alcohol
b) Duration
2. Pattern of use c) Quantity
d) Frequency of use
e) Amount spent

3. Tolerance Over time, do you find that you have to drink more to get the
same effect?

a) Do you become anxious & tremulous if you do not have a


drink every day?
4. Withdrawal

318
319 Oceanic Series End of Posting Compilation

b) Do you have to drink in the morning to prevent this


feeling?

5. Maladaptive pattern of Do you spend a lot of time and money to obtain alcohol
use every day?

6. Craving for drink Do you crave/keep thinking about having a drink when at
work?

a) Do you sometimes leave early from/ come late to/miss


work or miss family dinners/activities because you are out
7. Problem fulfilling his
social, occupational & drinking?
family obligations
b) Do you find yourself short of money because you
have spent it obtaining alcohol?

a) Do you drive after drinking?


b) Have you ever got into trouble with the police for drunken
behaviour/ drunk driving?
8. Hazardous use
c) Have you ever got into brawls/fighting after drinking?
d) Have you ever been involved in accidents because of
alcohol?

a) Have you been feeling depressed/anxious?

9. Psychiatric b) Are there times you experienced hallucinations?


complications c) Are there time you feel hopeless/feel like giving up?
d) Have you experienced blackouts?

a) Have you been hospitalized for excessive drinking?


10. Delirium Tremens
b) Have you ever developed seizures after stopped
drinking?

319
320 Oceanic Series End of Posting Compilation

a) Do you feel that you need to cut down on alcohol?


b) Do you get annoyed by criticism regarding your
11. CAGE
alcohol use?
c) Do you feel guilty regarding your alcohol use?
d) Have you had a drink first thing in the morning to
steady your hands?

a) Do you feel that you have a problem with alcohol?


12. Insight/Motivation b) Do you think you need to stop drinking?
c) Would you be willing to come for counselling?

OSCE 2

Mr David visits his GP to consult about his emotional problems that had affected his work
performances. Mr David complained to his GP that he is unable to relax for a long time.

1. List 4 symptoms of clinical significance that brought him to see his doctor

i. Fear of dying
ii. Believes that he is having heart attack
iii. Experiences symptoms of heart attack
iv. Wouldn’t go for a long walk
v. Needs someone to be with him when he goes out

2. Comment on his functional capability

i. Employ people to do his work and losing clients

3. What do you think he is suffering from?

320
321 Oceanic Series End of Posting Compilation

i. Panic disorder

4. List 2 complications of the condition above

i. Depression
ii. Agoraphobia
iii. Functional decline
iv. Drinking alcohol excessively

5. List his 6 mood symptoms

i. Persistently low mood


ii. Hopelessness/no future for his business
iii. Functional decline
iv. Insomnia
v. Don’t feel rested
vi. More tired
vii. Decreased interest in sex

6. List 2 drugs that you would prescribe him and mention the frequency of each drug.

i. SSRI (Fluoxetine) – regular daily dose


ii. Benzodiazepine (alprazolam) – as needed/PRN

321
322 Oceanic Series End of Posting Compilation

OSCE 4

Interactive station

Take history of depressive symptoms (SIGECAPS)


Add-on: Need to rule out severe depression by asking any psychotic symptoms

OSCE 5

Static station

Mrs Jones, 65 years old woman has sudden change of behaviour. She has underlying
Alzheimer’s disease.

a. What cause her symptoms to worsen in the evening?


Sundowning, delirium

b. 2 physical features that can be observed in Mrs. Jones


Short, shuffling gait
Echhymosis around the eyes

c. 2 psychological features
Visual hallucination
Auditory hallucination
Paranoid delusion

322
323 Oceanic Series End of Posting Compilation

d. 2 reasons for changing behaviour


UTI
Due to multiple falls, maybe subdural haemorrhage
pain
Dementia behavioral and psychological symptom

e. 4 reasons for her frequent falls


Arthritis
Drugs effects
Parkinson’s disease
Poor vision
Postural hypotension

f. Provisional diagnosis - Dementia with delirium

OSCE 6

Video Station

a. Identify 5 psychopathologies (based on video)

Visual Halucination

Olfactory hallucination

Tactile hallucination

Thought echo

Delusion of reference

b. What was the questions given to elicit perception disturbance


“Do you think anything seems strange to you?”

323
324 Oceanic Series End of Posting Compilation

c. Do patient has perception disturbance?


No

d. Comment on affect of patient

CONSTRICTED/RESTRICTED AS PATIENT STILL CAN SMILE AND TALK

(Emotional blunting is usually for Parkinson disease, if flat the worse affect
when patient does not want to talk to you and no expression at all)

e. Comment on abstract
Good as patient talk about function rather than concrete

f. Example of abstract
APPLE AND ORANGE; UNCLE AND COUSIN: PAINTING AND POEM

g. Patient’s insight
PARTIALLY GOOD AS HE ONLY WANTS TO TREAT HIS SLEEP PROBLEMS
NOT HIS MENTAL ILLNESS

h. What question was given for insight?


Do you think you have mental illness, …

i. Provisional diagnosis : Schizophrenia

324
325 Oceanic Series End of Posting Compilation

OSCE 7

Video station:

Alcohol dependence

1.What made the patient to stop his drinking attitude?

2. what his attitude towards his drinking habit?

3.State 6 good manners that doctor showing in that video

4. State 3 bad manners shown by the doctor in that video

5.What is the diagnosis

6.Treatment for the diagnosis above

OSCE 8

Delirium

1.State the condition shown part 1?

Reason?

(HYPOACTIVE DELIRIUM)

2.State condition shown in part 2?

Reason?

(Hyperactive Delirium)

3.Final Diagnosis

4.Management for the condition

325
326 Oceanic Series End of Posting Compilation

OSCE 9

Schizophrenia

1. State your observation on his appearance in that video?

2. State the 2 delusion shown by the patient? Reason for each.

3. Any hallucination shown by the patient in the video? Give reason.

4. Final Diagnosis.

5. Management?

OSCE 10

Panic Disorder

1.What is the reason given by the patient for her current visit?

2. Give 2 Differential Diagnosis?

3.What are complications from her condition?

4.Causes for her current condition?(was given in that video)

5.Provisional Diagnosis.

6.Management?

326
327 Oceanic Series End of Posting Compilation

OSCE 11

Interactive station

Middle aged woman attempted suicide. She was under psychiatric follow up for past 8 months
for her depression.

1. Access the suicidal risk of the patient and state whether the patient need to hospitalizaton ?

2.I ADL (Instrumental activity of daily Living)- Access the IADL from the caretaker

OSCE 12

Station 1 (video)

A woman presented with fear of her left breast


lump.

a) Elicit 2 chief complaints


Breast lump for 2 week
duration
Painless and palpable breast
lump

b) Her affect – anxious and


worried

c) 3 biological symptoms
Loss of
appetite
Poor sleep

327
328 Oceanic Series End of Posting Compilation

Decrease
libido

d) 5 phychological symptoms
Low mood
Low
concentration
Anhedonia
Crying most of the
time
Fear of dying due to breast
ca

e) 2 differential diagnosis and


reasons
Illness anxiety disorder
(hypochondriasis)
MDD with anxious
mood

f) 3 plan of management
Breast examination (with/ without
mammogram)
Start anti
depressant
Anxiolytic agent
Cognitive behavioural
therapy

328
329 Oceanic Series End of Posting Compilation

OSCE 13

Station 3 (video)

A man preesented with


anxiety

a) 2 main complaints :
Fear /mindful
worry
Difficulty to get out of
house

b) His affect at the initial of the video : anxious and


angry

c) Physiological symptoms he experienced:


Restless , edgy , early awakening, preoccupied mind
with worry

d) 2 differential diagnosis and


reasons
GAD
MDD

e) 3 management plan
Anti depressant –
SSRIs
Anxiolytics
Coping
therapy
Relaxation
technique

329
330 Oceanic Series End of Posting Compilation

OSCE 14

Station 5 (video)

A elderly man presented with low mood and history of manic phase in the
past.

a) Elicit 5 symptoms he experienced.


Manic symptoms ( DIG
FAST)

b) His affect : Sad and appropriate to the


word

c) Diagnosis : bipolar affective disorder in depressed


phase
d) Poor prognostic factor
Poor insight
Fear of stigma
Family history of
depression
Recurrent
depression
Non compliance to
medications

e) 2 classes of pharmacology intervention and


examples
Antidepressant –
SSRI
Mood stabiliser – lamotrigine, carbamazepine, sodium
valproate

330
331 Oceanic Series End of Posting Compilation

OSCE 15

Station 6

A man presented with


hallucinations

a) 4 types of hallucination he has

Auditory, visual, tactile and olfactory


hallucinations

b) What is the examiner question about ‘delusional perception


‘?

c) What is the examiner question regarding’ idea of reference’


?

d) Comment on his insight and give


reasons

Poor insight – take medication only for sleep and can handle his
hallucinations

e) Comment on his abstract thinking and give 2


examples

f) 3 physical appearance

Tremor , restless,
mannerism

331
332 Oceanic Series End of Posting Compilation

g) Give 2 management to reduce side effects of


medication

Decrease the dose or change the medication to atypical anti


psychotics ‘
Add anti cholinergic
(benzhexol)

OSCE 16

VIDEO STATION (WATCH VIDEO AT THE STATION AND ANSWER THE QUESTIONS)

1. Dementia
1) List 3 attitudes shown by patient
2) List 3 clinical features of patient with reason
3) List reasons why patient’s condition worsen in the evening
4) List 3 reason why patient experienced visual hallucination
5) Your diagnosis
6) Treatment and management

OSCE 17

2. OCD
1) List 4 rapport building skills that a doctor must have
2) List 6 attitude shown by the patient
3) List 4 aspect of patient's obsessions
5) List compulsion activity done by patient
6) Your diagnosis
7) Treatment and management

3. Illness Anxiety Disorder (old term: Hypochondriasis)


4. Social Anxiety Disorder

332
333 Oceanic Series End of Posting Compilation

OSCE 18

INTERACTIVE STATION (SIMULATED PATIENT)

WAYS TO APPROACH:

a) Good morning. How are you today?

b) I am (your name), 4th year medical student. May I know your name and age please?

c) I am here today to ask you a few questions so that we can explore with regards to the
problems that you are facing.

d) I can assure you that whatever we have discussed will be private and kept confidential. Are
you comfortable/okay with that?

e) Then, ask for chief complaints.

What brought you here today?

Is there any problem that you are facing lately?

f) Before ending the interview question, don’t forget to summarize every symptoms that the
patient complained (safety net).

g) We’ve almost completed our interview, so I would like to summarize. Based on the interview,
I understand that you are having (mention all the symptoms that the patient is having) and is
there anything else that you’d like to tell me?

h) End the session with a thank you and a smile

333
334 Oceanic Series End of Posting Compilation

OSCE 19

EXAM QUESTIONS

Elicit the First Rank Schneider symptoms in the patient

1. Voices that:

a) repeat your thoughts – or anticipate your thoughts (audible thoughts)

b) 2 or more voices discussing about you

c) Keep up a running commnentary on your thought/action/activities

2. Thought insertion

3. Thought withdrawal

4. Thought broadcast

5. Feeling controlled by outside force (somatic passivity)

6. Delusional perception – have you experienced that you suddenly understood what you
are seeing/hearing had a special meaning?

OSCE 20

Elicit the Manic symptoms in the patient

Have you ever had a period of a week or so in which you felt so happy and energetic that
people around you told you that you are behaving strangely or talking too fast?

DIG FAST

1. Distractibility

2. Irritable mood

3. Grandiose

4. Flight of ideas

334
335 Oceanic Series End of Posting Compilation

5. Activity (goal directed activity / activity increase) spending spree many things to
do plan

6. Sleep deficit (ask about energy as well)

7. Talkativeness

OSCE 21

Station 2 (video)

a) Explain her affect

Ans: Affect - anxious

b) Give 2 differential diagnosis and reason

Avoidant PD & Social Phobia

c) Treatment

(1) biology (pharmacology) (2) psychosocial (3) behaviour

335
336 Oceanic Series End of Posting Compilation

OSCE 22

(interactive)

Patient was diagnosed severe depression. She had no response to antidepressant. Consultant
suggested for ECT. You are asked to explain about ECT and get consent from the patient.

Ans: -

greet, introduce yourself,

explain the discussion will kept private,

consent.

- procedure (anaes, muscle relaxant)

- how often should be given (3/week)

- feel better after the third session depends on individual

- side effect (headache, transient memory loss, physical injury, confusion)

- chances getting better (75% get better + antidep therapy)

- minimal risk of complication

- precaution (overnight fasting, full blood count - anaes assessment)

- it won't cause brain damage

336
337 Oceanic Series End of Posting Compilation

OSCE 23

(video)

Patient has psychological symptoms.

1. Chief complaint.

• Feeling sad for the loss of the husband

2. 2 ddx and 3 reason each.

• MDD

• Depressive disorder

3. Level for suicidal risk assessment and 2 reasons.

• Moderate – (wish she was dead, but no plan)

4. Pharmacotherapy and 2 psychotherapies

• Antidepressants

• Behavior therapy

• Cognitive therapy

• Group therapy

337
338 Oceanic Series End of Posting Compilation

OSCE 24

(interactive)

Patient with depressive disorder but refuse taking antidepressant.

1) advise to take medication

• how it works (MOA - block serotonin reuptake inhibitor)

• what type of antidepressant (SSRI, TCA, MAOI)

• side effect

• 60-70% get better after Rx

• how long should I take med before I feel better (2-3week)

• how long should be on med (6-9 months)

2) asess insight

• Ask did she know about her illness

• Did she need to take medication?

OSCE 25

(Video)

Schizophrenia.

1. Four hallucination

2. How interviewer ask about delusional perception .

3. How interviewer ask about delusion of reference

338
339 Oceanic Series End of Posting Compilation

4. How is patient abstract thinking and 2 examples

5. 3 physical signs and reason.

6. Mx for side effects

OSCE 26

Video : https://www.youtube.com/watch?v=ZB28gfSmz1Y

Schizophrenia Video (4 mins)

1. Patient’s thought disturbance and example

2. Perceptual disturbance and example

3. Diagnosis and give reason

4. Management and reason

OSCE 27

Interactive

Came to see GP due to excessive hand washing. She went to see dermatologist due to afraid of
dermatitis.

Take a history for her symptoms and make a diagnosis to her complaint

339
340 Oceanic Series End of Posting Compilation

OSCE 28

Video : https://www.youtube.com/watch?v=zA-fqvC02oM&t=63s

Bipolar in manic phase

1. Three good interview skill by Dr. Betty

2. Three bad interview skill by Dr. Betty

3. Three patient’s abnormal attitude

4. Three patient’s speech

5. Diagnosis and give reason

OSCE 29

(Interactive-Panic Disorder)

1. Intro (2m)

2. (pt presented with palpitations)

-ask about the rate

-is there any pain

-any discomfort

-radiates/not

340
341 Oceanic Series End of Posting Compilation

3. Symptoms

- how long has the symptoms started? (onset)

- how it happens from the beginning

- how frequent you get the feeling

- each frequency – how long does it last?

(panic attack will go to a peak)

-so that peak, how long does it last long?

4. During attack

-do you breathe fast?

-do you feel numbness or tingling sensation?

-any choking?

-fear of dying?

-shaking or trembling?

5. (confirmed that nothing wrong with heart , based on ix,)

-give SSRI – need to give 2-3 weeks to start acting

-teach pt, before symptoms comes, do de breathing : can reduce intensity of panic

-give Alprazolam (short acting) not Lorazepam

-(don’t say want to do CBT, just do de breathing)

341
342 Oceanic Series End of Posting Compilation

OSCE 30

(Video: Dementia)

1. what did the wife tell the doctor about Mac symptoms? Give 4.

-forgetfulness

-lose temper

2. 3 salient behaviors change in Mac

-over confident

-personality change

3. 5 MMSE component:

- Orientation

- Registration

- Attention

- Recall

- Naming

4. 5 depressive symptoms asked by the Dr: (SAD A FACES / SIGECAPS)

- Any feeling very happy or very sad

- Any abnormal sleeping pattern

- Any decrease in activity

- Any appetite change

- Any lose in concentration

342
343 Oceanic Series End of Posting Compilation

5. Final diagnosis – Alzheimer Dementia

6. Differential diagnosis

- Frontotemporal Dementia

-BMD (manic phase)

OSCE 31

Mrs. B was assessed on her cognitive functions by the doctor.

Instruction:

Watch the video attentively and answer all the questions. You can pause at any section that is
necessary to get the answers.

https://www.youtube.com/watch?v=jejkdRotqrc

A. Give TWO items that we missed out in the assessment of “Time Orientation” (2 marks)

B. Give TWO errors made by the interviewer regarding the test items on “Memory
Registration” (2 marks)

C. State your score on “Memory Registration” and “5 – Minutes Recall” (2 marks)


Registration score:

343
344 Oceanic Series End of Posting Compilation

5 – minutes recall:

D. Name the test item that was missed out between “Memory Registration” and “5 –
minutes Recall” and give any TWO test options that can be used (3 marks)
Test item missed:
Test Options:

E. State your score on the test item “Obey the command” (1 mark)
“Obey the command” score:

F. State your score on the test item “Copy the design” and give TWO reasons (3 marks)
“Copy the design” score:
Reasons:

G. State your score on “Clock drawing test” and give FOUR reasons (5 marks)
“Clock drawing test” score:
Reasons:

H. State your score on “Write a complete sentence” and give ONE reason (2 marks)
“Write a complete sentence” score:
Reason:

344
345 Oceanic Series End of Posting Compilation

OSCE 32

The doctor is breaking the bad news about the results of breast lump biopsy to her patient,

Instruction:

View the video carefully and answer the following questions

https://www.youtube.com/watch?v=HWAZnhCuAeE

A. List FOUR positive attributes shown by the doctor during the introduction phase. (4
marks)
B. List FOUR reactions observed in the patient on hearing the bad news. (4 marks)
C. List FOUR errors made by the doctor before breaking the bad news. (4 marks)
D. List FIVE good gestures made by the doctor while giving information. (5 marks)
E. List THREE relevant information given by the doctor about the management of the
condition. (3 marks)

345
346 Oceanic Series End of Posting Compilation

OSCE 33

John presented to the General Practitioner Dr. Betty with symptoms suggestive of major mental
disorder.

Instruction:

Watch the video carefully and answer all the questions.

You may replay the video if needed.

https://www.youtube.com/watch?v=zA-fqvC02oM

John Riley is being interviewed by his General Practitioner Dr. Betty

A. Give THREE good interview skills shown by Dr. Betty (3 marks)


o She greet John by name,
o Explain the purpose of the interview
o Friendly in manner
o She able to get relevant John’s biodata
B. Give THREE bad interview skills shown by Dr. Betty (3 marks)
o Impatient
o She interrupt the patient
o She did not listen well to the John
o She respond to John by making faces towards him

346
347 Oceanic Series End of Posting Compilation

C. List TWO John’s appearance and behavior observed during the interview (4 marks)
No. Appearance Behavior
i.
ii.

0 Overfriendly
o Bad attitude towards Dr
o He is not cooperative
o He disrespect
o He use vulgar words
o He wore colorful shirt
o Uncombed hair
o Distracted
o Making jokes with the Dr
o disinhibitant
D. Give THREE signs observed in his speech (3 marks)
o Pressure of speech
o Irrelevant speech
o Flight of ideas
o Circumstantiality
o Talkative
E. Give THREE examples of thought disturbance expressed by John (3 marks)
F. State your diagnosis in full and give THREE reasons (4 marks)
Bipolar Disorder Type II in Manic Phase
elated mood, high energy, hyperactive, goal directed,
grandiose, distractibility

347
348 Oceanic Series End of Posting Compilation

OSCE 34

Emma presented with some psychological problems that is affecting her life. The General
Practitioner is interviewing her in order to make a diagnosis.

Instruction:

Watch the video carefully and answer ALL the questions. You are allowed to replay the video if
you wish to view again.

Emma has some psychological problems that is distressing her. She seeks help from a General
Practitioner.

A. List FOUR distressing physical and psychological symptoms stated by Emma (4 marks)
No. Physical Symptoms Psychological Symptoms
i.
ii.
iii.
iv.

B. State TWO differential diagnoses and give ONE reasons each (4 marks)
No. Diagnosis Reason
i.
ii.

C. List TWO DSM Axis V complications experienced by Emma (2 marks)


D. List FOUR positive attributes shown by the doctor during the interview (2 marks)

348
349 Oceanic Series End of Posting Compilation

E. List FOUR management strategies with your reasons. (8 marks)


Management Reason
Pharmacological
i.
ii.
Psychological

iii.
iv.

OSCE 35

Elderly woman cannot get over her husband’s death

1. 2 differential diagnoses with reasons


• Pathological grief disorder (more than one year, feel her husband presence,
want to be with her husband)
• Major depressive disorder (low mood, fatigue, poor sleep)
2. Suicide risk level
• Low level because has no plan or attempt and has to live for her children’
sake
3. 3 comments for the doctor
• Good eye contact
• Listen well
• Taking good history
• Systematic
4. Therapy intervention
• Bereavement therapy
• Support therapy
• CBT
5. Medications and example
• SSRI
• Benzodiazepine

349
350 Oceanic Series End of Posting Compilation

OSCE 36

Instruction: elicit alcohol dependence syndrome and problem drinking

Main point for questions alcohol dependence syndrome

• Rapid reinstatement after period of abstinence


• Persistent drinking despite harmful effect
• Primacy of drinking (alcohol is the top priority in life?)
• Relieve drinking
• Withdrawal symptoms
• Compulsive drinking
• Tolerance
• Problem drinking: CAGE

OSCE 37

Mrs Jones, 65 years old woman has sudden change of behaviour. She has underlying
Alzheimer’s disease

1. Psychological features
• Irritable
• Shouting
• Confusion
• Visual hallucination
• Auditory hallucination
• Paranoid delusion

2. 3 physical features
• Ecchymosis around the eyes
• Shuffling gait
• Unkempt appearance

350
351 Oceanic Series End of Posting Compilation

3. Reason of changing behaviour


• Dementia in pain
• Due to fall
• Side effect of drug
• Sundowning
• UTI
4. 4 reasons of frequent fall
• Arthritis
• EPS
• Side effect of drug
• Parkinson gait
• Postural hypotension
5. Diagnosis: Delirium in dementia
6. Reason: acute onset, smelly urine, psychotic symptoms

OSCE 38

Patient diagnosed with major depressive disorder and need to start treatment but reluctant to
take medication

What should do: Convince the patient. Use layman term

• List group of medication: SSRI, SNRI, TCA


• Explain side effect
• Insight

351
352 Oceanic Series End of Posting Compilation

OSCE 39

Alex, 22 years old has schizophrenia. He was prescribed with olanzapine 10 mg. You need to
give information regarding the benefit, side effect, and how does the medication work to his
parents.

OSCE 39 – video (delirium)

First part

1. Give 3 symptoms based on the video


-could not identify the wife
-visual hallucination
-confused
2. Give the likely diagnosis
-hypoactive delirium

Second part

1. Give 3 symptoms based on the video

-aggressive

-hallucination

2. Give the likely diagnosis


-hyperactive delirium

3. Give the overall diagnosis


-Mixed delirium with underlying dementia

352
353 Oceanic Series End of Posting Compilation

OSCE 39 -video (hypochondriasis)

1. Five symptoms that she has


2. What is her main concern?
3. Give 2 likely diagnosis and reason
4. Give 2 management

OSCE 40 Bipolar Mood disorder

1. State 4 question to assess the perceptual disturbance


2. State 2 question to access Thought disturbance
3. Give the final Diagnosis
OSCE 41 Schneider 1st Rank Schizophrenia

OSCE 42 Interactive Information Giving – ECT, Anti- depressant

OSCE 43 Interactive Major depressive disorder

353
354 Oceanic Series End of Posting Compilation

SBA

THEME 1: EPIDEMIOLOGY.

Schizophrenia is a global public health burden. Choose the CORRECT LIFETIME

PREVALENCE of schizophrenia in the community.

a. 0.5-0.9 %

b. 1.0-1.5%

c. 1.6-2.0%

d. 2.1-2.5%

e. 2.6-3.0%

THEME 2: PSYCHIATRY HISTORY TAKING

A collaborative history from an informant is important in a psychiatry interview. Select

the INCORRECT information from the option stated below.

a. Patient’s premorbid personality

b. Patient’s subjective mood

c. Patient’s home circumstances

d. Patient’s substance used and abused

e. Patient’s past history of self-harm

354
355 Oceanic Series End of Posting Compilation

THEME 3: PSYCHIATRIC EMERGENCIES.

A 30 years old man was brought to the emergency room by the police for being

aggressive and picking fight in the public. Identify the INCORRECT SIGN OF AN

IMPENDING VIOLENCE.

a. Insisting for a hot drink

b. Raising his voice

c. Motor restlessness

d. Sweating profusely

e. Verbally abusive towards the interviewer

THEME 4: DISORGANIZED SPEECH.

A 25 years old man with schizophrenia was interviewed in a psychiatric clinic. State

the INCORRECT answer regarding DISORGANIZED SPEECH.

a. Word salad

b. Thought block

c. Perseveration

d. Circumstantiality

e. Mutism

355
356 Oceanic Series End of Posting Compilation

THEME 5: DEMENTIA.

A 56 years old lady was presented to the memory clinic with impaired ability to carry

out IADL for the past one year. Choose the INCORRECT statement regarding the

PATHOLOGY OF ALZHEIMER’S DISEASE.

a. The gene for Beta Amyloid is located in chromosome 18.

(The gene for Beta Amyloid supposed to be located in Chromosome 21)

b. Neuro imaging typically shows wide sulci and large ventricles.

c. Amyloid plaques are key histological finding.

d. Neuro fibrillary tangles are commonly seen in the cerebral cortex.

e. Significant decrease in the concentration of Acetylcholine at the neuronal

synaptic cleft.

THEME 6: DELIRIUM.

A 45 years old man with acute encephalitis became restless and disorientated. Select

the INCORRECT statement regarding DELIRIUM.

a. The term delirium and acute confusional state are the same.

b. Symptoms are usually worsened in the morning. (Worsened In the evening)

c. Visual hallucinations are more common that auditory hallucinations.

d. Impaired level of consciousness is a defining clinical feature.

e. Onset is usually acute over hours to days.

356
357 Oceanic Series End of Posting Compilation

THEME 7: OBSESSIVE-COMPULSIVE DISORDER (OCD).

Obsessive-compulsive disorder (OCD) is a debilitating psychological disorder. Choose

a CORRECT statement with respect to OCD.

a. OCD is usually an acute condition with recovering over weeks to a few months.

b. The most common comorbidity is an eating disorder.

c. Compulsive ritual always involved physical acts. E.g., checking locks.

d. Patient acknowledges that their OCD thoughts are excessive and

unreasonable.

e. CBT is of limited value in patient with mild OCD.

THEME 8: ANXIETY DISORDER.

Many physical diseases are commonly associated with anxiety symptoms. Choose the

INCORRECT answer.

a. Hyperthyroidism

b. Hypothyroidism

c. Pheochromocytoma

d. irritable bowel syndrome (IBS)

e. Myocardial Infarction

357
358 Oceanic Series End of Posting Compilation

THEME 9: POST-TRAUMATIC DISORDER (PTSD).

A 30 years old ICU nurse witnessed several COVID-19 victims dying of severe

respiratory distress. Select the INCORRECT answer about PTSD.

a. PTSD is common in male gender

b. Childhood abuse

c. Witnessing traumatic events

d. Can occur at any age

e. Poor social support

20. THEME 10: LEARNING DISABILITIES.

A 20 years old man with a history of learning disabilities was recommended to get an

OKU card by a medical social worker. Identify the INCORRECT statement regarding

LEARNING DISABILITIES.

Most patients with learning disabilities live with their families.

B. CBT can be successfully used in all patients with learning disabilities.

(Not all patients)

C. Periodical medical checkups are recommended to be done by primary care

doctor.

D. Patients with learning disabilities are incapable of any type of employment.

E. Medications used to treat specific behavior symptoms are effective.

358

You might also like